[REPOST] The Non-Libertarian FAQ

[This is a repost of the Non-Libertarian FAQ (aka “Why I Hate Your Freedom”), which I wrote about five years ago and which used to be hosted on my website. It no longer completely reflects my current views. I don’t think I’ve switched to believing anything on here is outright false, but I’ve moved on to different ways of thinking about certain areas. I’m reposting it by popular request and for historical interest only. I’ve made some very small updates, mostly listing rebuttals that came out over the past few years. I haven’t updated the statistics and everything is accurate as of several years ago. I seem to have lost the sources of my images, and I’m sorry; if I’ve used an image of yours, please let me know and I’ll cite you.]

Contents

0. Introduction

A. Economic Issues

1. Externalities
2. Coordination Problems
3. Irrational Choices
4. Lack of Information

B. Social Issues

5. Just Desserts and Social Mobility
6. Taxation

C. Political Issues

7. Competence of Government
8. Health Care
9. Prison Privatization
10. Gun Control
11. Education

D. Moral Issues

12. Moral Systems
13. Rights and Heuristics

E. Practical Issues

14. Slippery Slopes
15. Strategic Activism
16. Miscellaneous and Meta

Introduction

0.1: Are you a statist?

No.

Imagine a hypothetical country split between the “tallists”, who think only tall people should have political power, and the “shortists”, who believe such power should be reserved for the short.

If we met a tallist, we’d believe she was silly – but not because we favor the shortists instead. We’d oppose the tallists because we think the whole dichotomy is stupid – we should elect people based on qualities like their intelligence and leadership and morality. Knowing someone’s height isn’t enough to determine whether they’d be a good leader or not.

Declaring any non-libertarian to be a statist is as silly as declaring any non-tallist to be a shortist. Just as we can judge leaders on their merits and not on their height, so people can judge policies on their merits and not just on whether they increase or decrease the size of the state.

There are some people who legitimately believe that a policy’s effect on the size of the state is so closely linked to its effectiveness that these two things are not worth distinguishing, and so one can be certain of a policy’s greater effectiveness merely because it seems more libertarian and less statist than the alternative. Most of the rest of this FAQ will be an attempt to disprove this idea and assert that no, you really do have to judge the individual policy on its merits.

0.2: Do you hate libertarianism?

No.

To many people, libertarianism is a reaction against an over-regulated society, and an attempt to spread the word that some seemingly intractable problems can be solved by a hands-off approach. Many libertarians have made excellent arguments for why certain libertarian policies are the best options, and I agree with many of them. I think this kind of libertarianism is a valuable strain of political thought that deserves more attention, and I have no quarrel whatsoever with it and find myself leaning more and more in that direction myself.

However, there’s a certain more aggressive, very American strain of libertarianism with which I do have a quarrel. This is the strain which, rather than analyzing specific policies and often deciding a more laissez-faire approach is best, starts with the tenet that government can do no right and private industry can do no wrong and uses this faith in place of more careful analysis. This faction is not averse to discussing politics, but tends to trot out the same few arguments about why less regulation has to be better. I wish I could blame this all on Ayn Rand, but a lot of it seems to come from people who have never heard of her. I suppose I could just add it to the bottom of the list of things I blame Reagan for.

To the first type of libertarian, I apologize for writing a FAQ attacking a caricature of your philosophy, but unfortunately that caricature is alive and well and posting smug slogans on Facebook.

0.3: Will this FAQ prove that government intervention always works better than the free market?

No, of course not.

Actually, in most cases, you won’t find me trying to make a positive proof of anything. I believe that deciding on, for example, an optimal taxation policy takes very many numbers and statistical models and other things which are well beyond the scope of this FAQ, and may well have different answers at different levels and in different areas.

What I want to do in most cases is not prove that the government works better than the free market, or vice versa, but to disprove theories that say we can be absolutely certain free market always works better than government before we even investigate the issue. After that, we may still find that this is indeed one of the cases where the free market works better than the government, but we will have to prove it instead of viewing it as self-evident from first principles.

0.4: Why write a Non-Libertarian FAQ? Isn’t statism a bigger problem than libertarianism?

Yes. But you never run into Stalinists at parties. At least not serious Stalinists over the age of twenty-five, and not the interesting type of parties. If I did, I guess I’d try to convince them not to be so statist, but the issue’s never come up.

But the world seems positively full of libertarians nowadays. And I see very few attempts to provide a complete critique of libertarian philosophy. There are a bunch of ad hoc critiques of specific positions: people arguing for socialist health care, people in favor of gun control. But one of the things that draws people to libertarianism is that it is a unified, harmonious system. Unlike the mix-and-match philosophies of the Democratic and Republican parties, libertarianism is coherent and sometimes even derived from first principles. The only way to convincingly talk someone out of libertarianism is to launch a challenge on the entire system.

There are a few existing documents trying to do this (see Mike Huben’s Critiques of Libertarianism and Mark Rosenfelder’s What’s (Still) Wrong With Libertarianism for two of the better ones), but I’m not satisfied with any of them. Some of them are good but incomplete. Others use things like social contract theory, which I find nonsensical and libertarians find repulsive. Or they have an overly rosy view of how consensual taxation is, which I don’t fall for and which libertarians definitely don’t fall for.

The main reason I’m writing this is that I encounter many libertarians, and I need a single document I can point to explaining why I don’t agree with them. The existing anti-libertarian documentation makes too many arguments I don’t agree with for me to feel really comfortable with it, so I’m writing this one myself. I don’t encounter too many Stalinists,
so I don’t have this problem with them and I don’t see any need to write a rebuttal to their position.

If you really need a pro-libertarian FAQ to use on an overly statist friend, Google suggests The Libertarian FAQ.

0.5: How is this FAQ structured?

I’ve divided it into three main sections. The first addresses some very abstract principles of economics. They may not be directly relevant to politics, but since most libertarian philosophies start with abstract economic principles, a serious counterargument has to start there also. Fair warning: there are people who can discuss economics without it being INCREDIBLY MIND-NUMBINGLY BORING, but I am not one of them.

The second section deals with more concrete economic and political problems like the tax system, health care, and criminal justice.

The third section deals with moral issues, like whether it’s ever permissible to initiate force. Too often I find that if I can convince a libertarian that government regulation can be effective, they respond that it doesn’t matter because it’s morally repulsive, and then once I’ve finished convincing them it isn’t, they respond that it never works anyway. By having sections dedicated to both practical and moral issues, I hope to make that sort of bait-and-switch harder to achieve, and to allow libertarians to evaluate the moral and practical arguments against their position in whatever order they find appropriate.

Part A: Economic Issues

The Argument:

In a free market, all trade has to be voluntary, so you will never agree to a trade unless it benefits you.

Further, you won’t make a trade unless you think it’s the best possible trade you can make. If you knew you could make a better one, you’d hold out for that. So trades in a free market are not only better than nothing, they’re also the best possible transaction you could make at that time.

Labor is no different from any other commercial transaction in this respect. You won’t agree to a job unless it benefits you more than anything else you can do with your time, and your employer won’t hire you unless it benefits her more than anything else she can do with her money. So a voluntarily agreed labor contract must benefit both parties, and must do so more than any other alternative.

If every trade in a free market benefits both parties, then any time the government tries to restrict trade in some way, it must hurt both parties. Or, to put it another way, you can help someone by giving them more options, but you can’t help them by taking away options. And in a free market, where everyone starts with all options, all the government can do is take options away.

The Counterargument:

This treats the world as a series of producer-consumer dyads instead of as a system in which every transaction affects everyone else. Also, it treats consumers as coherent entities who have specific variables like “utility” and “demand” and know exactly what they are, which doesn’t always work.

In the remainder of this section, I’ll be going over several ways the free market can fail and several ways a regulated market can overcome those failures. I’ll focus on four main things: externalities, coordination problems, irrational choice, and lack of information.

I did warn you it would be mind-numbingly boring.

1. Externalities

1.1: What is an externality?

An externality is when I make a trade with you, but it has some accidental effect on other people who weren’t involved in the trade.

Suppose for example that I sell my house to an amateur wasp farmer. Only he’s not a very good wasp farmer, so his wasps usually get loose and sting people all over the neighborhood every couple of days.

This trade between the wasp farmer and myself has benefited both of us, but it’s harmed people who weren’t consulted; namely, my neighbors, who are now locked indoors clutching cans of industrial-strength insect repellent. Although the trade was voluntary for both the wasp farmer and myself, it wasn’t voluntary for my neighbors.

Another example of externalities would be a widget factory that spews carcinogenic chemicals into the air. When I trade with the widget factory I’m benefiting – I get widgets – and they’re benefiting – they get money. But the people who breathe in the carcinogenic chemicals weren’t consulted in the trade.

1.2: But aren’t there are libertarian ways to solve externalities that don’t involve the use of force?

To some degree, yes. You can, for example, refuse to move into any neighborhood unless everyone in town has signed a contract agreeing not to raise wasps on their property.

But getting every single person in a town of thousands of people to sign a contract every time you think of something else you want banned might be a little difficult. More likely, you would want everyone in town to unanimously agree to a contract saying that certain things, which could be decided by some procedure requiring less than unanimity, could be banned from the neighborhood – sort of like the existing concept of neighborhood associations.

But convincing every single person in a town of thousands to join the neighborhood association would be near impossible, and all it would take would be a single holdout who starts raising wasps and all your work is useless. Better, perhaps, to start a new town on your own land with a pre-existing agreement that before you’re allowed to move in you must belong to the association and follow its rules. You could even collect dues from the members of this agreement to help pay for the people you’d need to enforce it.

But in this case, you’re not coming up with a clever libertarian way around government, you’re just reinventing the concept of government. There’s no difference between a town where to live there you have to agree to follow certain terms decided by association members following some procedure, pay dues, and suffer the consequences if you break the rules – and a regular town with a regular civic government.

As far as I know there is no loophole-free way to protect a community against externalities besides government and things that are functionally identical to it.

1.3: Couldn’t consumers boycott any company that causes externalities?

Only a small proportion of the people buying from a company will live near the company’s factory, so this assumes a colossal amount of both knowledge and altruism on the part of most consumers. See also the general discussion of why boycotts almost never solve problems in the next session.

1.4: What is the significance of externalities?

They justify some environmental, zoning, and property use regulations.

2. Coordination Problems

2.1: What are coordination problems?

Coordination problems are cases in which everyone agrees that a certain action would be best, but the free market cannot coordinate them into taking that action.

As a thought experiment, let’s consider aquaculture (fish farming) in a lake. Imagine a lake with a thousand identical fish farms owned by a thousand competing companies. Each fish farm earns a profit of $1000/month. For a while, all is well.

But each fish farm produces waste, which fouls the water in the lake. Let’s say each fish farm produces enough pollution to lower productivity in the lake by $1/month.

A thousand fish farms produce enough waste to lower productivity by $1000/month, meaning none of the fish farms are making any money. Capitalism to the rescue: someone invents a complex filtering system that removes waste products. It costs $300/month to operate. All fish farms voluntarily install it, the pollution ends, and the fish farms are now making a profit of $700/month – still a respectable sum.

But one farmer (let’s call him Steve) gets tired of spending the money to operate his filter. Now one fish farm worth of waste is polluting the lake, lowering productivity by $1. Steve earns $999 profit, and everyone else earns $699 profit.

Everyone else sees Steve is much more profitable than they are, because he’s not spending the maintenance costs on his filter. They disconnect their filters too.

Once four hundred people disconnect their filters, Steve is earning $600/month – less than he would be if he and everyone else had kept their filters on! And the poor virtuous filter users are only making $300. Steve goes around to everyone, saying “Wait! We all need to make a voluntary pact to use filters! Otherwise, everyone’s productivity goes down.”

Everyone agrees with him, and they all sign the Filter Pact, except one person who is sort of a jerk. Let’s call him Mike. Now everyone is back using filters again, except Mike. Mike earns $999/month, and everyone else earns $699/month. Slowly, people start thinking they too should be getting big bucks like Mike, and disconnect their filter for $300 extra profit…

A self-interested person never has any incentive to use a filter. A self-interested person has some incentive to sign a pact to make everyone use a filter, but in many cases has a stronger incentive to wait for everyone else to sign such a pact but opt out himself. This can lead to an undesirable equilibrium in which no one will sign such a pact.

The most profitable solution to this problem is for Steve to declare himself King of the Lake and threaten to initiate force against anyone who doesn’t use a filter. This regulatory solution leads to greater total productivity for the thousand fish farms than a free market could.

The classic libertarian solution to this problem is to try to find a way to privatize the shared resource (in this case, the lake). I intentionally chose aquaculture for this example because privatization doesn’t work. Even after the entire lake has been divided into parcels and sold to private landowners (waterowners?) the problem remains, since waste will spread from one parcel to another regardless of property boundaries.

2.1.1: Even without anyone declaring himself King of the Lake, the fish farmers would voluntarily agree to abide by the pact that benefits everyone.

Empirically, no. This situation happens with wild fisheries all the time. There’s some population of cod or salmon or something which will be self-sustaining as long as it’s not overfished. Fishermen come in and catch as many fish as they can, overfishing it. Environmentalists warn that the fishery is going to collapse. Fishermen find this worrying, but none of them want to fish less because then their competitors will just take up the slack. Then the fishery collapses and everyone goes out of business. The most famous example is the Collapse of the Northern Cod Fishery, but there are many others in various oceans, lakes, and rivers.

If not for resistance to government regulation, the Canadian governments could have set strict fishing quotas, and companies could still be profitably fishing the area today. Other fisheries that do have government-imposed quotas are much more successful.

2.1.2: I bet [extremely complex privatization scheme that takes into account the ability of cod to move across property boundaries and the migration patterns of cod and so on] could have saved the Atlantic cod too.

Maybe, but left to their own devices, cod fishermen never implemented or recommended that scheme. If we ban all government regulation in the environment, that won’t make fishermen suddenly start implementing complex privatization schemes that they’ve never implemented before. It will just make fishermen keep doing what they’re doing while tying the hands of the one organization that has a track record of actually solving this sort of problem in the real world.

2.2: How do coordination problems justify environmental regulations?

Consider the process of trying to stop global warming. If everyone believes in global warming and wants to stop it, it’s still not in any one person’s self-interest to be more environmentally conscious. After all, that would make a major impact on her quality of life, but a negligible difference to overall worldwide temperatures. If everyone acts only in their self-interest, then no one will act against global warming, even though stopping global warming is in everyone’s self-interest. However, everyone would support the institution of a government that uses force to make everyone more environmentally conscious.

Notice how well this explains reality. The government of every major country has publicly declared that they think solving global warming is a high priority, but every time they meet in Kyoto or Copenhagen or Bangkok for one of their big conferences, the developed countries would rather the developing countries shoulder the burden, the developing countries would rather the developed countries do the hard work, and so nothing ever gets done.

The same applies mutans mutandis to other environmental issues like the ozone layer, recycling, and anything else where one person cannot make a major difference but many people acting together can.

2.3: How do coordination problems justify regulation of ethical business practices?

The normal libertarian belief is that it is unnecessary for government to regulate ethical business practices. After all, if people object to something a business is doing, they will boycott that business, either incentivizing the business to change its ways, or driving them into well-deserved bankruptcy. And if people don’t object, then there’s no problem and the government shouldn’t intervene.

A close consideration of coordination problems demolishes this argument. Let’s say Wanda’s Widgets has one million customers. Each customer pays it $100 per year, for a total income of $100 million. Each customer prefers Wanda to her competitor Wayland, who charges $150 for widgets of equal quality. Now let’s say Wanda’s Widgets does some unspeakably horrible act which makes it $10 million per year, but offends every one of its million customers.

There is no incentive for a single customer to boycott Wanda’s Widgets. After all, that customer’s boycott will cost the customer $50 (she will have to switch to Wayland) and make an insignificant difference to Wanda (who is still earning $99,999,900 of her original hundred million). The customer takes significant inconvenience, and Wanda neither cares nor stops doing her unspeakably horrible act (after all, it’s giving her $10 million per year, and only losing her $100).

The only reason it would be in a customer’s interests to boycott is if she believed over a hundred thousand other customers would join her. In that case, the boycott would be costing Wanda more than the $10 million she gains from her unspeakably horrible act, and it’s now in her self-interest to stop committing the act. However, unless each boycotter believes 99,999 others will join her, she is inconveniencing herself for no benefit.

Furthermore, if a customer offended by Wanda’s actions believes 100,000 others will boycott Wanda, then it’s in the customer’s self-interest to “defect” from the boycott and buy Wanda’s products. After all, the customer will lose money if she buys Wayland’s more expensive widgets, and this is unnecessary – the 100,000 other boycotters will change Wanda’s mind with or without her participation.

This suggests a “market failure” of boycotts, which seems confirmed by experience. We know that, despite many companies doing very controversial things, there have been very few successful boycotts. Indeed, few boycotts, successful or otherwise, ever make the news, and the number of successful boycotts seems much less than the amount of outrage expressed at companies’ actions.

The existence of government regulation solves this problem nicely. If >51% of people disagree with Wanda’s unspeakably horrible act, they don’t need to waste time and money guessing how many of them will join in a boycott, and they don’t need to worry about being unable to conscript enough defectors to reach critical mass. They simply vote to pass a law banning the action.

2.3.1: I’m not convinced that it’s really that hard to get a boycott going. If people really object to something, they’ll start a boycott regardless of all that coordination problem stuff.

So, you’re boycotting Coke because they’re hiring local death squads to kidnap, torture, and murder union members and organizers in their sweatshops in Colombia, right?

Not a lot of people to whom I have asked this question have ever answered “yes”. Most of them had never heard of the abuses before. A few of them vaguely remembered having heard something about it, but dismissed it as “you know, multinational corporations do a lot of sketchy things.” I’ve only met one person who’s ever gone so far as to walk twenty feet further to get to the Pepsi vending machine.

If you went up to a random guy on the street and said “Hey, does hiring death squads to torture and kill Colombians who protest about terrible working conditions bother you?” 99.9% of people would say yes. So why the disconnect between words and actions? People could just be lying – they could say they cared so they sounded compassionate, but in reality it doesn’t really bother them.

But maybe it’s something more complicated. Perhaps they don’t have the brainpower to keep track of every single corporation that’s doing bad things and just how bad they are. Perhaps they’ve compartmentalized their lives and after they leave their Amnesty meetings it just doesn’t register that they should change their behaviour in the supermarket. Or perhaps the Coke = evil connection is too tenuous and against the brain’s ingrained laws of thought to stay relevant without expending extraordinary amounts of willpower. Or perhaps there’s some part of the subconscious that really is worry about that game theory and figuring it has no personal incentive to join the boycott.

And God forbid that it’s something more complicated than that. Imagine if the company that made the mining equipment that was bought by the mining company that mined the aluminum that was bought by Coke to make their cans was doing something unethical. You think you could convince enough people to boycott Coke that Coke would boycott the mining company that the mining company would boycott the equipment company that the equipment company would stop behaving unethically?

If we can’t trust people to stay off Coke when it uses death squads and when Pepsi tastes exactly the same (don’t argue with me on that one!) how can we assume people’s purchasing decisions will always act as a general moral regulatory method for the market?

2.3.2: And you really think governments can do better?

Sure seems that way. Many laws currently exist banning businesses from engaging in unethical practices. Some of these laws were passed by direct ballot. Others were passed by representatives who have incentives to usually follow the will of their constituents. So it seems fair to say that there are a lot of business practices that more than 51% of people thought should be banned.

But the very fact that a law was needed to ban them proves that those 51% of people weren’t able to organize a successful boycott. More than half of the population, sometimes much more, hated some practice so much they thought it should be illegal, yet that wasn’t enough to provide an incentive for the company to stop doing it until the law took effect.

To me, that confirms that boycotts are a very poor way of allowing people’s morals to influence corporate conduct.

2.4: How do coordination problems justify government spending on charitable causes?

Because failure to donate to a charitable cause might also be because of a coordination problem.

How many people want to end world hunger? I’ve never yet met someone who would answer with a “not me!”, but maybe some of those people are just trying to look good in front of other people, so let’s make a conservative estimate of 50%.

There’s a lot of dispute over what it would mean to “end world hunger”, all the way from “buy and ship food every day to everyone who is hungry that day” all the way to “create sustainable infrastructure and economic development such that everyone naturally produces enough food or money”. There are various estimates about how much these different definitions would cost, all the way from “about $15 billion a year” to “about $200 billion a year” – permanently in the case of shipping food, and for a decade or two in the case of promoting development.

Even if we take the highest possible estimate, it’s still well below what you would make if 50% of the population of the world donated $1/week to the cause. Now, certainly there are some very poor people in the world who couldn’t donate $1/week, but there are also some very rich people who could no doubt donate much, much more.

So we have two possibilities. Either the majority of people don’t care enough about world hunger to give a dollar a week to end it, or something else is going on.

That something else is a coordination problem. No one expects anyone else to donate a dollar a week, so they don’t either. And although somebody could shout very loudly “Hey, let’s all donate $1 a week to fight world hunger!” no one would expect anyone else to listen to that person, so they wouldn’t either.

When the government levies tax money on everyone in the country and then donates it to a charitable cause, it is often because everyone in the country supports that charitable cause but a private attempt to show that support would fall victim to coordination problems.

2.5: How do coordination problems justify labor unions and other labor regulation?

It is frequently proposed that workers and bosses are equal negotiating partners bargaining on equal terms, and only the excessive government intervention on the side of labor that makes the negotiating table unfair. After all, both need something from one another: the worker needs money, the boss labor. Both can end the deal if they don’t like the terms: the boss can fire the worker, or the worker can quit the boss. Both have other choices: the boss can choose a different employee, the worker can work for a different company. And yet, strange to behold, having proven the fundamental equality of workers and bosses, we find that everyone keeps acting as if bosses have the better end of the deal.

During interviews, the prospective employee is often nervous; the boss rarely is. The boss can ask all sorts of things like that the prospective pay for her own background check, or pee in a cup so the boss can test the urine for drugs; the prospective employee would think twice before daring make even so reasonable a request as a cup of coffee. Once the employee is hired, the boss may ask on a moment’s notice that she work a half hour longer or else she’s fired, and she may not dare to even complain. On the other hand, if she were to so much as ask to be allowed to start work thirty minutes later to get more sleep or else she’ll quit, she might well be laughed out of the company. A boss may, and very often does, yell at an employee who has made a minor mistake, telling her how stupid and worthless she is, but rarely could an employee get away with even politely mentioning the mistake of a boss, even if it is many times as unforgivable.

The naive economist who truly believes in the equal bargaining position of labor and capital would find all of these things very puzzling.

Let’s focus on the last issue; a boss berating an employee, versus an employee berating a boss. Maybe the boss has one hundred employees. Each of these employees only has one job. If the boss decides she dislikes an employee, she can drive her to quit and still be 99% as productive while she looks for a replacement; once the replacement is found, the company will go on exactly as smoothly as before.

But if the employee’s actions drive the boss to fire her, then she must be completely unemployed until such time as she finds a new job, suffering a long period of 0% productivity. Her new job may require a completely different life routine, including working different hours, learning different skills, or moving to an entirely new city. And because people often get promoted based on seniority, she probably won’t be as well paid or have as many opportunities as she did at her old company. And of course, there’s always the chance she won’t find another job at all, or will only find one in a much less tolerable field like fast food.

We previously proposed a symmetry between a boss firing a worker and a worker quitting a boss, but actually they could not be more different. For a boss to fire a worker is at most a minor inconvenience; for a worker to lose a job is a disaster. The Holmes-Rahe Stress Scale, a measure of the comparative stress level of different life events, puts being fired at 47 units, worse than the death of a close friend and nearly as bad as a jail term. Tellingly, “firing one of your employees” failed to make the scale.

This fundamental asymmetry gives capital the power to create more asymmetries in its favor. For example, bosses retain a level of control on workers even after they quit, because a worker may very well need a letter of reference from a previous boss to get a good job at a new company. On the other hand, a prospective employee who asked her prospective boss to produce letters of recommendation from her previous workers would be politely shown the door; we find even the image funny.

The proper level negotiating partner to a boss is not one worker, but all workers. If the boss lost all workers at once, then she would be at 0% productivity, the same as the worker who loses her job. Likewise, if all the workers approached the boss and said “We want to start a half hour later in the morning or we all quit”, they might receive the same attention as the boss who said “Work a half hour longer each day or you’re all fired”.

But getting all the workers together presents coordination problems. One worker has to be the first to speak up. But if one worker speaks up and doesn’t get immediate support from all the other workers, the boss can just fire that first worker as a troublemaker. Being the first worker to speak up has major costs – a good chance of being fired – but no benefits – all workers will benefit equally from revised policies no matter who the first worker to ask for them is.

Or, to look at it from the other angle, if only one worker sticks up for the boss, then intolerable conditions may well still get changed, but the boss will remember that one worker and maybe be more likely to promote her. So even someone who hates the boss’s policies has a strong selfish incentive to stick up for her.

The ability of workers to coordinate action without being threatened or fired for attempting to do so is the only thing that gives them any negotiating power at all, and is necessary for a healthy labor market. Although we can debate the specifics of exactly how much protection should be afforded each kind of coordination, the fundamental principle is sound.

2.5.1: But workers don’t need to coordinate. If working conditions are bad, people can just change jobs, and that would solve the bad conditions.

About three hundred Americans commit suicide for work-related reasons every year – this number doesn’t count those who attempt suicide but fail. The reasons cited by suicide notes, survivors and researchers investigating the phenomenon include on-the-job bullying, poor working conditions, unbearable hours, and fear of being fired.

I don’t claim to understand the thought processes that would drive someone to do this, but given the rarity and extremity of suicide, we can assume for every worker who goes ahead with suicide for work-related reasons, there are a hundred or a thousand who feel miserable but not quite suicidal.

If people are literally killing themselves because of bad working conditions, it’s safe to say that life is more complicated than the ideal world in which everyone who didn’t like their working conditions quits and get a better job elsewhere (see the next section, Irrationality).

I note in the same vein stories from the days before labor regulations when employers would ban workers from using the restroom on jobs with nine hour shifts, often ending in the workers wetting themselves. This seems like the sort of thing that provides so much humiliation to the workers, and so little benefit to the bosses, that a free market would eliminate it in a split second. But we know that it was a common policy in the 1910s and 1920s, and that factories with such policies never wanted for employees. The same is true of factories that literally locked their workers inside to prevent them from secretly using the restroom or going out for a smoking break, leading to disasters like the Triangle Shirtwaist Fire when hundreds of workers died when the building they were locked inside burnt down. And yet even after this fire, the practice of locking workers inside buildings only stopped when the government finally passed regulation against it.

3. Irrational Choices

3.1: What do you mean by “irrational choices”?

A company (Thaler, 2007, download study as .pdf) gives its employees the opportunity to sign up for a pension plan. They contribute a small amount of money each month, and the company will also contribute some money, and overall it ends up as a really good deal for the employees and gives them an excellent retirement fund. Only a small minority of the employees sign up.

The libertarian would answer that this is fine. Although some outsider might condescendingly declare it “a really good deal”, the employees are the most likely to understand their own unique financial situation. They may have a better pension plan somewhere else, or mistrust the company’s promises, or expect not to need much money in their own age. For some outsider to declare that they are wrong to avoid the pension plan, or worse to try to force them into it for their own good, would be the worst sort of arrogant paternalism, and an attack on the employees’ dignity as rational beings.

Then the company switches tactics. It automatically signs the employees up for the pension plan, but offers them the option to opt out. This time, only a small minority of the employees opt out.

That makes it very hard to spin the first condition as the employees rationally preferring not to participate in the pension plan, since the second condition reveals the opposite preference. It looks more like they just didn’t have the mental energy to think about it or go through the trouble of signing up. And in the latter condition, they didn’t have the mental energy to think about it or go through the trouble of opting out.

If the employees were rationally deciding whether or not to sign up, then some outsider regulating their decision would be a disaster. But if the employees are making demonstrably irrational choices because of a lack of mental energy, and if people do so consistently and predictably, then having someone else who has considered the issue in more depth regulate their choices could lead to a better outcome.

3.1.1: So what’s going on here?

Old-school economics assumed choice to be “revealed preference”: an individual’s choices will invariably correspond to their preferences, and imposing any other set of choices on them will result in fewer preferences being satisfied.

In some cases, economists have gone to absurd lengths to defend this model. For example, Bryan Caplan says that when drug addicts say they wish that they could quit drugs, they must be lying, since they haven’t done so. Seemingly unsuccessful attempts to quit must be elaborate theater, done to convince other people to continue supporting them, while they secretly enjoy their drugs as much as ever.

But the past fifty years of cognitive science have thoroughly demolished this “revealed preference” assumption, showing that people’s choices result from a complex mix of external compulsions, internal motivations, natural biases, and impulsive behaviors. These decisions usually approximate fulfilling preferences, but sometimes they fail in predictable and consistent ways.

The field built upon these insights is called “behavioral economics”, and you can find more information in books like Judgment Under Uncertainty, Cognitive Illusions, and Predictably Irrational, or on the website Less Wrong.

3.2: Why does this matter?

The gist of this research, as it relates to the current topic, is that people don’t always make the best choice according to their preferences. Sometimes they consistently make the easiest or the most superficially attractive choice instead. It may be best not to think of them as a “choice” at all, but as a reflexive reaction to certain circumstances, which often but not always conforms to rationality.

Such possibilities cast doubt on the principle that every trade that can be voluntarily made should be voluntarily made.

If people’s decisions are not randomly irrational, but systematically irrational in predictable ways, that raises the possibility that people who are aware of these irrationalities may be able to do better than the average person in particular fields where the irrationalities are more common, raising the possibility that paternalism can sometimes be justified.

3.2.1: Why should the government protect people from their own irrational choices?

By definition of “irrational”, people will be happier and have more of their preferences satisfied if they do not make irrational choices. By the principles of the free market, as people make more rational decisions the economy will also improve.

If you mean this question in a moral sense, more like “How dare the government presume to protect me from my own irrational choices!”, see the section on Moral Issues.

3.2.2: What is the significance of predictably irrational behavior?

It justifies government-mandated pensions, some consumer safety and labor regulations, advertising regulations, concern about addictive drugs, and public health promotion, among other things.

4. Lack of Information

4.1: What do you mean by “lack of information”?

Many economic theories start with the assumption that everyone has perfect information about everything. For example, if a company’s products are unsafe, these economic theories assume consumers know the product is unsafe, and so will buy less of it.

No economist literally believes consumers have perfect information, but there are still strong arguments for keeping the “perfect information” assumption. These revolve around the idea that consumers will be motivated to pursue information about things that are important to them. For example, if they care about product safety, they will fund investigations into product safety, or only buy products that have been certified safe by some credible third party. The only case in which a consumer would buy something without information on it is if the consumer had no interest in the information, or wasn’t willing to pay as much for the information as it would cost, in which case the consumer doesn’t care much about the information anyway, and it is a success rather than a failure of the market that it has not given it to her.

In nonlibertarian thought, people care so much about things like product safety and efficacy, or the ethics of how a product is produced, that the government needs to ensure them. In libertarian thought, if people really care about product safety, efficacy and ethics, the market will ensure them itself, and if they genuinely don’t care, that’s okay too.

4.1.1: And what’s wrong with the libertarian position here?

Section 5 describes how we can sometimes predict when people will make irrational choices. One of the most consistent irrational choices people make is buying products without spending as much effort to gather information as the amount they care about these things would suggest. So in fact, the nonlibertarians are right: if there were no government regulation, people who care a lot about things like safety and efficacy would consistently be stuck with unsafe and ineffective products, and the market would not correct these failures.

4.2: Is this really true? Surely people would investigate the safety, ethics, and efficacy of the products they buy.

Below follows a list of statements about products. Some are real, others are made up. Can you identify which are which?

1. Some processed food items, including most Kraft cheese products, contain methylarachinate, an additive which causes a dangerous anaphylactic reaction in 1/31000 people who consume it. They have been banned in Canada, but continue to be used in the United States after intense lobbying from food industry interests.

2. Commonly used US-manufactured wood products, including almost all plywood, contain formaldehyde, a compound known to cause cancer. This has been known in scientific circles for years, but was only officially reported a few months ago because of intense chemical industry lobbying to keep it secret. Formaldehyde-containing wood products are illegal in the EU and most other developed nations.

3. Total S.A., an oil company that owns fill-up stations around the world, sometimes uses slave labor in repressive third-world countries to build its pipelines and oil wells. Laborers are coerced to work for the company by juntas funded by the corporation, and are shot or tortured if they refuse. The company also helps pay for the military muscle needed to keep the juntas in power.

4. Microsoft has cooperated with the Chinese government by turning over records from the Chinese equivalents of its search engine “Bing” and its hotmail email service, despite knowing these records would be used to arrest dissidents. At least three dissidents were arrested based on the information and are currently believed to be in jail or “re-education” centers.

5. Wellpoint, the second largest US health care company, has a long record of refusing to provide expensive health care treatments promised in some of its plans by arguing that their customers have violated the “small print” of the terms of agreement; in fact they make it so technical that almost all customers violate them unknowingly, then only cite the ones who need expensive treatment. Although it has been sued for these practices at least twice, both times it has used its legal muscle to tie the cases up in court long enough that the patients settled for an undisclosed amount believed to be fraction of the original benefits promised.

6. Ultrasonic mosquito repellents like those made by GSI, which claim to mimic frequencies produced by the mosquito’s natural predator, the bat, do not actually repel mosquitoes. Studies have shown that exactly as many mosquitoes inhabit the vicinity of such a mosquito repellent as anywhere else.

7. Listerine (and related mouth washes) probably do not eliminate bad breath. Although it may be effective at first, in the long term it generally increases bad breath by drying out the mouth and inhibiting the salivary glands. This may also increase the population of dental bacteria. Most top dentists recommend avoiding mouth wash or using it very sparingly.

8. The most popular laundry detergents, including most varieties of Tide and Method, have minimal to zero ability to remove stains from clothing. They mostly just makes clothing smell better when removed from the laundry. Some of the more expensive alkylbenzenesulfonate detergents have genuine stain-removing action, but aside from the cost, these detergents have very strong smells and are unpopular.

4.2.1: Okay, I admit I’m not sure of most of these. What’s your point?

This is a complicated FAQ about complicated philosophical issues. Most likely its readers are in the top few percentiles in terms of intelligence and education.

And we live in a world where there are many organizations, both private and governmental, that exist to evaluate products and disseminate information about their safety.

And all of the companies and products above are popular ones that most American consumers have encountered and had to make purchasing decisions about. I tried to choose safety issues that were extremely serious and carried significant risks of death, and ethical issues involving slavery and communism, which would be of particular importance to libertarians.

If the test was challenging, it means that the smartest and best-educated people in a world full of consumer safety and education organizations don’t bother to look up important life-or-death facts specifically tailored to be relevant to them about the most popular products and companies they use every day.

And if that’s the case, why would you believe that less well-educated people in a world with less consumer safety information trying to draw finer distinctions between more obscure products will definitely seek out the consumer information necessary allows them to avoid unsafe, unethical, or ineffective products?

The above test is an attempt at experimental proof that people don’t seek out even the product information that is genuinely important to them, but instead take the easy choice of buying whatever’s convenient based on information they get from advertising campaigns and the like.

4.2.2: Fine, fine, what are the answers to the test?

Four of them are true and four of them are false, but I’m not saying which are which, in the hopes that people will observe their own thought processes when deciding whether or not it’s worth looking up.

4.2.3: Right, well of course people don’t look up product information now because the government regulates that for them. In a real libertarian society, they would be more proactive.

All of the four true items on the test above are true in spite of government regulation. Clearly, there are still significant issues even in a regulated environment.

If you honestly believe you have no incentive to look up product information because you trust the government to take care of that, then you’re about ten times more statist than I am, and I’m the guy writing the Non-Libertarian FAQ.

4.3: What other unexpected consequences might occur without consumer regulation?

It could destroy small business.

In the absence of government regulation, you would have to trust corporate self-interest to regulate quality. And to some degree you can do that. Wal-Mart and Target are both big enough and important enough that if they sold tainted products, it would make it into the newspaper, there would be a big outcry, and they would be forced to stop. One could feel quite safe shopping at Wal-Mart.

But suppose on the way to Wal-Mart, you see a random mom-and-pop store that looks interesting. What do you know about its safety standards? Nothing. If they sold tainted or defective products, it would be unlikely to make the news; if it were a small enough store, it might not even make the Internet. Although you expect the CEO of Wal-Mart to be a reasonable man who understands his own self-interest and who would enforce strict safety standards, you have no idea whether the owner of the mom-and-pop store is stupid, lazy, or just assumes (with some justification) that no one will ever notice his misdeeds. So you avoid the unknown quantity and head to Wal-Mart, which you know is safe.

Repeated across a million people in a thousand cities, big businesses get bigger and small businesses get unsustainable.

4.4: What is the significance of lack of information?

It justifies some consumer and safety regulations, and the taxes necessary to pay for them.

Part B: Social Issues

The Argument:

Those who work hardest (and smartest) should get the most money. Not only should we not begrudge them that money, but we should thank them for the good they must have done for the world in order to satisfy so many consumers.

People who do not work hard should not get as much money. If they want more money, they should work harder. Getting more money without working harder or smarter is unfair, and indicative of a false sense of entitlement.

Unfortunately, modern liberal society has internalized the opposite principle: that those who work hardest are greedy people who must have stolen from those who work less hard, and that we should distrust them at until they give most of their ill-gotten gains away to others. The “progressive” taxation system as it currently exists serves this purpose.

This way of thinking is not only morally wrong-headed, but economically catastrophic. Leaving wealth in the hands of the rich would “make the pie bigger”, allowing the extra wealth to “trickle down” to the poor naturally.

The Counterargument:

Hard work and intelligence are contributory factors to success, but depending on the way you phrase the question, you find you need other factors to explain between one-half and nine-tenths of the difference in success within the United States; within the world at large the numbers are much higher.

If we think factors other than hard work and intelligence determining success are “unfair”, then most of Americans’ life experiences are determined by “unfair” factors.

Although it would be overly ambitious to want to completely eliminate all unfairness, we know that most other developed countries have successfully eliminated many of the most glaring types of unfairness, and reaped benefits greater than the costs from doing so.

The progressive tax system is part of this policy of eliminating unfairness, but if you disagree with that, that’s okay, as more and more of the country’s wealth is staying in the hands of the super-rich. None of this wealth has trickled down to the poor and none of it ever will, as the past thirty years of economic history have repeatedly and decisively demolished the “trickle-down” concept.

None of this implies that any particular rich person is “greedy”, whatever that would mean.

5. Just Desserts and Social Mobility

5.1: Government is the recourse of “moochers”, who want to take the money of productive people and give it to the poor. But rich people earned their money, and poor people had the chance to earn money but did not. Therefore, the poor do not deserve rich people’s money.

The claim of many libertarians is that the wealthy earned their money by the sweat of their brow, and the poor are poor because they did not. The counterclaim of many liberals is that the wealthy gained their wealth by various unfair advantages, and that the poor never had a chance. These two conflicting worldviews have been the crux of many an Internet flamewar.

Luckily, this is an empirical question, and can be solved simply by collecting the relevant data. For example, we could examine whether the children of rich parents are more likely to be rich than poor parents, and, if so, how much more likely they are. This would give us a pretty good estimate of how much of rich people’s wealth comes from superior personal qualities, as opposed to starting with more advantages.

If we define “rich” as “income in the top 5%” and “poor” as “income in the bottom 5%” then children of rich parents are about twenty times more likely to become rich themselves than children of poor parents.

But maybe that’s an extreme case. Instead let’s talk about “upper class” (top 20%) and “lower class” (bottom 20%). A person born to a lower-class family only has a fifty-fifty chance of ever breaking out of the lower class (as opposed to 80% expected by chance), and only about a 3% chance of ending up in the upper class (as opposed to 20% expected by chance). The children of upper class parents are six times more likely to end up in the upper class than the lower class; the children of lower class families are four times more likely to end up in the lower class than the upper class.

The most precise way to measure this question is via a statistic called “intergenerational income mobility”, which studies have estimated at between .4 and .6. This means that around half the difference in people’s wealth, maybe more, can be explained solely by who their parents are.

Once you add in all the other factors besides how hard you work – like where you live (the average Delawarean earns $30000; the average Mississippian $15000) and the quality of your local school district, there doesn’t seem to be much room for hard work to determine more than about a third of the difference between income.

5.1.1: The conventional wisdom among libertarians is completely different. I’ve heard of a study saying that people in the lower class are more likely to end up in the upper class than stay in the lower class, even over a period as short as ten years!

First of all, note that this is insane. Since the total must add up to 100%, this would mean that starting off poor actually makes you more likely to end up rich than someone who didn’t start off poor. If this were true, we should all send our children to school in the ghetto to maximize their life chances. This should be a red flag.

And, in fact, it is false. Most of the claims of this sort come from a single discredited study. The study focused on a cohort with a median age of twenty-two, then watched them for ten years, then compared the (thirty-two-year-old) origins with twenty-two-year-olds, then claimed that the fact that young professionals make more than college students was a fact about social mobility. It was kind of weird.

Why would someone do this? Far be it from me to point fingers, but Glenn Hubbard, the guy who conducted the study, worked for a conservative think tank called the “American Enterprise Institute”. You can see a more complete criticism of the study here.

5.1.2: Okay, I acknowledge that at least half of the differences in wealth can be explained by parents. But that needn’t be rich parents leaving trust funds to their children. It could also be parents simply teaching their children better life habits. It could even be genes for intelligence and hard work.

This may explain a small part of the issue, but see 5.1.3 and 5.1.3.1, which show that under different socioeconomic conditions, this number markedly decreases. These socioeconomic changes would not be expected to affect things like genetics.

5.1.3: So maybe children of the rich do have better opportunities, but that’s life. Some people just start with advantages not available to others. There’s no point in trying to use Big Government to regulate away something that’s part of the human condition.

This lack of social mobility isn’t part of the human condition, it’s a uniquely American problem. Of eleven developed countries investigated in a recent study on income mobility, America came out tenth out of eleven. Their calculation of US intergenerational income elasticity (the number previously cited as probably between .4 and .6) was .47. But other countries in the study had income elasticity as low as .15 (Denmark), .16 (Australia), .17 (Norway), and .19 (Canada). In each of those countries, the overwhelming majority of wealth is earned by hard work rather than inherited.

The United States, is just particularly bad at this; the American Dream turns out to be the “nearly every developed country except America” Dream.

5.1.3.1: That’s depressing, but don’t try to turn it into a political narrative. Given the government’s incompetence and wastefulness, there’s no reason to think more government regulation and spending could possibly improve social mobility at all.

Studies show that increasing government spending significantly improves social mobility. States with higher government spending have about 33% more social mobility than states with lower spending.

This also helps explain why other First World countries have better social mobility than we do. Poor American children have very few chances to go to Harvard or Yale; poor Canadian children have a much better chance to go to to UToronto or McGill, where most of their tuition is government-subsidized.

5.2: Then perhaps it is true that rich children start out with a major unfair advantage. But this advantage can be overcome. Poor children may have to work harder than rich children to become rich adults, but this is still possible, and so it is still true, in the important sense, that if you are not rich it’s mostly your own fault.

Several years ago, I had an interesting discussion with an evangelical Christian on the ethics of justification by faith. I promise you this will be relevant eventually.

I argued that it is unfair for God to restrict entry to Heaven to Christians alone. After all, 99% of native-born Ecuadorans are Christian, but less than 1% of native born Saudis are same. It follows that the chance of any native-born Ecuadorian of becoming Christian is 99%, and that of any native born Saudi, 1%. So if God judges people by their religion, then within 1% He’s basically just decided it’s free entry for Ecuadorians, but people born in Saudi Arabia can go to hell (literally).

My Christian friend argued that is not so: that there is a great difference between 0% of Saudis and 1% of Saudis. I answered that no, there was a 1% difference. But he said this 1% proves that the Saudis had free will: that even though all the cards were stacked against them, a few rare Saudis could still choose Christianity.

But what does it mean to have free will, if external circumstances can make 99% of people with free will decide one way in Ecuador, and the opposite way in Saudi Arabia?

I do sort of believe in free will, or at least in “free will”. But where my friend’s free will was unidirectional, an arrow pointing from MIND to WORLD, my idea of free will is circular: MIND affects WORLD affects MIND affects WORLD and so on.

Yes, it is ultimately the mind and nothing else that decides whether to accept or reject Islam or Christianity. But it is the world that shapes the mind before it does its accepting or rejecting. A man raised in Saudi Arabia uses a mind forged by Saudi culture to make the decision, and chooses Islam. A woman raised in Ecuador uses a mind forged by Ecuador to make the decision, and chooses Christianity. And so there is no contradiction in the saying that the decision between Islam and Christianity is up entirely to the individual, yet that it is almost entirely culturally determined. For the mind is a box, filled with genes and ideas, and although it is a wonderful magical box that can take things and combine them and forge them into something quite different and unexpected, it is not infinitely magical, and it cannot create out of thin air.

Returning to the question at hand, every poor person has the opportunity to work hard and eventually become rich. Whether that poor person grasps the opportunity comes from that person’s own personality. And that person’s own personality derives eventually from factors outside that person’s control. A clear look at the matter proves it must be so, or else personality would be self-created, like the story of the young man who received a gift of a time machine from a mysterious aged stranger, spent his life exploring past and future, and, in his own age, goes back and gives his time machine to his younger self.

5.2.1: And why is this relevant to politics?

Earlier, I offered a number between .4 and .6 as the proportion of success attributable solely to one’s parents’ social class. This bears on, but does not wholly answer, a related question: what percentage of my success is my own, and what percentage is attributable to society? People have given answers to this question as diverse as (100%, 0%), (50%, 50%), (0%, 100%).

I boldly propose a different sort of answer: (80%, 100%). Most of my success comes from my own hard work, and all of my own hard work comes from external factors.

If all of our success comes from external factors, then it is reasonable to ask that we “pay it forward” by trying to improve the external factors of others, turning them into better people who will be better able to seize the opportunities to succeed. This is a good deal of the justification for the liberal program of redistribution of wealth and government aid to the poor.

5.2.2: This is all very philosophical. Can you give some concrete examples?

Lead poisoning, for example. It’s relatively common among children in poorer areas (about 7% US prevalence) and was even more common before lead paint and leaded gasoline was banned (still >30% in many developing countries).

For every extra ten millionths of a gram per deciliter concentration of lead in their blood, children permanently lose five IQ points; there’s a difference of about ten IQ points among children who grew up in areas with no lead at all, and those who grew up in areas with the highest level of lead currently considered “safe”. Although no studies have been done on severely lead poisoned children from the era of leaded gasoline, they may have lost twenty or more IQ points from chronic lead exposure.

Further, lead also decreases behavioral inhibition, attention, and self-control. For every ten ug/dl lead increase, children were 50% more likely to have recognized behavioral problems. People exposed to higher levels of blood lead as a child were almost 50% more likely to be arrested for criminal behavior as adults (adjusting for confounders).

Economic success requires self-control, intelligence, and attention. It is cruel to blame people for not seizing opportunities to rise above their background when that background has damaged the very organ responsible for seizing opportunities. And this is why government action, despite a chorus of complaints from libertarians, banned lead from most products, a decision which is (controversially) credited with the most significant global drop in crime rates in decades, but which has certainly contributed to social mobility and opportunity for children who would otherwise be too lead-poisoned to succeed.

Lead is an interesting case because it has obvious neurological effects preventing success. The ability of psychologically and socially toxic environments to prevent success is harder to measure but no less real.

If a poor person can’t keep a job solely because she was lead-poisoned from birth until age 16, is it still fair to blame her for her failure? And is it still so unthinkable to take a little bit of money from everyone who was lucky enough to grow up in an area without lead poisoning, and use it to help her and detoxify her neighborhood?

5.3: What is the significance of whether success is personally or environmentally determined?

It provides justification for redistribution of wealth, and for engineering an environment in which more people are able to succeed.

6. Taxation

6.1: Isn’t taxation, the act of taking other people’s money by force, inherently evil?

See the Moral Issues section for a more complete discussion of this point.

6.2: Isn’t progressive taxation, the tendency to tax the rich at higher rates than the poor, unfair?

The most important justification for progressive tax rates is the idea of marginal utility.

This is easier to explain with movie tickets than money. Suppose different people are allotted a different number of non-transferable movie tickets for a year; some people get only one, other people get ten thousand.

A person with only two movie ticket might love to have one extra ticket. Perhaps she is a huge fan of X-Men, Batman, and Superman, and with only two movie ticket she will only be able to see two of the three movies she’s super-excited about this year.

A person with ten movie tickets would get less value from an extra ticket. She can already see the ten movies that year she’s most interested in. If she got an eleventh, she’d use it for a movie she might find a bit enjoyable, but it wouldn’t be one of her favorites.

A person with a hundred movie tickets would get minimal value from an extra ticket. Even if your tickets are free, you’re not likely to go to the movies a hundred times a year. And even if you did, you’d start scraping the bottom of the barrel in terms of watchable films.

A person with a thousand tickets would get practically no value from an extra ticket. At this point,t here’s no way she can go to any more movies. The extra ticket might not have literally zero value – she could burn it for warmth, or write memos on the back of it – but it’s pretty worthless.

So although all movie tickets provide an equal service – seeing one movie – one extra movie ticket represents a different amount of value to the person with two tickets and the person with a thousand tickets. Furthermore, 50% of their movie ticket holdings represent a different value to the person with two tickets and the person with a thousand movie tickets. The person with two tickets loses the ability to watch the second-best film of the year. The person with a thousand tickets still has five hundred tickets left, more than enough to see all the year’s best films, and at worst will have to buy some real memo paper.

Money works similarly to movie tickets. Your first hundred dollars determine whether you live or starve to death. Your next five hundred dollars determine whether you have a roof over your head or you’re freezing out on the street. But by your ten billionth dollar, all you’re doing is buying a slightly larger yacht.

50% of what a person with $10,000 makes is more valuable to her than 50% of what a billionaire makes is to the billionaire.

Progressive taxation is an attempt to tax everyone equally, not by lump sum or by percentage, but by burden. Just as taking extra movie tickets away from the person with a thousand is more fair than taking some away from the person with only two, so we tax the rich at a higher rate because a proportionate amount of money has less marginal value to them.

6.2.1: But the progressive tax system is unfair and perverse. Imagine the tax rate on people making $100,000 or less is 30%, and the tax rate on people making more than $100,000 is 50%. You make $100,000, and end up with after tax income of $70,000. Then one day your boss tells you that you did a good job, and gives you a $1 bonus. Now you make $100,001, but end up with only $50,000.50 after tax income. How is that at all fair?

It’s not, but this isn’t how the tax system works.

What those figures mean is that your first $100,000, no matter how much you earn, is taxed at 30%. Then the money you make after that is taxed at 50%. So if you made $100,001, you would be taxed 30% on the first $100,000 (giving you $70,000), and 50% on the next $1 (giving you $.50), for an after-tax income of $70,000.50. The intuitive progression where someone who makes more money ends up with more after-tax income is preserved.

I know most libertarians don’t make this mistake, and that there are much stronger arguments against progressive taxation, but this has come up enough times that I thought it was worth mentioning, with apologies to those readers whose time it has wasted.

6.3: Taxes are too high.

Too high by what standard?

6.3.1: Too high by historical standards. Thanks to the unstoppable growth of big government, people have to pay more taxes now than ever before.

Actually, income tax rates for people on median income are around the lowest they’ve been in the past seventy-five years

6.3.1.1: I meant for the rich. It’s only tolerable for people on median income because “progressive” governments are squeezing every last dollar out of successful people.

Actually, income tax rates for the rich are around the lowest they’ve been in the past seventy-five years.

6.3.1.1.1: But I heard that the share of tax revenue coming from the rich is at its highest level ever.

This is true. As the rich get richer and the poor get poorer (see 3.4), more of the money concentrates in the hands of the rich, and so more of the taxes come from the rich as well. This doesn’t contradict the point that the tax rates on the rich are near historic lows.

6.3.1.2: I meant for corporations.

Actually, income tax rates for corporations are around the lowest they’ve been in the past seventy-five years.

6.3.2: I meant income taxes are too high compared to what’s best for the economy, and even best for the Treasury. With taxes as high as they are, people will stop producing, rather than see so much of each dollar they make go to the government. This will hurt the economy and lower tax revenue.

The Laffer curve certainly exists, but the consensus is that we’re still well on the left half of it.

Although it’s become a truism that high tax rates discourage production, studies have found this to be mostly false, with low elasticity of real income – see for example Gruber & Saez and Saez, Slemrod, and Giertz.

What studies have found is a high elasticity of taxable income. That is, raising taxes encourages people to find more tax loopholes, decreasing revenue. However, although this effect means a 10% higher tax rate would lead to less than 10% higher government income, the change in government income would still be positive – even by this stricter criterion, we’re still on the left side of the Laffer curve. And of course, this effect could be eliminated by switching to a flat tax or closing tax loopholes.

6.4: Our current tax system is overzealous in its attempts to redistribute money from the rich to the poor. If instead we lowered taxes on the rich, this money would “trickle down” to the rest of the economy, driving growth. Instead of redistributing the pie, we’d make the pie larger for everyone.

If we’re in an overzealous campaign for “equality” intended to lower the rich to the level of the poor, we’re certainly not doing a very good job of it. Over the past thirty years, the rich have consistently gotten richer. None of this money has trickled down to the poor or middle-class, whose income has remained the same in real terms.

“Trickle-down” should be rejected as an interesting and plausible-sounding economic theory which empirical data have soundly disconfirmed.

6.5: Raising taxes would be useless for the important things like cutting the deficit. The deficit is $1.2 trillion. The most we could realistically raise from extra taxes on the rich would be maybe $200 billion. The most we could raise from insane levels of extra taxes on the rich and middle class would be about $500 billion – less than half the deficit. The real problem is spending.

Yes and no.

The deficit is, indeed, very, very large. It’s so large that no politically palatable option is likely to make more than a small dent in it. This is true of tax increases. It’s also true of spending cuts.

Cutting all redistributive government services for the poor including welfare, unemployment insurance, disability, food stamps, scholarships, you name it – would save about $200 billion. That’s less than 20% of the deficit. Cutting all health care, including Medicaid for senior citizens, would only eliminate $400 billion or so. Even eliminating the entire military down to the last Jeep would only get us $800 billion or so. The targets for cuts that have actually been raised are rounding errors: the Republicans trumpeted an end for government aid to NPR, but this is about $4 million – all of .000003% of the problem.

So “darnit, this one thing doesn’t completely solve the deficit” is not a good reason to reject a proposal. Solving the deficit will, if it’s possible at all, take a lot of different methods, including some unpalatable to liberals, some unpalatable to conservatives, and yes, some unpalatable to libertarians.

In particular, we need to avoid the “bee sting” fallacy, where we have so many problems that we just stop worrying. It would be irresponsible to say that since a few billion dollars doesn’t affect the deficit either way, we might as well just spend $5 billion on some random project we don’t need. For the same reason, it would be irresponsible to say we might as well just renew tax cuts on the rich that cost hundreds of billions of dollars each year.

6.6: Taxes are basically a racket where they take my money and then give it to foreign governments and poor people.

According to a CNN poll, on average Americans estimate that about 10% of our taxes go to foreign aid. The real number is about 0.6%.

And although people believe that food and housing for the poor take up about 20% of the federal budget, the real number is actually less than 5%.

So although people worry that 30% of the budget goes to help the less fortunate, the real number is about 6%.

(And this is actually sort of depressing, when you think about it.)

The majority of your taxes go to programs that benefit you and other middle-class Americans, such as Social Security and Medicare, and to programs that “benefit” you and other middle-class Americans, such as the military.

Part C: Political Issues

The Argument: Government can’t do anything right. Its forays into every field are tinged in failure. Whether it’s trying to create contradictory “state owned businesses”, funding pet projects that end up over budget and useless, or creating burdensome and ridiculous “consumer protection” rules, its heavy-handed actions are always detrimental and usually embarrassing.

With this track record, what sane person would want to involve government in even more industries? The push to get government deeper into health care is a disaster waiting to happen, and could give us a chronically broken system like those in Europe, where people die because of bureaucratic inefficiency.

Other places from which we can profitably eliminate government’s prying hands include our schools, our prisons, our gun dealerships, and the friendly neighborhood meth lab.

The Counterargument: Government sometimes, though by no means always, does things right, and some of its institutions and programs are justifiably considered models of efficiency and human ingenuity. There are various reasons why people are less likely to notice these.

Government-run health systems empirically produce better health outcomes for less money than privately-run health systems for reasons that include economies of scale. There are a mountain of statistics that prove this. Although not every proposal to introduce government into health will necessarily be successful, we would do well to consider emulating more successful systems.

We should think twice about exactly how much government we are willing to remove from our schools, gun dealerships, and meth labs, and run away screaming at the proposal to privatize prisons.

7. Competence of Government

7.1: Government never does anything right.

7.1.1: Okay, fine. But that’s a special case where, given an infinite budget, they were able to accomplish something that private industry had no incentive to try. And to their credit, they did pull it off, but do you have any examples of government succeeding at anything more practical?

Eradicating smallpox and polio globally, and cholera and malaria from their endemic areas in the US. Inventing the computer, mouse, digital camera, and email. Building the information superhighway and the regular superhighway. Delivering clean, practically-free water and cheap on-the-grid electricity across an entire continent. Forcing integration and leading the struggle for civil rights. Setting up the Global Positioning System. Ensuring accurate disaster forecasts for hurricanes, volcanoes, and tidal waves. Zero life-savings-destroying bank runs in eighty years. Inventing nuclear power and the game theory necessary to avoid destroying the world with it.

7.1.1.1: All right… all right… but apart from better sanitation and medicine and education and irrigation and public health and roads and a freshwater system and baths and public order… what has the government done for us?

Brought peace. But see also Government Success Stories and The Forgotten Achievements of Government

7.2: Large government projects are always late and over-budget.

The only study on the subject I could find, “What Causes Cost Overrun in Transport Infrastructure Projects?” (download study as .pdf) by Flyvbjerg, Holm, and Buhl, finds no difference in cost overruns between comparable government and private projects, and in fact find one of their two classes of government project (those not associated with a state-owned enterprise) to have a trend toward being more efficient than comparable private projects. They conclude that “…one conclusion is clear…the conventional wisdom, which holds that public ownership is problematic whereas private ownership is a main source of efficiency in curbing cost escalation, is dubious.”

Further, when government cost overruns occur, they are not usually because of corrupt bureaucrats wasting the public’s money. Rather, they’re because politicians don’t believe voters will approve their projects unless they spin them as being much cheaper and faster than the likely reality, leading a predictable and sometimes commendable execution to be condemned as “late and over budget” (download study as .pdf) While it is admittedly a problem that government provides an environment in which politicians have to lie to voters to get a project built, the facts provide little justification for a narrative in which government is incompetent at construction projects.

7.3: State-run companies are always uncreative, unprofitable, and unpleasant to use.

Some of the greatest and most successful companies in the world are or have been state-run. Japan National Railways, which created the legendarily efficient bullet trains, and the BBC, which provides the most respected news coverage in the world as well as a host of popular shows like Doctor Who, both began as state-run corporations (JNR was later privatized).

In cases where state-run corporations are unprofitable, this is often not due to some negative effect of being state-run, but because the corporation was put under state control precisely because it was something so unprofitable no private company would touch it, but still important enough that it had to be done. For example, the US Post Office has a legal mandate to ship affordable mail in a timely fashion to every single god-forsaken town in the United States; obviously it will be out-competed by a private company that can focus on the easiest and most profitable routes, but this does not speak against it. Amtrak exists despite passenger rail travel in the United States being fundamentally unprofitable, but within its limitations it has done a relatively good job: on-time rates better than that of commercial airlines, 80% customer satisfaction rate, and double-digit year-on-year passenger growth every year for the past decade.

7.3.1: State-run companies may be able to paper-push with the best of them, but the government can never be truly innovative. Only the free market can do that. Look at Silicon Valley!

Advances invented either solely or partly by government institutions include, as mentioned before, the computer, mouse, Internet, digital camera, and email. Not to mention radar, the jet engine, satellites, fiber optics, artificial limbs, and nuclear energy. And that doesn’t the less recognizable inventions used mostly in industry, or the scores of other inventions from government-funded universities and hospitals.

Even those inventions that come from corporations often come not from startups exposed to the free market, but from de facto state-owned monopolies. For example, during its fifty years as a state-sanctioned monopoly, the infamous Ma Bell invented (via its Bell Labs division) transistors, modern cryptography, solar cells, the laser, the C programming language, and mobile phones; when the monopoly was broken up, Bell Labs was sold off to Alcatel-Lucent, which after a few years announced it was cutting all funding for basic research to focus on more immediately profitable applications.

Although the media celebrates private companies like Apple as centers of innovation, Apple’s expertise lies, at best, in consumer packaging. They did not invent the computer, the mp3 player, or the mobile phone, but they developed versions of these products that were attractive and easy to use. This is great and they deserve the acclaim and heaps of money they’ve gathered from their success, but let’s make sure to call a spade a spade: they are good at marketing and design, not at brilliant invention of totally new technologies.

That sort of de novo invention seems to come mostly from very large organizations that can afford basic research without an obsession on short-term profitability. Although sometimes large companies like Ma Bell, invention-rich IBM and Xerox can fulfill this role, such organizations are disproportionately governments and state-sponsored companies, explaining their impressive track record in this area.

7.4: Most government programs are expensive failures.

I think this may be a form of media bias – not in the sense that some sinister figure in the media is going through and censoring all the stories that support one side, but in the sense that “Government Program Goes More Or Less As Planned” doesn’t make headlines and so you never hear about it.

Let’s say the government wants to spent $1 million to give food to poor children. If there are bureaucratic squabbles over where the money’s supposed to come from, that’s a headline. If they buy the food at above-market prices, that’s a headline. If some corrupt official manages to give the contract to provide the food to a campaign donor along the way, that’s a big headline.

But what if none of these things happen, and poor children get a million dollars worth of food, and eat it, and it makes them healthier? I don’t know about you, but I’ve never seen a headline about this. “Remember that time last year when Congress voted to give food to poor children. Well, they got it.” What newspaper would ever publish something like that?

This is in addition to newspapers’ desire to outrage people, their desire to sound “edgy” by pointing out the failures of the status quo rather than sounding like they’re “pandering”, and honestly that they’re caught up in the same “government can never do anything right” narrative as everyone else.

Since every single time you ever hear about a government project it is always because that government project is going wrong, of course you feel like all government projects go wrong.

7.4.1: But a specific initiative to get money to the poor is one thing. What about a whole federal agency? We would know if it were failing, but we’d also be able to appreciate it when it succeeds, too.

Federal agencies that are successful sink into background noise, so that we don’t think to thank them or celebrate them any more than we would celebrate that we have clean water (four billion people worldwide don’t; thank the EPA and your local water board)

For example, the Federal Aviation Administration helps keep plane crashes at less than one per 21,000 years of flight time; you never think about this when you get on a plane. The National Crime Information Center collects and processes information about criminals from every police department in the country; you never think about this when you go out without being mugged. Zoning regulations, building codes, and the fire department all help prevent fires from starting and keep them limited when they do; you never think of this when you go the day without your house burning down.

One of government’s major jobs is preventing things, and it’s very hard to notice how many bad things aren’t happening, until someone comes out with a report like e. coli poisoning has dropped by half in the past fifteen years. Even if you do hear the statistics, you may never think to connect them to the stricter food safety laws you wrote a letter to the editor opposing fifteen years ago.

7.4.2: You list cases where government regulation exists at the same time as a happy outcome, like the FAA and the lack of plane crashes, but that doesn’t prove it was the regulation that caused the happy outcome.

No, it doesn’t. For example, although workplace accidents have been cut in half since OSHA was founded, CATO wrote a very credible takedown in which they argue that was only a continuation of trends that have been going on since before OSHA existed.

Sometimes there are things we can do to identify cause. For example, as in the CATO study, we can compare trends before and after changes in government regulation; if there is a discontinuity, it may suggest the government was responsible. Second, we can compare trends in a country where a new regulation was introduced to trends in a country where it was not introduced; if the trend only changes in one country, that suggests an effect of the regulation. For example, after the FAA mandated “terrain awareness systems” in airplanes, the terrain-related accident rate sharply dropped to zero in the United States but was not affected in countries without similar rules.

But the important thing is that we apply our skepticism fairly and evenly: that we do not require mountains of evidence that a government regulation caused a positive result, while accepting that a regulation caused a negative result without a shred of proof.

It is very tempting for libertarians, when faced with anything going well even in a tightly regulated area, to say “Well, that just shows even this tight regulations can’t hide how great private industry is!” and when anything goes wrong even in a very loosely regulated area, to say “Well, that just shows how awful regulation is, that even a little of it can screw things up!” But this is unfair, and ignores that we do have some ways to disentangle cause and effect.

And in any case, there is still the difference between “Government destroys everything it touches” and “Everything government touches is doing pretty well, but you can’t prove that it’s directly caused by government action.”

7.4.3: A lot of what government trumpets as “successful regulation” is just obvious stuff anyway that any individual in a free market would do of her own accord.

Very often, yesterday’s regulation is today’s obvious good idea that no one would dream of ignoring even if there were no regulation demanding it. But that neglects the role of government regulation in establishing social norms. Very often these are the regulations which those being regulated fought tooth and nail against at the time.

Many cars did not even include seatbelts until the government mandated that they do so. In 1983, the seat belt use rate in the United States was 14%. It was very clearly the government sponsored awareness campaigns and, later, mandatory seat belt laws that began being implemented around that era that raised seat belt rates; we know because we can watch the statistics state in different states as their legislation either led the campaign or lagged behind it.

After almost three decades of intense government pressure on automakers to allow and promote seatbelts, and on motorists to use them, seatbelt rates are now as high as 85%.

According to estimates, seatbelts save about 11,000 lives a year in the US. Different studies estimate between 80,000 and 100,000 lives saved in the last decade alone. For some perspective that’s the number of American deaths from 9/11 + the Vietnam War + both Iraq Wars + the Afghanistan War + Hurricane Katrina.

I completely acknowledge that if the government completely dropped all seatbelt regulations tomorrow, automakers would continue putting seatbelts in cars, and drivers would keep wearing them. That doesn’t mean government is useless, that means government, the only entity big enough to effect a nationwide change not just in behaviors but in social norms, did its job very very well.

8. Health Care

8.1: Government would do a terrible job in health care. We should avoid government-run “socialized” medicine unless we want cost overruns, long waiting times, and death panels.

Government-run health systems empirically do better than private health systems, while also costing much less money.

Let’s compare, for example, Sweden, France, Canada, the United Kingdom, and the United States. The first four all have single-payer health care (a version of government-run health system); the last has a mostly private health system (although it shouldn’t matter, we’ll use statistics from before Obamacare took effect). We’ll look at three representative statistics commonly used to measure quality of health care: infant mortality, life expectancy, and cancer death rate.

Infant mortality is the percent of babies who die in the first few weeks of life, usually a good measure of pediatric and neonatal care. Of the five countries, Sweden has the lowest infant mortality at 2.56 per 1,000 births, followed by France at 3.54, followed by the UK at 4.91, followed by Canada at 5.22, with the United States last at 6.81. (source)

Life expectancy, the average age a person born today can expect to live, is a good measurement of lifelong and geriatric care. Here Sweden is again first at 80.9, France and Canada tied for second at 80.7, the UK next at 79.4, and the United States once again last at 78.3. (source)

Taking cancer deaths per 100,000 people per year as representative of deaths from serious disease, here we find the UK doing best at 253.5 deaths, Sweden second at 268.2, France in third at 286.1, and the United States again in last place at 321.9 deaths (source: OECD statistics; data for Canada not available).

So we notice that the United States does worse than all four countries with single-payer health systems, even though America is wealthier per capita than any of them. This is not statistical cherry-picking: any way you look at it, the United States has one of the least effective health systems in the developed world.

8.2: Government-run health care would be bloated, bureaucratic, and unnecessarily expensive, as opposed to the sleek, efficient service we get from the free market.

Actually, government-run health care is empirically more efficient than market health care. For example, Blue Cross New England employs more people to administer health insurance for its 2.5 million customers than the Canadian health system employs to administer health insurance for 27 million Canadians. Health care spending per person (public + private) in Canada is half what it is in America, yet Canadians have longer life expectancy, lower infant mortality, and are healthier by every objective standard.

Remember those five countries from the last question?
The UK spends $1,675 per person per year on health care. Canada spends $1,939. Sweden, which you’ll remember did best on most of the statistics, spends $2,125. France spends $2,288. Americans spend on average $4,271 – almost three times as much as Britain, a country which delivers better health care.

When this argument gets put in graph form, it becomes even clearer that US health inefficiency is literally off the chart.

If these were companies in the free market, the company that charges three times as much to provide a worse service would have gone bankrupt long ago. That company is American-style private health care.

8.3: In government-run health care, people are relegated to “waiting lists”, where they have to wait months or even years for doctor visits, surgeries, and other procedures. Sometimes people die on these waiting lists. Obviously, this is unacceptable and a knock-down argument against government-run health care.

The laws of supply and demand apply in health care as much as anywhere else: people would like to see doctors as quickly as possible, but doctors are a scarce resource that must be allocated somehow.

In a private system, doctor access is allocated based on money; this has the advantage of incentivizing the production of more doctors and of ensuring that people with enough money can see doctors quickly. These are also its disadvantages: assuming more people want to see a doctor than need to do so, costs will spiral out of control and poor people will have limited or no access.

In a public system, doctor access is allocated based on medical need. Although no one will be turned away from a doctor in an emergency situation, people may have to wait a long amount of time for elective surgeries in order that other sicker people, including poor people who would not be seen at all in a private system, can be seen first.

The relative effectiveness of the two systems can once again be seen in the infant mortality, life expectancy, and cancer survival rate statistics.

8.4: Government-run health care inevitably includes “death panels” who kill off expensive patients in order to save money on health care costs.

The private system as it exists now in America also has bodies that make these kinds of rationing decisions. Health care rationing is not some sinister conspiracy but a reasonable response to limited resources. The complete argument is here, but I can sum up the basics:

Insurance providers, whether they are a government agency or a private corporation, have a finite amount of money; they can only spend money they have. In one insurance company, customers might pay hundred million dollars in fees each year, so the total amount of money the insurance company can spend on all its customers that year is a hundred million dollars. In reality, since it is a business, it wants to make a profit. Let’s say it wants a profit of ten percent. That means the total amount of money it has to spend is ninety million dollars.

But as a simplified example, let’s reduce this to an insurance company with one hundred customers, each of whom pays $1. This insurance company wants 10% profit, so it has $90 to spend (instead of our real company’s $90 million). Seven people on the company’s plan are sick, with seven different diseases, each of which is fatal. Each disease has a cure. The cures cost, in order, $90, $50, $40, $20, $15, $10, and $5.

We are far too nice to ration health care with death panels; therefore, we have decided to give everyone every possible treatment. So when the first person, the one with the $90 disease, comes to us, we gladly spend $90 on their treatment; it would be inhuman to just turn them away. Now we have no money left for anyone else. Six out of seven people die.

The fault here isn’t with the insurance company wanting to make a profit. Even if the insurance company gave up its ten percent profit, it would only have $10 more; enough to save the person with the $10 disease, but five out of seven would still die.

A better tactic would be to turn down the person with the $90 disease. Instead, treat the people with $5, $10, $15, $20, and $40 diseases. You still use only $90, but only two out of seven die. By refusing treatment to the $90 case, you save four lives. This solution can be described as more cost-effective; by spending the same amount of money, you save more people. Even though “cost-effectiveness” is derided in the media as being opposed to the goal of saving lives, it’s actually all about saving lives.

If you don’t know how many people will get sick next year with what diseases, but you assume it will be pretty close to the amount of people who get sick this year, you might make a rule for next year: Treat everyone with diseases that cost $40 or less, but refuse treatment to anyone with diseases that cost $50 or more.

This rule remains true in the case of the $90 million insurance company. In their case, no one patient can use up all the money, but they still run the risk of spending money in a way that is not cost-effective, causing many people to die. Like the small insurance company, they can increase cost-effectiveness by creating a rule that they won’t treat people with diseases that cost more than a certain amount.

So, as one commentator pointed out, “death panels” should be called “life panels”: they aim to maximize the total number of lives that can be saved with a certain limited amount of resources.

8.5: Why is government-run health care so much more effective?.

A lot of it is economies of scale: if the government is ensuring the entire population of a country, it can get much better deals than a couple of small insurance companies. But a lot of it is more complicated, and involves people’s status as irrational consumers of health products. A person sick with cancer doesn’t want to hear a cost-benefit analysis suggesting that the latest cancer treatment is probably not effective. He wants that treatment right now, and the most successful insurance companies and hospitals are the ones that will give it to him. Here’s a good article explaining some of the systematic flaws in the economics of health care under the American system.

It could also be that really good health care and the profit motive don’t mix: studies show that for-profit hospitals are more expensive, and have poorer care (as measured in death rates) than not-for-profit hospitals.

9. Prison Privatization

9.1: Privatized, for-profit prisons would be a great way to save money.

No one likes criminals very much. Even so, most of us agree that even criminals deserve humane conditions. We reject cruel and unusual punishment, and try to keep prisoners relatively warm, clean, and well-fed. This is not only a moral issue, but a practical one: we don’t want prisoners to go insane or suffer breakdowns, because we want them to be able to re-adjust into normal society after they are released.

For-profit prisons have all of the flaws of for-profit companies with none of the advantages. Normal companies want to cut costs wherever possible, but this is balanced by customer satisfaction: if they treat their customers poorly or create a low-quality product, they won’t make money. In prisons, the ability to get new “customers” comes completely uncoupled from the quality of the product they provide. If the government pays them a certain fixed amount per prisoner, the prison’s only way to increase profits is by treating prisoners as shabbily as possible without killing them. Indeed, statistics show that prisoners in private prisons have worse medical care, terrible living conditions, and rates of in-prison violence 150% greater than those in public prisons. Private prisons refuse to collect data on recidivism rates, but a moment’s thought reveals that they have an economic incentive to keep them as high as possible.

But the real dangers lie in the corruptibility of the political process, something with which libertarians are already familiar. Private prisons have been active in lobbying for stricter sentencing guidelines like the Three Strikes Law, which encourages governments to imprison criminals for life. In a country that already imprisons more of its population than any other country in the world, it is extremely dangerous to create a powerful political force whose self-interest lies in imprisoning as many people as possible.

But the most striking example of the danger of private prisons is the case of two judges who received bribes from private prisons to jail innocent people.

If this is the alternative, I’m willing to bite the bullet and accept the overpaid prison guards with annoying unions who dominate the public prisons.

9.2: What? Libertarians don’t actually believe in private prisons!

Fair enough; I got this complaint a few times on the first version and I acknowledge it’s not an integral component of libertarian philosophy. I included it because it seems to stem from the same “government can never do anything right and we should privatize everything” idea that drives a lot of libertarian thinking, and because I really, really don’t like private prisons.

10. Gun Control

10.1: Gun control laws only help criminals, who are not known for following laws in any case, make sure that their victims are unarmed and unable to resist; as such, they increase crime.

The statistics supporting this view seem relatively solid and I agree that attempts to ban or restrict access to guns are a bad idea.

On the other hand, many of the issues surrounding gun control are much less restrictive. For example, some involve restrictions on sales to criminals, “cooldown periods” before purchase, mandatory safety training, et cetera.

Although I haven’t seen any evidence either way on whether these laws are beneficial, they should be evaluated on their own merits rather than as part of a narrative in which all gun laws must be opposed because gun control is bad.

11. Education

11.1: Government sponsored public education is a horrible failure.

Compared to what?

Compared to the period when there wasn’t government-sponsored public education…well, that’s hard to say because of poor statistic-keeping at that time, and how one counts minorities and women, who usually weren’t educated at all back then. The most official statistics (eg NOT the ones you find without citation on libertarian blogs that say literacy was 100% way back when and became abysmal as soon as public schooling started) say that white illiteracy declined from about 11.5% in the mid-1800s to about 0.5% in 1980, and black illiteracy from about 80% to 1.5% over the same period.

Compared to other countries, the US does relatively poorly considering its wealth, but all the other countries that do better than the US also have government-sponsored public education, sometimes to a much greater degree than we do.

Compared to private schools, public schools actually do better once confounders like race, class, and income have been adjusted out of the analysis.

(Yes, without such adjustment private schools do better – but considering that private schools cater towards wealthy students – who usually do better in school – and often have selective admission policies in which they only take students who are already pretty smart – whereas public schools have to take everyone including dumb kids, kids with learning disabilities, and kids from broken families in ghettos – such unadjusted data is meaningless. It’s the equivalent of noting that the doctor who specializes in acne has fewer patients die than the doctor who specializes in cancer: it’s not that she’s a better doctor, just that she only takes cases who are pretty healthy already.)

Our educational system certainly has immense room for improvement. But the country that consistently tops world education rankings, Finland, has zero private schools (even all the universities are public) and no “school choice”. What it does have is extremely well-credentialed, highly paid teachers (and, unfortunately, an ethnically homogenous population without any dire poverty or broken families, which probably counts for a heck of a lot more than anything else). So whatever America’s specific failures or successes, the mere existence of public education is not a credible scapegoat.

11.2: Why not dismantle the public education system and have a voucher system that offers parents free choice over where to send their kids?

I think this idea has merit, and that we should at least experiment with it and see if it works. That having been said, I do see one huge caveat.

Libertarians tend not to believe in equality of results – they think it’s okay if more skilled people are more successful – but one of the qualities I most admire about them is that they usually do believe in equality of opportunity: that everyone gets an equal chance at life. I mentioned before how inheriting money from your parents can complicate that, but it would be ethically complicated to try and “solve” that problem, so it might be the sort of thing we just have to live with.

But imagine if your parents chose where to send you for school. Even if we somehow eliminated the cost issue by making everyone accept a school voucher of equal value, clever parents would compare the pros and cons of various schools and send their child to the best one. Not-so-clever parents would get fooled by TV commercials with sexy celebrities and send their kids to terrible schools. Super religious parents would send their kids to schools that taught only religious education and shunned math and science and history as the evil trappings of the secular world. Muslim parents would send their kids to madrassas. Immigrant parents might send their kids to Spanish-only schools so that they didn’t drift too far away from their families. Parents with strong political beliefs could send their kids to schools that did their best to brainwash their kids into having the same beliefs as them.

And there would be kids who succeeded in spite of all this, who made it through twelve years of constant brainwashing and ignorance, and somehow managed to become intelligent adults who could learn all the education they missed during their free time. But statistically, there wouldn’t be very many of them, any more than there were a bunch of Christians in Saudi Arabia in the example a few pages back.

Right now, parents can screw up lots of facets of a kid’s life, but they can only do so much to screw up their education. And I have this vague hope that maybe a kid with horrible parents, if she was exposed to decent people and a free exchange of ideas in school might be able to use that brief period of respite to gain a foothold on sanity.

So what I’m saying is, if there were school choice, if we wanted to protect equality of opportunity and children’s’ rights, we’d probably have to regulate the heck out of them, which to some degree would defeat the point.

11.3: I don’t believe the government should be in the business of “protecting” children from their parents.

You should. It’s a pretty important business, even if you subscribe to libertarian assumptions. Even libertarians tend to agree that the government should generally be protecting people from slavery and from the use of force.

Children are basically slaves to their parents for the first ten to fifteen years of their lives, and parents have a special social permission to use force against their children.

In the best possible case, this is an incredibly silly metaphor and one no one would ever even think about. In the worst possible case, it’s completely and literally true.

I have met people with horrible parents. The first eighteen years (or less, if they were able to get themselves legally emancipated early) of their lives were a living hell. These are people who literally have control of every single thing you do, from whether you can eat dinner to who you are allowed to make friends with to what church you go to to what opinions you can express to whether you’re allowed to sleep at night. They are people who can torture and beat you to within an inch of your life, and maybe a social worker will take you away for a few months, and then that social worker will probably return you right back to them. And if it’s just emotional torture, you can forget about even getting the social worker.

And obviously the parent-child relationship is a healthy one in 99% of cases, and child-rearing has been around since deep prehistoric time, and we would be idiots to mess with it, and no one wants a dystopia where the government takes kids from their parents and raises them in a commune or whatever.

But unless you think rights and morality only start existing on someone’s eighteenth birthday, if if there were one form of government intervention that even libertarians should be able to get behind, it would be protecting children from their parents, in the rare few cases where this is necessary.

Part D: Moral Issues

The Argument: Moral actions are those which do not initiate force and which respect people’s natural rights. Government is entirely on force, making it fundamentally immoral. Taxation is essentially theft, and dictating the conditions under which people may work (or not work) via regulation is essentially slavery. Many government programs violate people’s rights, especially their right to property, and so should be opposed as fundamentally immoral regardless of whether or not they “work”.

The Counterargument: Moral systems based only on avoiding force and respecting rights are incomplete, inelegant, counterintuitive, and usually riddled with logical fallacies. A more sophisticated moral system, consequentialism, generates the principles of natural rights and non-initiation of violence as heuristics that can be used to solve coordination problems, but also details under what situations such heuristics no longer apply. Many cases of government intervention are such situations, and so may be moral.

12. Moral Systems

12.1: Freedom is incredibly important to human happiness, a precondition for human virtue, and a value almost everyone holds dear. People who have it die to protect it, and people who don’t have it cross oceans or lead revolutions in order to gain it. But government policies all infringe upon freedom. How can you possibly support this?

Freedom is one good among many, albeit an especially important one.

In addition to freedom, we value things like happiness, health, prosperity, friends, family, love, knowledge, art, and justice. Sometimes we have to trade off one of these goods against another. For example, a witness who has seen her brother commit a crime may have to decide between family and justice when deciding whether to testify. A student who likes both music and biology may have to decide between art and knowledge when choosing a career. A food-lover who becomes overweight may have to decide between happiness and health when deciding whether to start a diet.

People sometimes act as if there is some hierarchy to these goods, such that Good A always trumps Good B. But in practice people don’t act this way. For example, someone might say “Friendship is worth more than any amount of money to me.” But she might continue working a job to gain money, instead of quitting in order to spend more time with her friends. And if you offered her $10 million to miss a friend’s birthday party, it’s a rare person indeed who would say no.

In reality, people value these goods the same way they value every good in a market economy: in comparison with other goods. If you get the option to spend more time with your friends at the cost of some amount of money, you’ll either take it or leave it. We can then work backward from your choice to determine how much you really value friendship relative to money. Just as we can learn how much you value steel by learning how many tons of steel we can trade for how many barrels of oil, how many heads of cabbages, or (most commonly) how many dollars, so we can learn how much you value friendship by seeing when you prefer it to opportunities to make money, or see great works of art, or stay healthy, or become famous.

Freedom is a good much like these other goods. Because it is so important to human happiness and virtue, we can expect people to value it very highly.

But they do not value it infinitely highly. Anyone who valued freedom from government regulation infinitely highly would move to whichever state has the most lax regulations (Montana? New Hampshire?), or go live on a platform in the middle of the ocean where there is no government, or donate literally all their money to libertarian charities or candidates on the tiny chance that it would effect a change.

Most people do not do so, and we understand why. People do not move to Montana because they value aspects of their life in non-Montana places – like their friends and families and nice high paying jobs and not getting eaten by bears – more than they value the small amount of extra freedom they could gain in Montana. Most people do not live on a platform in the middle of the ocean because they value aspects of living on land – like being around other people and being safe – more than they value the rather large amount of extra freedom the platform would give them. And most people do not donate literally all their money to libertarian charities because they like having money for other things.

So we value freedom a finite amount. There are trade-offs of a certain amount of freedom for a certain amount of other goods that we already accept. It may be that there are other such trade-offs we would also accept, if we were offered them.

For example, suppose the government is considering a regulation to ban dumping mercury into the local river. This is a trade-off: I lose a certain amount of freedom in exchange for a certain amount of health. In particular, I lose the freedom to dump mercury into the river in exchange for the health benefits of not drinking poisoned water.

But I don’t really care that much about the freedom to dump mercury into the river, and I care a lot about the health benefits of not drinking poisoned water. So this seems like a pretty good trade-off.

And this generalizes to an answer to the original question. I completely agree freedom is an extremely important good, maybe the most important. I don’t agree it’s an infinitely important good, so I’m willing to consider trade-offs that sacrifice a small amount of freedom for a large amount of something else I consider valuable. Even the simplest laws, like laws against stealing, are of this nature (I trade my “freedom” to steal, which I don’t care much about, in exchange for all the advantages of an economic system based on private property).

The arguments above are all attempts to show that some of the trade-offs proposed in modern politics are worthwhile: they give us enough other goods to justify losing a relatively insignificant “freedom” like the freedom to dump mercury into the river.

12.1.1: But didn’t Benjamin Franklin say that those who would trade freedom for security deserve neither?

No, he said that those who would trade essential liberty for temporary security deserved neither. Dumping mercury into the river hardly seems like essential liberty. And when Franklin was at the Constitutional Convention he agreed to replace the minimal government of the Articles of Confederation with a much stronger centralized government just like everyone else.

12.2: Taxation is theft. And when the government forces you to work under their rules, for the amount of money they say you can earn, that’s slavery. Surely you’re not in favor of theft and slavery.

Consider the argument “How can we have a holiday celebrating Martin Luther King? After all, he was a criminal!”

Technically, Martin Luther King was a criminal, in that he broke some laws against public protests that the racist South had quickly enacted to get rid of him. It’s why he famously spent time in Birmingham Jail.

And although “criminal” is a very negative-sounding and emotionally charged word, in this case we have to step back from our immediate emotional reaction and notice that the ways in which Martin Luther King was a criminal don’t make him a worse person.

A philosopher might say we’re equivocating between two meanings of “criminal”, one meaning of “person who breaks the law”, and another meaning of “horrible evil person.” Just because King satisfies the first meaning (he broke the law) doesn’t mean he has to satisfy the second (be horrible and evil).

Or consider the similar argument: “Ayn Rand fled the totalitarian Soviet Union to look for freedom in America. That makes her a traitor!” Should we go around shouting at Objectivists “How can you admire Ayn Rand when she was a dirty rotten traitor“?

No. Once again, although “traitor” normally has an automatic negative connotation, we should avoid instantly judging things by the words we can apply to them, and start looking at whether the negative feelings are deserved.

Or once again the philosopher would say we should avoid equivocating between “traitor” meaning “someone who switches sides from one country to an opposing country” and “horrible evil untrustworthy person.”

Our language contains a lot of words like these which package a description with a moral judgment. For example, “murderer” (think of pacifists screaming it at soldiers, who do fit the technical definition “someone who kills someone else”), “greedy” (all corporations are “greedy” if you mean they would very much like to have more money, but politicians talking about “greedy corporations” manage to transform it into something else entirely) and of course that old stand-by “infidel”, which sounds like sufficient reason to hate a member of another religion, when in fact it simply means a member of another religion. It’s a stupid, cheap trick unworthy of anyone interested in serious rational discussion.

And calling taxation “theft” is exactly the same sort of trick. What’s theft? It’s taking something without permission. So it’s true that taxation is theft, but if you just mean it involves taking without permission, then everyone from Lew Rockwell up to the head of the IRS already accepts that as a given.

This only sounds like an argument because the person who uses it is hoping people will let their automatic negative reaction to theft override their emotions, hoping they will equivocate from theft as “taking without permission” to “theft as a terrible act worthy only of criminals”.

Real arguments aren’t about what words you can apply to things and how nasty they sound, real arguments about what good or bad consequences those things produce.

12.3: Government actions tend to involve the initiation of force against innocent people. Isn’t that morally wrong?

Why should it be morally wrong?

12.3.1: Because the initiation of force always has bad consequences, like ruining the economy or making people unhappy.

Sometimes it does. Other times it has good consequences.

Take cases like the fish farming, boycott, and charity scenarios above. There the use of force to solve the coordination problem meets an extraordinarily strict set of criteria: not only does it benefit the group as a whole, not only does it benefit every single individual in the group, but every single individual in the group knows that it benefits them and endorses that benefit (eg would vote for it).

In other cases, such as the retirement savings example above, the use of force meets only a less strict set of criteria: it benefits the group as a whole, it benefits every single individual in the group, but not every individual in the group necessarily knows that it benefits them or endorses that benefit. These are the cases libertarians might call “paternalism”.

Still more cases satisfy an even looser criterion. They benefit the group as a whole, but they might not benefit every single individual in the group, and might harm some of them. These are the cases that libertarians might call “robbing Peter to pay Paul”.

All three of these sets of cases belie the idea that the use of force must on net have bad consequences.

12.3.2: Okay, maybe it’s wrong because some moral theory that’s not about consequences tells me it’s wrong.

If your moral theory doesn’t involve any consequences, why follow it? It seems sort of like an arbitrary collection of rules you like.

The Jews believe that God has commanded them not to murder. They also believe God has commanded them not to start fires on Saturdays. Jews who lose their belief in God usually continue not to murder, but stop worrying about whether or not they light fires on Saturdays. Likewise, evangelical Christians believe stealing is a sin, and that homosexuality is also a sin. If they de-convert and become atheists, most of them will still oppose stealing, but most will stop worrying about homosexuality. Why?

Killing and stealing both have bad consequences; in fact, that seems to be the essence of why they’re wrong. Fires on Saturday and homosexuality don’t hurt anybody else, but killing and stealing do.

Why are consequences to other people seems such a specially relevant category? The argument is actually itself pretty libertarian. I can do whatever I want with my own life, which includes following religious or personal taboos. Other people can do whatever they want with their own lives too. The stuff that matters – the stuff where we have to draw a line in the sand and say “Nope, this is moral and this is immoral, doesn’t matter what you think” is because it has some consequence in the real world like hurting other people.

12.3.2.1: I was always taught that the essence of morality was the Principle of Non-Aggression: no one should ever initiate force, except in self-defense. What exactly is wrong with this theory?

At least two things. First, once you disentangle it from the respect it gets as the Traditional Culturally Approved Ground Of Morality, the actual rational arguments for it as a principle are surprisingly weak. Second, in order to do anything practical with it you need such a mass of exceptions and counter-exceptions and stretches that one starts to wonder whether it’s doing any philosophical work at all; it becomes a convenient hook upon which to hang our pre-existing prejudices rather than a useful principle for solving novel moral dilemmas.

12.3.2.1.1: What do you mean by saying that the rational arguments for the Principle of Non-Aggression are weak?

There are dozens of slightly different versions of these arguments, and I don’t want to get into all of them here, so I’ll concentrate on the most common.

Some people try to derive the Principle of Non-Aggression from self-ownership. But this is circular reasoning: the form of “private property” you need to own anything, including your self/body, is a very complicated concept and one that requires some form of morality in order to justify; you can’t use your idea of private property as a justification for morality. Although it’s obvious that in some sense you are your body, there’s no way to go from here to “And therefore the proper philosophical relationship between you and your body is the concept of property exactly as it existed in the 17th century British legal system.”

This also falls afoul of the famous is-ought dichotomy, the insight that just because something is true doesn’t mean it should be true. Just because we notice some factual relationship between yourself and your body doesn’t mean that relationship between yourself and your body is good or important or needs to be protected in laws. We might eventually decide it should be (and hopefully we will!) but we need to have other values in order to come to that decision; we can’t use the decision as a basis for our values.

The self-ownership argument then goes from this questionable assumption to other even more questionable ones. If you use your body to pick fruit, that fruit becomes yours, even though you didn’t make it. If you use your body to land on Tristan de Cunha and plant a flag there and maybe pick some coconuts, that makes Tristan de Cunha and everything on your property and that of your heirs forever, even though you definitely didn’t make the island. And if someone else lands on Tristan de Cunha the day after you, you by right control every facet of their life on the island and they have to do whatever you say or else leave. There are good arguments for why some of these things make economic sense, but they’re all practical arguments, not moral ones positing a necessary relationship.

Oddly enough, although apparently your having a body does license you to declare yourself Duke of Tristan de Cunha, it doesn’t license you to use your fist to punch your enemy in the gut, or use your legs to walk across a forest someone else has said they claim, even though your ability to move your hand rapidly in the direction of your enemy’s abdomen, or your feet along a forest path, seems like a much more fundamental application of your body than taking over an island.

All of these rules about claiming islands and not punching people you don’t like and so on are potentially good rules, but trying to derive them just from the fact that you have a body starts to seem a bit hokey.

12.3.2.1.2: What do you mean by saying that the Non-Aggression Principle requires so many exceptions and counter-exceptions that it becomes useless except as a hook upon which to hang prejudices we from other sources?

First, the principle only even slightly makes sense by defining “force” in a weird way. The NAP’s definition of “force” includes walking into your neighbor’s unlocked garden when your neighbor isn’t home and picking one of her apples. It includes signing a contract promising to deliver a barrel of potatoes, but then not delivering the potatoes when the time comes. Once again, I agree these are bad things that we need rules against. But it takes quite an imagination to classify them under “force”, or as deriving from the fact that you have a body. This is a good start to explaining what I mean when I say that people claim that they’re using the very simple-sounding “no initiation of force” principle but are actually following a more complicated and less justified “no things that seem bad to me even though I can’t explain why”.

Second, even most libertarians agree it can be moral to initiate force in certain settings. For example, if the country is under threat from a foreign invader or from internal criminals, most libertarians agree that it is moral to levy a small amount of taxation to support an army or police force that restores order. Again, this is a very good idea – but also a blatant violation of the Non-Aggression Principle. When libertarians accept the initiation of force to levy taxes for the police, but protest that initiating force is always wrong when someone tries to levy taxes for welfare programs, it reinforces my worry that the Non-Aggression Principle is something people claim to follow while actually following their own “no things that seem bad to me even though I can’t explain why, but things that seem good to me are okay” principle.

(I acknowledge that some libertarians take a stand against taxes for the military and the police. I admire their consistency even while I think their proposed policies would be a disaster.)

Third, when push comes to shove the Non-Aggression Principle just isn’t strong enough to solve hard problems. It usually results in a bunch of people claiming conflicting rights and judges just having to go with whatever seems intuitively best to them.

For example, a person has the right to live where he or she wants, because he or she has “a right to personal self-determination”. Unless that person is a child, in which case the child has to live where his or her parents say, because…um…the parents have “a right to their child” that trumps the child’s “right to personal self-determination”. But what if the parents are evil and abusive and lock the child in a fetid closet with no food for two weeks? Then maybe the authorities can take the child away because…um…the child’s “right to decent conditions” trumps the parents’ “right to their child” even though the latter trumps the child’s “right to personal self-determination”? Or maybe they can’t, because there shouldn’t even be authorities of that sort? Hard to tell.

Another example. I can build an ugly shed on my property, because I have a “right to control my property”, even though the sight of the shed leaves my property and irritates my neighbor; my neighbor has no “right not to be irritated”. Maybe I can build a ten million decibel noise-making machine on my property, but maybe not, because the noise will leave my property and disturbs neighbor; my “right to control my property” might or might not trump my neighbor’s “right not to be disturbed”, even though disturbed and irritated are synonyms. I definitely can’t detonate a nuclear warhead on my property, because the blast wave will leave my property and incinerates my neighbor, and my neighbor apparently does have a “right not to be incinerated”.

If you’ve ever seen people working within our current moral system trying to solve issues like these, you quickly realize that not only are they making it up as they go along based on a series of ad hoc rules, but they’re so used to doing so that they no longer realize that this is undesirable or a shoddy way to handle ethics.

12.4: Is there a better option than the Non-Aggression Principle?

Yes. It’s consequentialism, the principle that it is moral to do whatever has, on net, the best consequences. This is about equivalent to saying “to do whatever makes the world a better place”. It’s the principle we’ve been using implicitly throughout this FAQ and the principle most people use implicitly throughout their lives.

It’s also the principle that drives capitalism, where people are able to create incredible businesses and innovations because they are trying to do whatever has the best financial consequences for themselves. Consequentialism just takes that insight and says that instead of just doing it with money, let’s do it with everything we value.

12.4.1: Best consequences according to whom?

Well, if you’re the one making the moral decision, then best consequences according to you. All it’s saying is that your morality should be a reflection of your value system and your belief in a better world. Your job as a moral agent is to try to make the world a better place by whatever your definition of “better place” might be.

Sticking to the capitalism analogy, consumerism “tells you” (not that you need to be told) to get whatever goods you value most. Consequentialism does the same, but tells you to try to get the collection of abstract moral goods you value the most.

But remember our discussion of trade-offs above. Most people value many different moral goods, and you are no exception. If you’re trying to make the world a better place, you should be thinking about your relative valuation of all these goods and what trade-offs you are willing to make.

12.4.2: Best consequences for me, or best consequences for everyone?

Again, this is your decision. If you’re completely selfish, then consequentialism tells you to seek out the best consequences for yourself. This probably wouldn’t mean being a libertarian – thankless activism for an unpopular political position is really a terrible way to go about looking out for Number One. It would probably mean cheating off the government – either in the form of welfare abuse if you’re poor and lazy, or in the form of crony capitalism if you’re rich and ambitious. As icing on the cake, make sure to become a sanctimonious and hypocritical liberal, as it’s a great way to become popular and get invited to all the fancy parties.

But if you care about people other than yourself, consequentialism tells you to seek out the best consequences for the people you care about (which could be anything from your family to your country to the world). This could involve political activism, and it could even involve political activism in favor of libertarianism if you think it’s the best system of government.

Alternately, it could justify trying to start a government, if there’s no government yet and you think a world with government would be better for the people you care about than one without it.

Most of the rest of this section will be assuming you do in fact care for other people at least a little.

12.4.3: Since many people probably want different things and care about different people, don’t we end out in a huge war of all against all until either everyone is dead or one guy is dictator?

Would that be a good consequence? If not, people who try to promote good consequences and make the world a better place would try to avoid it.

Because this world of violence and competition is so obviously a bad consequence, any consequentialist who gives it a moment’s thought agrees not to start a huge war of all against all that ends with everyone dead or one guy as dictator by binding themselves by moral rules whenever binding themselves by those moral rules seems like it would have good consequences or make the world a better place; see Section 13 for more.

12.4.4: Doesn’t that sound a lot like “the ends justify the means”? Wouldn’t it lead to decadence, slavery, or some other dystopia?

Once again, if you consider dictatorship, slavery, and dystopia to be bad consequences, then by definition following this rule is the best way to avoid doing that.

The rule isn’t “do whatever sounds like it would have the best consequences if you have an IQ of 20 and refuse to think about it for even five seconds”, it’s “do what would actually have the best consequences. Sometimes this involves admitting human ignorance and fallibility and not pursuing every hare-brained idea that comes into your head.

12.4.5: Okay, okay, I understand that if people did what actually had good consequences it would have good consequences, but I worry that if people do what they think has good consequences, it will lead to violence and dictatorship and dystopia and all those other things you mentioned above.

Yes, I agree this is an important distinction. There are two uses for a moral system. The first is to define what morality is. The second is to give people a useful tool for choosing what to do in moral dilemmas. I am arguing that consequentialism does the first. I don’t think it does the second right out of the box.

To try a metaphor, doctors sometimes have two ways of defining disease; the gold standard and the clinical standard. The gold standard is the “perfect” test for the disease; for example, in Alzheimer’s disease, it’s to autopsy the brain after the person has died and see if it has certain features under the microscope. Obviously you can’t autopsy a person who’s still alive, so when doctors are actually trying to diagnose Alzheimer’s they use a more practical method, like how well the person does on a memory test.

Right now I’m arguing that consequentialism is the gold standard for morality: it’s the purest, most sophisticated explanation of what morality actually is. At the same time, it might be a terrible idea to make your everyday decisions based on it, just as it’s a terrible idea to diagnose Alzheimer’s with an autopsy in someone who’s still alive.

However, once we know that consequentialism is the gold standard for morality, we can start designing our clinical standards by trying to figure out which “clinical standard” for morality will produce the best consequences. See Section 13 for more.

12.4.6: I still am not completely on board with consequentialism, or I’m not sure I understand it.

For more information on consequentialism, see the sister document to this FAQ, the Consequentialism FAQ.

13. Rights and Heuristics

13.1: Is there a moral justification for rights, like the right to free speech or the right to property?

Yes. Rights are the “clinical standard” for morality, the one we use to make our everyday decisions after we acknowledge that pure consequentialism might not lead to the best consequences when used by fallible humans.

In this conception, rights are conclusions rather than premises. They are heuristics (heuristic = a rule-of-thumb that usually but not always works) for remembering what sorts of things usually have good or bad consequences, a distillation of moral wisdom that is often more trustworthy than morally fallible humans.

For example, trying to tell people what religions they can or can’t follow almost always has bad consequences. At best, people are miserable because they’re being forced to follow a faith they don’t believe in. At worst, they resist and then you get Inquisitions and Holy Wars and everyone ends up dead. Restriction of religion causing bad consequences is sufficiently predictable that we generalize it into a hard and fast rule, and call that rule something like the “right to freedom of religion”.

Other things like banning criticism of the government, trying to prevent people from owning guns, and seizing people’s property willy-nilly also work like this, so we call those “rights” too.

13.2: So if you think that violating rights will have good consequences, then it’s totally okay, right?

It’s not quite so simple. Rights are not just codifications of the insight that certain actions lead to bad consequences, they’re codifications of the insight that certain actions lead to bad consequences in ways that people consistently fail to predict or appreciate.

All throughout history, various despots and princes have thought “You know, the last hundred times someone tried to restrict freedom of religion, it went badly. Luckily, my religion happens to be the One True Religion, and I’m totally sure of this, and everyone else will eventually realize this and fall in line, so my plan to restrict freedom of religion will work great!”

Every revolution starts with an optimist who says “All previous attempts to kill a bunch of people and seize control of the state have failed to produce a utopia, but luckily my plan is much better and we’re totally going to get to utopia this time.” Or, as Huxley put it: “Only one more indispensable massacre of Capitalists or Communists or Fascists and there we are – there we are – in the Golden Future.”

So another way to put it is that rights don’t just say “Doing X has been observed to have bad consequences”, but also “Doing X has been observed to have bad consequences, even when smart people are quite certain it will have good consequences.”

13.3: Then even though you got to rights by a different route than the libertarians, it sounds like you agree with them that they’re inalienable.

It’s not as simple as that either. Every so often, the conventional wisdom is wrong. So many lunatics and crackpots spent their lives trying to turn lead into gold that it became a classic metaphor for a foolish wild goose chase. The rule “stop trying to transmute elements into each other, it never works” was no doubt a good and wise rule. If more would-be alchemists had trusted this conventional wisdom, and fewer had thought “No, even though everyone else has failed, I will be the one to discover transmutation”, it would have prevented a lot of wasted lives.

…and then we discovered nuclear physics, which is all about transmuting elements into one another, and which works very well and is a vital source of power. And yes, nuclear physicists at Berkeley successfully used a giant particle accelerator to turn lead into gold, although it only works a few atoms at a time and isn’t commercially viable.

The point is, the heuristic that you shouldn’t waste your life studying transmutation was a good one and very well-justified at the time, but if we had elevated it into a timeless and unbreakable principle, we never would have been able to abandon it after we learned more about nuclear physics and trying to transmute things was no longer so foolish.

Rights are a warning sign that we should not naively expect breaking them to have good consequences. In order to claim even the possibility of good consequences from violating a right, we need to be at least as far away from the actions they were meant to prevent as nuclear physics is to alchemy.

13.3.1: Can you give an example of a chain of reasoning where some government violation of a right is so radically different from the situation that led the right to exist in the first place?

Let’s take for example the right that probably dominates discussions between libertarians and non-libertarians: the right to property. On the individual scale, taking someone else’s property makes them very unhappy, as you know if you’ve ever had your bike stolen. On the larger scale, abandoning belief in private property has disastrous results for an entire society, as the experiences of China and the Soviet Union proved so conclusively. So it’s safe to say there’s a right to private property.

Is it ever acceptable to violate that right? In the classic novel Les Miserables, Jean Valjean’s family is trapped in bitter poverty in 19th century France, and his nephew is slowly starving to death. Jean steals a loaf of bread from a rich man who has more than enough, in order to save his nephew’s life. This is a classic moral dilemma: is theft acceptable in this instance?

We can argue both sides. A proponent might say that the good consequences to Jean and his family were very great – his nephew’s life was saved – and the bad consequences to the rich man were comparatively small – he probably has so much food that he didn’t even miss it, and if he did he could just send his servant to the bakery to get another one. So on net the theft led to good consequences.

The other side would be that once we let people decide whether or not to steal things, we are on a slippery slope. What if we move from 19th century France to 21st century America, and I’m not exactly starving to death but I really want a PlayStation? And my rich neighbor owns like five PlayStations and there’s no reason he couldn’t just go to the store and buy another. Is it morally acceptable for me to steal one of his PlayStations? The same argument that applied in Jean Valjean’s case above seems to suggest that it is – but it’s easy to see how we go from there to everyone stealing everyone’s stuff, private property becoming impossible, and civilization collapsing. That doesn’t sound like a very good consequence at all.

If everyone violates moral heuristics whenever they personally think it’s a good idea, civilization collapses. If no one ever violates moral heuristics, Jean Valjean’s nephew starves to death for the sake of a piece of bread the rich man never would have missed.

We need to bind society by moral heuristics, but also have some procedure in place so that we can suspend them in cases where we’re exceptionally sure of ourselves without civilization instantly collapsing. Ideally, this procedure should include lots of checks and balances, to make sure no one person can act on her own accord. It should reflect the opinions of the majority of people in society, either directly or indirectly. It should have access to the best minds available, who can predict whether violating a heuristic will be worth the risk in this particular case.

Thus far, the human race’s best solution to this problem has been governments. Governments provide a method to systematically violate heuristics in a particular area where it is necessary to do so without leading to the complete collapse of civilization.

If there was no government, I, in Jean Valjean’s situation, absolutely would steal that loaf of bread to save my nephew’s life. Since there is a government, the government can set a certain constant amount of theft per year, distribute the theft fairly among people whom it knows can bear the burden, and then feed starving children and do other nice things. The ethical question of “is it ethical for me to steal/kill/stab in this instance?” goes away, and society can be peaceful and stable.

13.3.2: So you’re saying that you think in this case violating the right will have good consequences. But you just agreed that even when people think this, violating the right usually has bad consequences.

Yes, I admit it’s complicated. But we have to have some procedures for violating moral heuristics, or else we can’t tax to support a police force, we can’t fight wars, we can’t lie to a murderer who asks us where our friend is so he can go kill her when he finds her, and so on.

The standard I find most reasonable is when it’s universalizable and it avoids the issue that caused us to develop the heuristic in the first place.

By universalizable, I mean that it’s more complicated than me just deciding “Okay, I’m going to steal from this guy now”. There has to be an agreed-upon procedure where everyone gets input, and we need to have verified empirically that this procedure usually leads to good results.

And is has to avoid the issue that caused us to develop the heuristic. In the case of stealing, this is that theft makes property impossible or at least impractical, no one bothers doing work because it will all be stolen from them anyway, and so civilization collapses.

In the case of theft, taxation requires authorization by a process that most of us endorse (the government set up by the Constitution) and into which we all get some input via representative democracy. It doesn’t cause civilization to collapse because it only takes a small and extremely predictable amount from each person. And it’s been empirically verified to work: as I argued above, countries with higher tax rates like Scandinavia actually are nicer places to live than countries with lower tax rates like the United States. So we’ve successfully side-stepped the insight that stealing usually has bad consequences, even though we recognize that the insight remains true.

13.4: Governments will inevitably make mistakes when deciding when to violate moral heuristics. Those mistakes will cost money and even lives.

And the policy of never, ever doing anything will never be a mistake?

It’s very easy for governments to make devastating mistakes. For example, many people believe the US government’s War in Iraq did little more than devastate the country, kill hundreds of thousands of Iraqis, and replace Saddam with a weak government unable to stand up to extremist ayatollahs.

But the other solution – never intervening in a foreign country at all – didn’t work so well either. Just look at Holocaust-era Germany, or 1990s Rwanda.

Why, exactly, should moral questions be simple?

There is a certain tradition that the moral course of action is something anyone, from the high priest unto the youngest child, can find simply by looking deep in his heart. Anyone who does not find it in his heart is welcome to check the nearest Giant Stone Tablet, upon which are written infallible rules that can guide him through any situation. Intelligence has nothing to do with it. It should be blindingly obvious, and anyone who claims it has a smidgen of difficulty or vagueness is probably an agent of the Dark Lord, trying to seduce you from the True Path with his lies.

And so it is tempting to want to have some really easy principle like “Never get involved in a foreign war” and say it can never lead you wrong. It makes you feel all good and warm and fuzzy and moral and not at all like those evil people who don’t have strong principles. But real life isn’t that simple. If you get involved in the wrong foreign war, millions of people die. And if you don’t get involved in the right foreign war, millions of people also die.

So you need to have good judgment if you want to save lives and do the right thing. You can’t get a perfect score in morality simply by abdicating all responsibility. Part of the difficult questions that all of us non-libertarians have been working on is how to get a government that’s good at answering those sorts of questions correctly.

13.5: No, there’s a difference. When you enter a foreign war, you’re killing lots of people. When you don’t enter a foreign war, people may die, but it’s not your job to save them. The government’s job is only to protect people and property from force, not to protect people from the general unfairness of life.

Who died and made you the guy who decides what the government’s job is? Or, less facetiously: on what rational grounds are you making that decision?

Currently, several trillion dollars are being spent to prevent terrorism. This seems to fall within the area of what libertarians would consider a legitimate duty of government, since terrorists are people who initiate force and threaten our safety and the government needs to stop this. However, terrorists only kill an average of a few dozen Americans per year.

Much less money is being spent on preventing cardiovascular disease, even though cardiovascular disease kills 800,000 Americans per year.

Let us say, as seems plausible, that the government can choose to spend its money either on fighting terrorists, or on fighting CVD. And let us say that by spending its money on fighting terrorists, it saves 40 lives, and by spending the same amount of money on fighting CVD, it saves 40,000 lives.

All of these lives, presumably, are equally valuable. So there is literally no benefit to spending the money on fighting terrorism rather than CVD. All you are doing is throwing away 39,960 lives on an obscure matter of principle. It’s not even a good principle – it’s the principle of wanting to always use heuristics even when they clearly don’t apply because it sounds more elegant.

There’s a reason this is so tempting. It’s called the Bad Guy Bias, and it’s an evolutionarily programmed flaw in human thinking. People care much more about the same amount of pain when it’s inflicted by humans than when it’s inflicted by nature. Psychologists can and have replicated this in the lab, along with a bunch of other little irrationalities in human cognition. It’s not anything to be ashamed of; everyone’s got it. But it’s not something to celebrate and raise to the level of a philosophical principle either.

13.6: Stop calling principles like “don’t initiate force” heuristics! These aren’t some kind of good idea that works in a few cases. These are the very principles of government and morality , and it’s literally impossible for them to guide you wrong!

Let me give you a sketch of one possible way that a libertarian perfect world that followed all of the appropriate rules to the letter could end up as a horrible dystopia. There are others, but this one seems most black-and-white.

Imagine a terrible pandemic, the Amazon Death Flu, strikes the world. The Death Flu is 100% fatal. Luckily, one guy, Bob, comes up with a medicine that suppresses (but does not outright cure) the Death Flu. It’s a bit difficult to get the manufacturing process right, but cheap enough once you know how to do it. Anyone who takes the medicine at least once a month will be fine. Go more than a month without the medicine, and you die.

In a previous version of this FAQ, Bob patented the medicine, and then I got a constant stream of emails saying (some) libertarians don’t believe in patents. Okay. Let’s say that Bob doesn’t patent the medicine, but it’s complicated to reverse engineer, and it would definitely take more than a month. This will become important later.

Right now Bob is the sole producer of this medicine, and everyone in the world needs to have a dose within a month or they’ll die. Bob knows he can charge whatever he wants for the medicine, so he goes all out. He makes anyone who wants the cure pay one hundred percent of their current net worth, plus agree to serve him and do anything he says. He also makes them sign a contract promising that while they are receiving the medicine, they will not attempt to discover their own cure for the Death Flu, or go into business against him. Because this is a libertarian perfect world, everyone keeps their contracts.

A few people don’t want to sign their lives away to slavery, and refuse to sign the contract. These people receive no medicine and die. Some people try to invent a competing medicine. Bob, who by now has made a huge amount of money, makes life difficult for them and bribes biologists not to work with them. They’re unable to make a competing medicine within a month, and die. The rest of the world promises to do whatever Bob says. They end up working as peons for a new ruling class dominated by Bob and his friends.

If anyone speaks a word against Bob, they are told that Bob’s company no longer wants to do business with them, and denied the medicine. People are encouraged to inform on their friends and families, with the promise of otherwise unavailable luxury goods as a reward. To further cement his power, Bob restricts education to the children of his friends and strongest supporters, and bans the media, which he now controls, from reporting on any stories that cast him in a negative light.

When Bob dies, he hands over control of the medicine factory to his son, who continues his policies. The world is plunged into a Dark Age where no one except Bob and a few of his friends have any rights, material goods, or freedom. Depending on how sadistic Bob’s and his descendants are, you may make this world arbitrarily hellish while still keeping perfect adherence to libertarian principles.

Compare this to a similar world that followed a less libertarian model. Once again, the Amazon Death Flu strikes. Once again, Bob invents a cure. The government thanks him, pays him a princely sum as compensation for putting his cure into the public domain, opens up a medicine factory, and distributes free medicine to everyone. Bob has become rich, the Amazon Death Flu has been conquered, and everyone is free and happy.

13.6.1: This is a ridiculously unlikely story with no relevance to the real world.

I admit this particular situation is more a reductio ad absurdum than something I expect to actually occur the moment people start taking libertarianism seriously, but I disagree that it isn’t relevant.

The arguments that libertarianism will protect our values and not collapse into an oppressive plutocracy require certain assumptions: there are lots of competing companies, zero transaction costs, zero start-up costs, everyone has complete information, everyone has free choice whether or not to buy any particular good, everyone behaves rationally, et cetera. The Amazon Death Flu starts by assuming the opposite of all of these assumptions: there is only one company, there are prohibitive start-up costs, a particular good absolutely has to be bought, et cetera.

The Amazon Death Flu world, with its assumptions, is not the world we live in. But neither is the libertarian world. Reality lies somewhere between the “capitalism is perfect” of the one, and the “capitalism leads to hellish misery” of the other.

There’s no Amazon Death Flu, but there are things like hunger, thirst, unemployment, normal diseases, and homelessness. In order to escape these problems, we need things provided by other people or corporations. This is fine and as it should be, and as long as there’s a healthy free market with lots of alternatives, in most cases these other people or corporations will serve our needs and society’s needs while getting rich themselves, just like libertarians hope.

But this is a contingent fact about the world, and one that can sometimes be wrong. We can’t just assume that the heuristic “never initiate force” will always turn out well.

13.7: The government doesn’t need to violate moral heuristics. In the absence of government programs, private charity would make up the difference.

Find some poor people in a country without government-funded welfare, and ask how that’s working out for them.

Private charity from the First World hasn’t prevented the Rwandans, Ethiopians, or Haitians from dying of malnutrition or easily preventable disease.

It’s possible that this is just because we First Worlders place more importance on our own countrymen than on foreigners, and if Americans were dying of malnutrition or easily preventable disease, patriotism would make us help them.

The US government currently spends about $800 billion on welfare-type programs for US citizens. Americans give a total of $300 billion to charity per year.

Let’s assume that private charity is twice as efficient as the government (in reality, it’s probably much less, since the government has economies of scale, but libertarians like assumptions like this and I might as well indulge them).

Let’s also assume that only half of charity goes to meaningful efforts to help poor American citizens. The other half would be things like churches, the arts, and foreign countries.

Nowadays, a total of $550 billion (adjusted, govt+private) goes to real charity (800b*1/2+300b*1/2). If the government were to stop all welfare programs, this number would fall to $150 billion (adjusted). Private citizens would need to make up the shortfall of $400 billion to keep charity at its current (woefully low) level. Let’s assume that people, realizing this, start donating a greater proportion (66%) of their charity to the American poor instead of to other causes. That means people need to increase their charity to about $830 billion ([400b + 150b]/.66).

Right now, 25% is a normal middle-class tax rate. Let’s assume the government stopped all welfare programs and limited itself to defense, policing, and overhead. There are a lot of different opinions about what is and isn’t in the federal budget, but my research suggests that would cut it by about half, to lower tax rates to 12.5%.

So, we’re in the unhappy situation of needing people to almost triple the amount they give to charity even though they have only 12.5% more money. The real situation is much worse than this, because if the government stopped all programs except military and police, people would need to pay for education, road maintenance, and so on out of their own pocket.

My calculations are full of assumptions, of course. But the important thing is, I’ve never seen libertarians even try to do calculations. They just assume that private citizens would make up the shortfall. This is the difference between millions of people leading decent lives or starving to death, and people just figure it will work out without checking, because the free market is always a Good Thing.

That’s not reason, even if you read it on www.reason.com. That’s faith.

13.8: People stupid enough to make bad decisions deserve the consequences of their actions. If government bans them from making stupid decisions, it’s just preventing them from getting what they deserve.

One of my favorite essays, Policy Debates Should Not Appear One-Sided, provides a much better critique of this argument than I could. It starts by discussing a hypothetical in which the government stopped regulating the safety of medicines. Some quack markets sulfuric acid as medicine, and a “poor, honest, not overwhelmingly educated mother of five children” falls for it, drinks it, and dies.

If you were really in that situation, would you really laugh, say “Haha, serves her right” and go back to what you were doing? Or would it be a tragedy even though she “got what she deserved”?

The article ends by saying:

Saying ‘People who buy dangerous products deserve to get hurt!’ is not tough-minded. It is a way of refusing to live in an unfair universe. Real tough-mindedness is saying, ‘Yes, sulfuric acid is a horrible painful death, and no, that mother of 5 children didn’t deserve it, but we’re going to keep the shops open anyway because we did this cost-benefit calculation.’…I don’t think that when someone makes a stupid choice and dies, this is a cause for celebration. I count it as a tragedy. It is not always helping people, to save them from the consequences of their own actions; but I draw a moral line at capital punishment. If you’re dead, you can’t learn from your mistakes.

Read also about the just-world fallacy. “Making a virtue out of necessity” shouldn’t go as far as celebrating deaths if it makes your political beliefs more tenable.

Part E: Practical Issues

The Argument: Allowing any power to government is a slippery slope toward tyranny. No matter what the costs or benefits of any particular proposal, libertarians should oppose all government intrusion as a matter of principle.

The Counterargument: This fundamentally misunderstands the ways that nations collapse into tyranny. It also ignores political reality, and it doesn’t work. Libertarians should cooperate with people from across the ideological spectrum to oppose regulations that doesn’t work and keep an open mind to regulation that might.

14. Slippery Slopes

14.1: I’m on board with doing things that have the best consequences. And I’m on board with the idea that some government interventions may have good consequences. But allowing any power to government is a slippery slope. It will inevitably lead to tyranny, in which do-gooder government officials take away all of our most sacred rights in order to “protect us” from ourselves.

History has never shown a country sinking into dictatorship in the way libertarians assume is the “natural progression” of a big-government society. No one seriously expects Sweden, the United Kingdom, France, or Canada to become a totalitarian state, even though all four have gone much further down the big-government road than America ever will.

Those countries that have collapsed into tyranny have done so by having so weak a social safety net and so uncaring a government that the masses felt they had nothing to lose in instituting Communism or some similar ideology. Even Hitler gained his early successes by pretending to be a champion of the populace against the ineffective Weimar regime.

Czar Nicholas was not known for his support of free universal health care for the Russian peasantry, nor was it Chiang Kai-Shek’s attempts to raise minimum wage that inspired Mao Zedong. It has generally been among weak governments and a lack of protection for the poor where dictators have found the soil most fertile for tyranny.

14.1.1: But still, if we let down our guard, bureaucrats and politicians will have free rein to try to institute such a collapse into dictatorship.

I have always found the libertarian conviction that all politicians are secretly trying to build up their own power base to 1984-ish levels a bit weird.

All the time, I am hearing things like “No one really believes in global warming. It’s just a plot by the government to expand control over more areas of your life.” Or “since private charity is a threat to government’s domination of social welfare, once government gets powerful enough it will try to ban all private charity.”

Sure, people really do like power. But usually it’s the sort of power that comes with riches, fame, and beautiful women willing to attend to your every need. Just sitting in your office, knowing in an abstract way that because of you a lot of people who might otherwise be doing useful industry are fretting about their carbon emissions – that’s not the kind of power people sell their souls for. The path to ultimate domination of all humanity does not lead through the Dietary Fiber Levels in Food Act of 2006.

Most folk like to think of themselves as good people. Sure, they may take a bribe or two here, and have an affair or two there, and lie about this and that, “but only for the right reasons.” The thought process “Let me try to expand this unnecessary program so I can bathe in the feeling of screwing American taxpayers out of more of their hard-earned money” is not the kind that comes naturally, especially in a society where it leads to minimal personal gain. A politician who raises your taxes can’t use the money to buy himself a new Ferrari. At least, he can’t do it directly, and if he really wants that Ferrari there have got to be much easier ways to get it.

Human beings find it hard to get angry at a complicated system, and prefer to process things in terms of evil people doing evil things. Eliezer Yudkowsky of Less Wrong writes:

Suppose that someone says “Mexican-Americans are plotting to remove all the oxygen in Earth’s atmosphere.” You’d probably ask, “Why would they do that? Don’t Mexican-Americans have to breathe too? Do Mexican-Americans even function as a unified conspiracy?” If you don’t ask these obvious next questions when someone says, “Corporations are plotting to remove Earth’s oxygen,” then “Corporations!” functions for you as a semantic stopsign.

And if you don’t ask some of these same questions when someone says “Government wants to take away freedom!,” then you’re not thinking of government as a normal human institution that acts in normal human ways.

15. Strategic Activism

15.1: All you’ve argued so far is that it’s possible, in theory, for an ideal government making some very clever regulations to do a little more good than harm. But that doesn’t prove that the real government does more good than harm, and in fact it’s probably the opposite. So shouldn’t we admit that in a hypothetical perfect world government might do some good, while still being libertarians in reality?

I think if you’ve got enough intelligence and energy to be a libertarian, a better use of that intelligence and energy would be to help enact a properly working system.

15.2: It’s impossible to improve government; because power corrupts, all conceivable forms of government will be ineffective, wasteful, and dishonest.

“Impossible” is a really strong word.

Economist Robin Hanson has a proposal for a market-based open-source form of government called “futarchy”, in which government policies are decided entirely by a prediction market. Prediction markets operate similarly to stock markets and allow participants to buy or sell shares in predictions – for example, a share that pays out $100 if the economy improves this year, but $0 if the economy deteriorates. If it settles around a price of $60, this means the investing public predicts as 60% chance that the economy will go up.

A prediction market could be used to set policy by predicting its effects: for example, by comparing the prices of “we will institute the president’s economic plan, and the economy will improve” , “we will not institute the president’s economic plan, and the economy will improve” and “we will institute the president’s economic plan”, we can determine the public’s confidence that the president’s plan will improve the economy. There are some nifty theorems of economics that prove that such a market would produce a more accurate estimate of the plan’s chances than any other conceivable method (including consulting experts), and that it would be very difficult to corrupt. You can read more about it here.

My point isn’t that futarchy would definitely work. It’s that it’s an example of some of the best ideas that smart people trying to improve government can come up with. And unless you’re creative enough to develop futarchy on your own, or well-read enough to be sure you’ve heard of it and everything else like it, you’re being premature in calling improvements in government “impossible”.

15.3: Even if there are ways to improve government, they are impractical because they’re too politically unpopular.

Let’s be totally honest here. The US Libertarian Party currently has a grand total of zero state legislators, zero state governors, zero representatives, and zero senators. It’s never gotten much above one percent in any presidential election. Nor have any successful or nationally known major-party candidates endorsed genuinely libertarian ideals except maybe Ron Paul, who just suffered his third landslide defeat.

The libertarian vision of minimal government is politically impossible to enact. This is not itself an argument against it – most good ideas are – but it does mean you can’t condemn the alternatives for being politically impossible to enact.

Incremental attempts to improve government have a much better track record, both in terms of political palatability and success rate, than libertarian efforts to dismantle government whole-cloth. If you want to focus on something that might work, you should concentrate your efforts there.

15.4: Isn’t it better to draw a line in the sand and say no government intervention at all? This keeps us off the slippery slope to the kind of awful, huge government we have today.

Empirically, no. Again I point out that libertarianism has been completely ineffective as a political movement. The line-in-the-sand idea is an interesting one but obviously hasn’t worked.

And there are some serious advantages to erasing it. If non-libertarians see libertarians as ideologues who hate all government programs including the ones that could work, then they will dismiss any particular libertarian objection as meaningless: why pay attention to the fact that a libertarian hates this particular bill, when she hates every bill?

But if libertarians took a principled stand in favor of some government regulation that might work, they could credibly say “Look, it’s not that we have a knee-jerk hatred for all possible regulations, it’s just that this particular regulation is a horrible idea.” And people might listen.

It might also help arrest the polarization of society into factions who apply ideological “litmus tests” to all proposals before even hearing them out (eg pretty much all self-described “progressives” will automatically support any proposal to be tougher on pollution without even looking at what the economic costs versus health benefits will be, and most self-described libertarians will automatically oppose it just as quickly.) This sort of thing needs to stop, libertarians are one of the at least two groups who need to stop it, and the more people who stop, the more people on both sides will notice what they’re doing and think about it a little harder.

16. Miscellaneous and Meta

16.1: I still disagree with you. How should I best debate you and other non-libertarians in a way that is most likely to change your mind?

The most important advice I could give you is don’t come on too strong. Words like “thievery” and “enslave” are emotional button pressers, not rational arguments. Attempts to insult your opponents by calling them tyrants or suggesting they want to rule over the rest of humanity as slaves and cattle (yes, I’ve gotten that) is more likely to annoy than convince. And please, stop the “1984” references, especially when you’re talking about a modern liberal democracy. Seriously. It’s like those fundamentalists who have websites about how not having prayer in school is equivalent to the Holocaust.

Many non-libertarians aren’t going to be operating from within the same moral system you are. Sometimes the libertarians I debate don’t realize this and this causes confusion when they try to argue that something’s morally wrong. If you want to convince your opponent on moral grounds, you’re either going to have to show how their theories fail even by their own moral standards, or else prove your standards are right by deriving them from first principles (warning: this might be impossible).

Don’t immediately assume that just because we are not libertarians, we must worship Stalin, love communism, think government should be allowed to control every facet of people’s lives, or even support things like gun control or the War on Drugs. Non-libertarianism is a lot like non-Hinduism: it’s a pretty diverse collection of viewpoints with everything from full-on fascists to people who are totally libertarian except about one tiny thing.

Finally, you may have better luck convincing us of specific points, like “Government should not set a minimum wage” than broad slogans, like “Government can never do anything right.” It’s really hard to prove a universal negative.

16.2: Where can I go to see a rebuttal to this FAQ?

David Friedman wrote a short response here

Bryan Caplan wrote a response to some of the points about labor here.

Sarah wrote a longer rebuttal here: Why You Shouldn’t Hate My Freedom.

And Nintil wrote another long rebuttal here: The Non-Non Libertarian FAQ

If you’ve written another rebuttal or you know of one, email me and I’ll add it here.

16.3: Where can I go to find more non-libertarian information?

Mike Huben has a terrifyingly large collection of non-libertarian and anti-libertarian material of wildly varying quality and tone at his website.

This entry was posted in Uncategorized and tagged , , . Bookmark the permalink.

124 Responses to [REPOST] The Non-Libertarian FAQ

  1. Philosophisticat says:

    I agree with most of this, but I think there are some mistakes in the section on moral issues, specifically on consequentialism. To start:

    First, consequentialism is not the view that everyone ought to do whatever has the best consequences according to their own value system. It says that everyone ought to do whatever has the best consequences. People may have morally horrific value systems. Pretty much no self-respecting consequentialist would recommend that someone who values the suffering and death of innocent people should therefore make innocent people suffer and die. If Scott wants to use “consequentialism” in a nonstandard way, this should be flagged.

    Second, it is both false that consequentialism is the theory most people use, and dialectically strange to criticize the non-aggression principle on the basis of having counterintuitive implications, if one is going to propose consequentialism as the preferred alternative. One of consequentialism’s most famous problems concerns its inability to capture agent-relative reasons, like reasons to favor one’s own children, reasons to keep one’s own promises, etc. Everybody makes decisions on the basis of agent-relative reasons all the time, and consequentialist claims on these points are about as counterintuitive as it gets.

    • ChelOfTheSea says:

      Unless we’re taking consequentialism to a radical “donate everything you own to the poor” limit, agent-relative behaviors aren’t hard to explain. They increase social bonds (really important for personal utility) and provide social shielding against Moloch.

      • Philosophisticat says:

        If I think that the world is better with more social bonds, that does not involve genuinely agent-relative reasons, on a consequentialist picture, because agent relative reasons are reasons to respect MY social bonds, not reasons to promote social bonds as such. A genuinely agent relative reason to treat my children is special will tell me to do so even if it prevents two other people from treating their children as special, and hence even if it decreases the (agent neutral) value promoted by social bonds.

        • ChelOfTheSea says:

          I’m not saying they’re necessarily a net global utility gain. I’m saying that even consequentialists usually reserve some level of well-being for themselves in their own lives (e.g., Scott’s 10%-but-not-more charity thing).

          • Philosophisticat says:

            There are satisficing forms of consequentialism – these allow people not to maximize the value of consequences. But they do not entail that there are special moral reasons to keep promises or to take care of ones’ own children. So they still fail to capture the thing I’m talking about.

            There are also maximizing consequentialists who think we shouldn’t SAY that people shouldn’t treat their children as special, or break their promises whenever the consequences are better, because saying so would have bad consequences. This does not entail that there are special moral reasons to keep promises or to take care of ones’ own children. So they still fail to capture the thing I’m talking about.

            Rule consequentialists try to capture these agent-relative reasons, but they are a separate topic – their problems turn out to be even worse than the act consequentialist’s.

            And of course, there are many people who are just bad consequentialists: who do not act as their theory dictates they morally should. Many of them, including Peter Singer, openly admit it.

          • Wrong Species says:

            Scott has admitted that he doesn’t have a principled reason not to give more to charity.

            A Series Of Unprincipled Exceptions

        • phil says:

          Should I give any weight to the fact that I’m well positioned to know how to effectively help people the closer I am to them?

          ie, I understand my own set of circumstances best, so I can very effectively increase my own happiness (simply reflecting on things I should be grateful for usually does the trick)

          I understand my spouse and children, less well than I understand myself, but still, I spend a ton of time with them, and understand the general issues going on in their life pretty well, energy directed towards improving their happiness is usually pretty well targeted

          so on and so on ….

          people on the other side of the world, I don’t really understand that well, I could throw some money at their problems, but as usual when you throw money at problems, then you just have a different set of problems

          • Philosophisticat says:

            Yes, consequentialists sometimes point to things like this. But it doesn’t come close to capturing the ordinary commitment to agent relative reasons. People tend to think they have reasons to treat their own family as special even if they could help others more.

            Similarly, breaking a promise tends to make people sad and harm them, and consequentialists can take that into account, but it doesn’t come close to capturing the moral force of promise keeping. Intuitively, you have a reason to keep your promise greater than the mere harm it would cause – for example, you have greater reason to keep your promise than to make sure someone else keeps their promise.

          • IrishDude says:

            I’m curious, what’s your moral philosophy Philosophisticat?

          • Philosophisticat says:

            @Irishdude

            I don’t think anyone has found the right moral theory yet. All the views I know have pretty conclusive objections against them.

          • IrishDude says:

            Do you have morals? If so, how did you come to them?

          • Wrong Species says:

            There’s a difference between having ethical beliefs and having a fully consistent ethical belief system.

          • Philosophisticat says:

            @Irishdude

            As wrongspecies says, not being confident in any particular comprehensive moral theory is not the same as having no morals. I come to moral beliefs through something like reflective equilibrium, examining my strongest moral intuitions, their relationships, their implications, trying to improve coherence and eliminate inconsistency, etc.

          • IrishDude says:

            @Philosophisticat

            What’s your biggest objection to your method of determining right and wrong? Seems like a reasonable approach to me.

          • Philosophisticat says:

            @IrishDude

            Some variation on “garbage in, garbage out”. I don’t think any other way works, though.

          • IrishDude says:

            Some variation on “garbage in, garbage out”. I don’t think any other way works, though.

            Do you think you can detect garbage out in any way?

    • T says:

      I never see this included within the definition of consequentialsm that I see presented for uses such as these. Taking a look at https://en.wikipedia.org/wiki/Consequentialism, I don’t see it included in the definition at the top, although it is mentioned for other definitions below.

      Perhaps other areas have consequentialism closer to utilitarianism as it seems you do, although in this area I only see it as meaning a very basic “judge things based on their consequences. Not their intent, arbitrary right/wrongness, etc”.

      • Philosophisticat says:

        I don’t know what the referent of “this” is here.

        • Spookykou says:

          First, consequentialism is not the view that everyone ought to do whatever has the best consequences according to their own value system. It says that everyone ought to do whatever has the best consequences.

          This, I imagine.

          Edit: But you seem to respond to the fundamental consequentialism =/= utilitarianism claim that T is making farther down the thread so feel free to ignore this!

    • Jugemu says:

      >First, consequentialism is not the view that everyone ought to do whatever has the best consequences according to their own value system. It says that everyone ought to do whatever has the best consequences.

      Values aren’t up for grabs – no amount of moral reasoning can convince someone to adopt your bottom level values in preference to theirs. The best you can hope for is some combination of negotiation/trade/threats.

      It makes sense to try to get someone else to adopt consequentialism if you think there will be mutual benefit.

      (edit: I see consequentialism basically as just the definition of theoretically correct reasoning from values to actions in terms of maximizing values in the presence of sufficient computational power. Other moral theories basically act as approximations. It’s like saying the correct way to play a game like chess is via exhaustive search of the game tree. Always works in theory but only practical in some cases. And all this is still admittedly an oversimplification because it assumes human values are well-defined with all parts of the brain in agreement.)

      • Philosophisticat says:

        Thinking it is impossible to convince someone with repellent fundamental values to adopt yours is not the same as, nor entails, thinking that they ought to act according to those repellent values.

        It’s also important to distinguish consequentialism from maximizing expected utility – a decision-theoretic notion which is compatible with consequentialist or nonconsequentialist theories of values. People here often mistakenly conflate the two, I’ve noticed.

        • Jugemu says:

          >Thinking it is impossible to convince someone with repellent fundamental values to adopt yours is not the same as, nor entails, thinking that they ought to act according to those repellent values.

          Yes everyone ought to act according to the correct values (ie mine), agreed.

    • meltedcheesefondue says:

      >One of consequentialism’s most famous problems concerns its inability to capture agent-relative reasons, like reasons to favor one’s own children, reasons to keep one’s own promises,

      For favouring your own children, there’s no problem. You can put any set of consequences in the desirable outcome slot. Don’t confuse consequentialism with its much smaller but popular subset utilitarianism.

      As for promises, in a world where you have reputations, you want to be seen as likely to keep your promises. Think Tit-for-Tat in the iterated prisoners’ dilemma. Even for one-off promises (or the last promise in a sequence) it benefits you if you can demonstrate that you are likely to keep that promise. So if you can transform yourself in a known way into someone who keeps their promises, you would, on consequentialist grounds. Since humans are not completely opaque to each other, we can already do this (see http://lesswrong.com/lw/l1b/newcomblike_problems_are_the_norm/ and various discussions of Parffit’s Hitchhiker).

      >it is both false that consequentialism is the theory most people use

      Most people use consequentialism considerably in their everyday lives, and often in general situations without realising it. See the just world fallacy, for example; that’s an attempt to pretend that a favoured policy has only positive consequences. This makes no sense from a non-consequentialist perspective (why would you care about the consequences of your True Morality or True Virtue?) but makes sense if we see people as having partial consequentialist desiderata which they are trying to reconcile with their other reasons for following their Morality/ Virtue.

      • Philosophisticat says:

        If you allow anything, including agent-relative features of one’s action, to count as part of the consequences of that action, then basically every theory counts as a kind of consequentialism and the term loses any significance. For this reason, it’s better to understand consequentialism as the set of views where what one ought to do is a function of the agent-neutral value of the world that results from one’s action. This includes classical utilitarianism, but includes much more besides, since it’s compatible with all sorts of views about the value of worlds, and about the relationship between the value of a world and the action you ought to perform. But, appropriately, unlike the degenerate understanding you suggest, it doesn’t include the theories which have paradigmatically been contrasted to consequentialism. For fairness’ sake, I should note that there is some dispute in the literature over this.

        Thinking that consequences are morally relevant is not the same as implicitly accepting consequentialism. Ordinary moral reasoning acknowledges both reasons that stem from consequences and reasons that do not stem from consequences, and every plausible theory holds consequences to have at least derivative moral relevance.

        You’re misunderstanding the just world fallacy. It is not the fallacy of thinking that a favored policy only has positive consequences. Also, if one is interested in defending a view by noting that ordinary people act in accordance with it, it would probably be a bad move to focus on behavior we describe with the word “fallacy”.

        • meltedcheesefondue says:

          >If you allow anything, including agent-relative features of one’s action, to count as part of the consequences of that action, then basically every theory counts as a kind of consequentialism and the term loses any significance.

          Indeed: Virtue Ethics/Deontology in Utility-based Consequentialism: Your utility is 1 if you take actions that obey the rules/the virtuous actions, 0 if you take any actions that don’t. Utility-based Consequentialism in Virtue Ethics/Deontology: The prescribed/virtuous action is the one that maximises expected utility.

          But saying that consequences can be agent-based seems a reasonable requirement, not a twisting of the whole point of consequentialism. Universalism is not a trivial or obvious requirement of consequentialism, especially as virtue ethics and deontology have similar variants where only some people count as morally relevant, and some universalist formulations (eg “Universal Declaration of Human Rights”).

          >It is not the fallacy of thinking that a favored policy only has positive consequences.

          Saying that “if someone has bad consequences, they must have deserved it” seems to me the same as saying “those bad consequences aren’t *really* bad consequences”. But feelings on that may differ.

          >Also, if one is interested in defending a view by noting that ordinary people act in accordance with it, it would probably be a bad move to focus on behavior we describe with the word “fallacy”.

          If people didn’t have at least some consequentialist leanings, the fallacy would make no sense and wouldn’t exist.

        • blacktrance says:

          If you allow anything, including agent-relative features of one’s action, to count as part of the consequences of that action, then basically every theory counts as a kind of consequentialism and the term loses any significance.

          Not views in which there are genuine moral dilemmas, i.e. ones in which it is sometimes impossible not to act wrongly. Also, consequentialism is about consequences being the sole determining factor of rightness, and many theories reject both the agent-relative and neutral forms of that.

          • Philosophisticat says:

            You can come up with versions of consequentialism with dilemmas. Just off the top of my head: “Always act such that the consequences of your action have at least value X” leads to dilemmas when none of the actions available to you have consequences of at least value X. If you allow agent-relative evaluation of consequences, then the most natural dilemma-entailing views can be understood as notational variants of this.

            If you have an overly broad notion of consequences and their value, then whatever the determining factors of rightness can be understood as stemming from the goodness of consequences.

            Let X be the determining factor, such that an action is wrong if and only if and in virtue of having feature X. Well, one consequence of your action, understood in the broad sense, can be that you have performed an action with feature X. Assign this consequence negative agent-relative value, and ta da, you’d have a ‘consequentialist’ view (on the unrestricted understanding of consequentialism discussed above). You can do this with kant’s view and with virtue ethics, which are alternatives to consequentialism if anything is.

            This is why it is important to restrict what it takes to be consequentialist. There are a few ways people have tried to do this, but I think the one that best captures what’s distinctive about the paradigmatic consequentialist views is to limit it to agent-neutral value.

          • blacktrance says:

            “Always act such that the consequences of your action have at least value X” isn’t really consequentialism, because it’s central to consequentialism that more of the good is better and whatever produces the most of it is right (the latter is a sufficient condition, to not exclude satisficing) – if X is the good, it can’t be wrong to perform the act / adopt the rule / etc that produces the most X.

            Let X be the determining factor, such that an action is wrong if and only if and in virtue of having feature X. Well, one consequence of your action, understood in the broad sense, can be that you have performed an action with feature X. Assign this consequence negative agent-relative value, and ta da, you’d have a ‘consequentialist’ view (on the unrestricted understanding of consequentialism discussed above). You can do this with kant’s view and with virtue ethics, which are alternatives to consequentialism if anything is.

            If you do this, you don’t get Kant’s view or virtue ethics, you get a consequentialist theory that recommends the same acts/rules/etc as those theories. For example, if I refrain from lying to someone because I must treat them as an end in themselves, that’s Kantian, but if I add that such treatment is a good, and that’s what makes it right, then it’s no longer Kantian. (And it’d be self-contradictory, too.)

            There are a few ways people have tried to do this, but I think the one that best captures what’s distinctive about the paradigmatic consequentialist views is to limit it to agent-neutral value.

            But then you have absurd classifications like egoism not being a form of consequentialism, and more generally, no name for the natural category of views in which the right is solely determined by the good, where the good is consequential.

          • Philosophisticat says:

            @blacktrance:

            There’s a lot to say here, but I’ll start by pointing out that your own account of “consequentialism” doesn’t encompass “the set of views where the right is determined by the good, where the good is consequential”, because the view with dilemmas I just described is exactly such a view and you refused to characterize it as consequentialist.

            My definition, on the other hand, does, as long as we understand the good in an agent-neutral sense. And I think we should, because if we allow understanding the good as an agent-relative or act-relative notion, we end up counting views that are clearly nonconsequentialist as consequentialist.

            This is a case where people started with some paradigms for consequentialist and nonconsequentialist views, without thinking very hard about what it was that distinguished them, and where it turns out there are only family resemblances among the things we label as consequentialist. So the question is how best to precisify the distinction.

            The following view: “an outcome of an action A is bad if and only if and because it involves the agent performing A treating someone else as a means. Never perform actions with a bad outcome. So, never treat someone as a means” is a whole lot closer in the ways that matter to views that are paradigmatically nonconsequentialist than the views that are paradigmatically consequentialist. If we take the goodness of outcomes as a technical notion, then it’s just a notational variant of the Kantian view.

            It is true that on my definition, egoism does not count as a form of consequentialism. I accept that this is a cost in the sort of metalinguistic negotiation we’re engaged in. I think the lesson is that while egoism has some family resemblances to the paradigm, utilitarianism, and other views traditionally considered consequentialist have some family resemblances to utilitarianism, the things that other consequentialist views have in common with utilitarianism are not necessarily the things that egoism has in common with utilitarianism. Egoism’s commitment to agent-relative value, on the other hand, is a crucial difference between utilitarianism and views we take as paradigmatically nonconsequentialist. We draw the boundaries in the clearest and least damaging way by putting egoism outside the boundaries of the concept, rather than letting in virtue ethics and minor variants of Kant.

          • random832 says:

            @Philosophisticat

            You can come up with versions of consequentialism with dilemmas. Just off the top of my head: “Always act such that the consequences of your action have at least value X” leads to dilemmas when none of the actions available to you have consequences of at least value X.

            I’m not particularly impressed with the notion that my choice of what breakfast cereal to eat is a moral dilemma.

            Even for “substantive” choices, I don’t see a basis for necessarily attributing those consequences (or those disutilities, for the utilitarian variant on the same theme) to this choice rather than to the preceding choices leading up to that point.

  2. People who read Mike Huben’s anti-libertarian faq may be interested in my response. I include at the bottom a link to someone else’s rebuttal of my response and my rebuttal of that.

  3. ChelOfTheSea says:

    > Yes. But you never run into Stalinists at parties. At least not serious Stalinists over the age of twenty-five, and not the interesting type of parties. If I did, I guess I’d try to convince them not to be so statist, but the issue’s never come up.

    Well, this seems a tad ironic, in retrospect.

  4. Edmund Nelson says:

    Bryan caplan also has a reply here http://econlog.econlib.org/archives/2015/09/scott_alexander_3.html

    I make this post before finishing the whole FAQ to avoid being buried

  5. Thecommexokid says:

    In part A.3.1.1, you said

    But the past fifty years of cognitive science have thoroughly demolished this “revealed preference” assumption, showing that people’s choices result from a complex mix of external compulsions, internal motivations, natural biases, and impulsive behaviors. These decisions usually approximate fulfilling preferences, but sometimes they fail in predictable and consistent ways.

    So it would be better if you could make your point in part D.12.1, in which you say

    We can then work backward from your choice to determine how much you really value friendship relative to money. Just as we can learn how much you value steel by learning how many tons of steel we can trade for how many barrels of oil, how many heads of cabbages, or (most commonly) how many dollars, so we can learn how much you value friendship by seeing when you prefer it to opportunities to make money, or see great works of art, or stay healthy, or become famous.

    without falling back on the doctrine of revealed preferences that you have already rejected when it didn’t suit you.

    • Spookykou says:

      He is not saying that revealed preference is never true, he is saying that it is not always true which fits in nicely with the general theme of the essay “stop short circuiting complicated issues with assumptions that they are implicitly wrong based on assuming things like revealed preference, which are in fact demonstrably not universally-true.”

  6. Jiro says:

    I think I’ve put my finger on one of the big things that bothers me about this: It’s a Gish Gallop. People have already posted links refuting individual items in it, but if you refute any individual item, Scott or his supporters could just point out that even if the indivisidual item is wrong, the whole rest of the FAQ is still correct, so refuting the item didn’t do any good. And it would take too much time to refute the whole thing, especially without making any errors.

    If you go to the talk.origins archive, for instance, it’s long, but pretty much every single item in it is correct. Nobody’s going to be able to post links refuting five or six items in the talk.origins archive.

    • Jugemu says:

      A potential counter to this “giant list of nitpicks” style of argumentation is to try to draw a boundary around the details. For example, one could claim (rightly or wrongly) that, all things considered, libertarian leaning places do better than authoritarian-leaning ones.

      A better but harder way is to figure out (and hopefully agree) which details are the really important ones and focus on those.

      • Jiro says:

        A better but harder way is to figure out (and hopefully agree) which details are the really important ones and focus on those.

        Agreeing on which details are important, and agreeing that you only need to argue those, is equivalent to just striking the other items from the list. If Scott even had a reason to put the other items on the list in the first place, he must necessarily believe that those items should not be striken.

        • Jugemu says:

          Realistically he probably didn’t think that hard about “is this my true objection?” for each item, so even though he included each one for a reason, those reasons might not stand up to scrutiny in every case.

    • 1soru1 says:

      ‘Refute’ does not mean ‘disagree with’, and ‘Gish gallop’ does not mean ‘present a fixed list of numbered arguments’.

      • Jiro says:

        “Gish Gallop” means “present a list of numbered arguments, such that the size of the list makes it impractical to refute them regardless of their actual merit”. That applies here.

        • Montfort says:

          A gish gallop exists in context. It’s true that if something of this length were posted on a forum or as a comment it would be much longer than could reasonably be expected to be responded to. But it’s much shorter than, say, a book, and books are not intrinsically gish gallops just because they spend a lot of pages discussing an issue. People attempt to refute books all the time.

          • Jiro says:

            It’s true that if something of this length were posted on a forum or as a comment it would be much longer than could reasonably be expected to be responded to.

            How is a forum comment all that different from a blogpost with respect to that?

            Also, this list is point-by-point. Its structure is to make many relatively separate assertions, such that each one can succeed or fail individually. The nature of such a list is that in order to refute it you must refute each item on its own. Most books (unless they are books of essays) are structured differently, and can be refuted without trying to refute every example in the book separately.

          • Montfort says:

            Blog posts are typically much longer and are allowed to be more effortful. In a forum, or the comments section, people are engaging in dialogue directly with distinct time pressure on responses. A blog post is less like that and more like publishing an article or pamphlet, and invites further responses in the same idiom. The FAQs, I would argue, even more so, because they weren’t made as a single blog post that would fade away into the archives after a week, but as another node on raikoth.

    • JDG1980 says:

      The concept of “Gish gallop” only makes sense in contexts where the potential rebuttals are limited in length. Duane Gish hacked the debate format by rapidly making multiple statements that were all empirically false, but for which demonstrating the falsehood would take substantially longer than issuing the statements. This isn’t necessarily limited to formal debate; similar tactics can work on, say, TV talk shows. But it doesn’t apply in most contexts on the Internet. If you disagree with what Scott posted, you can post a rebuttal on your own blog (without any meaningful length restrictions) and link to it from here.

      I think it’s also significant that Duane Gish’s statements were, in fact, provably untrue. That’s not the case (as far as I can tell) with what Scott wrote here. Disagreement with Scott’s FAQ is likely to involve questions of historical interpretation, conflicting values, etc., none of which deserves the kind of moral opprobrium that Duane Gish got for lying.

    • Murphy says:

      What elements of the gish gallop does it have that it would be possible for it not to have while still covering the same points?

      In your view is it possible for any 30K word + text that covers multiple arguments to not be a gish gallop?

      • Jiro says:

        In your view is it possible for any 30K word + text that covers multiple arguments to not be a gish gallop?

        This blogpost is in a weird limbo where it contains multiple arguments that aren’t very connected, but the multiple arguments are all for the same thing. Most long texts that include multiple arguments are either arguing for several separate propositions (so that refuting one argument is useful), or are arguing for the same thing but are interconnected (so that refuting some of them calls the rest into question). These would not be Gish gallops. Long texts that don’t fall into either of those categories can still fail to be Gish gallops if they make claims so well-supported that it’s not even possible to make honest and plausible arguments against them, such as the talk.origins FAQ.

  7. P. George Stewart says:

    Great stuff, with lots of meat to chew over. I’ve just skimmed it for the moment. My commentarial offering on something you say early on:-

    This is the strain which, rather than analyzing specific policies and often deciding a more laissez-faire approach is best, starts with the tenet that government can do no right and private industry can do no wrong and uses this faith in place of more careful analysis.

    Well, it’s not like this is an idea that just comes out of the blue, like some harebrained notion that some hayseed came up with one day after being hit on the head by an apple.

    The quick and dirty way of answering this point is, that this is a natural response to the far more common delusion on which demagogues so often trade: that government is a wonderful omnitool with which we can do nothing but good if we can just get the structure of it right, or if the right people are in charge.

    We’ve seen how total government is shit, so prima facie it stands to reason that a medium amount of government is going to be medium shit, and a small amount of government is going to be a little bit shit.

    So either you have to argue that first appearances are deceptive, and at some point there’s some kind of Hegelian quantity-to-quality transition such that the effects of government somehow cease to be proportionately shit at that level; or that at some point down the line the trade-offs are worth it, for the sake of (things like, e.g.) moral reasons.

    • P. George Stewart says:

      Another cherry pick, re. Freedom and the Non-Aggression principle:-

      I think this has been miscast and misconstrued as much by its proponents as its detractors, because it’s actually quite subtle and tricky to capture; but there’s an intuition at the base of it that’s correct, and it’s even more intimately tied up with the idea of property than people think, and it goes something like this:-

      It’s not that aggression, the initiation of force, etc., is absolutely bad, but rather that aggression has to be justified, or at the very least in-principle justifiable. Think of aggression and interference as similarly disruptions of a system. What the core libertarian intuition says, is that what has to be justified is disrupting that system which is the coupled natural control of something by someone: hence “natural law”.

      The natural thing is the fact that human beings live by controlling things, shaping them, having designs on them and fulfilling them, etc. This is a process which involves the whole mind and body of the person, their rational faculties, their skills, etc. It is in fact the core dignity of what it means to be human (you can talk about all the nice smooshy stuff like relationships and all the rest of it, but animals have that stuff too, this is what’s specifically human, related to human dignity, the mind rationally operating, time-binding and transforming matter) To get an emotional feel for it (Dark Art-ing it here for a moment 🙂 ), think of an innocent kid happily playing with a toy, and then someone comes and BATS the toy out of their hand, grabs it and runs off with it.

      This natural process is a bit like a principle of moral inertia, and what requires justification is the disruption, the aggression, the interference. What doesn’t require justification is the process itself, which is innocent – until proven guilty.

      IOW, if the kid’s toy is a box with a nuclear button it looks like they’re about to accidentally press, then bat away – THEN you have justification.

      And that’s the famous “harm proviso”, and that’s how the harm proviso is tied in with the propertarian concept, which is really just “finders keepers” – that’s the plain man’s version of it. And if you think about it from a systemic point of view, if you want a rule that’s fit to co-ordinate EVERYONE in society, even those who are a bit thick, you need something even a child can understand. i.e. the simplicity of the rule is precisely its virtue – but that simplicity is capable of crystallizing a civilization of immense complexity around it.

      From a standing start, from a state of nature, if something is uncontrolled by anyone else, if there’s no ongoing natural relation of coupling between a person and a thing, then one may take control of that thing. One is not “depriving” anyone else of anything at that point, in moral terms, because one is not depriving anyone of some ACTUALLY EXISTING RELATIONSHIP OF CONTROL-OF-A-THING. The most that can be said that the thing could potentially have been controlled by any number of other people, and that’s true, but it’s not something that has any moral purport, there’s no there there, it’s just a cute, irrelevant abstraction – again, until and unless it’s possible to say that taking control of that thing would be likely to “do harm”, i.e. aggress or disrupt others’ dyadic couplings with bits of stuff.

      Now obviously, if you think of it at the highest level of abstraction, the first thing we find ourselves in control of in a state of nature, the first thing we “find and keep”, is our bodies. From some time in the womb, we start to develop this ongoing relationship of control with our bodies. That’s what makes that first freedom – freedom of life, freedom to flap our gums and make noises as we please (again, until and unless we do harm), etc. Then by extension, as we develop, and genetically connected with that control, we extend to control things, etc. (e.g. tools – e.g. walking stick as felt extension of body, again, coupled system, human dignity), then with those things we control other things, and so on.

      Meanwhile in the human realm (and because we understand we can do more with co-operation than we ever could alone) we can’t control people directly (because of the principle – and this is where the inalienability comes in, barring future tech, which ought of course to be forbidden under the vast majority of circumstances, we can’t directly teleoperate people), but nor can we just willy nilly snatch things from their hands, so the only moral course is to persuade them to VOLUNTARILY RELINQUISH CONTROL.

      So that’s the general picture: certainly property and freedom may be limited, but those limits have to be justified on the basis of harm, not on a balance of goods. The idea that freedom is a good on a level with or interchangeable with others is daft, it’s the very root of all other possible goods in the social sphere (because other goods come from people having those relationships of control of things, whether it’s building things you use, or freely falling in love with you). Freedom can certainly be limited: but it’s limited by the principle that human being and its extensions into matter (instances of control-of-stuff) is innocent until proven guilty, and may be interfered with or aggressed against only if it’s verifiably/falsifiably provable guilty in some sense, by being showable to be itself an instance of aggression (“initiation of aggression”) or interference, or some other analog of batting a toy out of an innocent kid’s hand for no good reason.

    • ChelOfTheSea says:

      > We’ve seen how total government is shit, so prima facie it stands to reason that a medium amount of government is going to be medium shit, and a small amount of government is going to be a little bit shit.

      That seems like a ridiculous argument. It’s like saying “well, drinking water constantly kills you, so drinking some water should kill you some and drinking no water should kill you none”. Things in systems of this size are very rarely linear in the way you’re implicitly assuming.

      > So either you have to argue that first appearances are deceptive, and at some point there’s some kind of Hegelian quantity-to-quality transition such that the effects of government somehow cease to be proportionately shit at that level

      You don’t need any claims that strong when you’re trying to extrapolate a trend line from a single point.

      • P. George Stewart says:

        The problem is you have to show that the government example is like the water example, the analogy doesn’t prove anything. The appropriate analogy could be poison or something like that.

        IOW, when I say prima facie I mean prima facie, it’s a reasonable assumption to make, because it holds in all sorts of cases, and we also have some empirical observation that shows the medium amount is medium shit too (e.g. social democracies of the type the UK had post-War till the 60s and 70s).

    • beleester says:

      I think this gets addressed by the “Government never does anything right” section, which points out quite a lot of non-shit things that the government does. Looking at totalitarian governments and concluding that all governments do nothing but add shittiness is leaving out quite a lot of data points.

      In other words, while total government is shit, medium-sized governments of the sort most countries have do a lot of non-shitty things. The FAQ also establishes that private-sector efforts also have shitty problems, which means that 0% government does not mean 0% shit.

      So following your reasoning, we can conclude that there’s a size of government smaller than “total” and larger than “nonexistent” which maximizes the gains for the amount of shit we have to put up with.

      • P. George Stewart says:

        Yeah I do think that’s a reasonable line to take, that’s what I mean by trade-offs.

        I was simply objecting to the “these swivel-eyed loons have this harebrained apriori template they’re forcing on everything” line – not that Scott does this quite, but it’s a typical way for people to dismiss libertarianism without looking at the arguments.

  8. Peter Gerdes says:

    Stalinists go to some great parties. In grad school it was always the really crazy humanities students (which always includes some ~30 year old pseudo-stalinists who hangs around) with the good parties.

    As an aside how do you write such long complex posts while keeping them basically readable for a large number of people. Anytime I try and cover that level of detail I end up with something that only mathematicians and philosophers can parse…largely due to the complex counterfactual logic but you seem to manage that here just fine.

    Is it that you just expand out to an even longer length? Or do you think your use of something like dialog form (in this case the interrogatives interspersed with your replies.)?

    • marvy says:

      maybe you could post a link to a piece of your writing and someone will chime in with an idea to clarify it. (doubt it, but could happen.)

  9. The problem with taxation is that it’s compulsory, whereas business patronage is elective. The government, unlike a business, does not have to ‘compete’ or make a ‘case’ for my money; it just takes it through force and spends it how it wishes. A business has to make a proposition as to why someones patronize it; a government does not. Make the government have to compete. Also, the money the government does get is often spent sub-optimally.

    • Ilya Shpitser says:

      Governments compete with other governments in places where there is relatively free movement of people.

      For example right now our good mutual friend Mr. Trump is causing all the smart Iranians to apply in Australia and Canada for graduate school (with the bring-back-the-Hapsburgs bleacher section here cheering up him). Because, I guess, it’s very important to protect us from all zero past instances of Iranian grad student terrorism repeating in the future.

      There are fixed costs to immigrations, but there are also fixed costs to switching a bank.

      And there are vast bits of US where Comcast isn’t competing with anymore.

      And there are lots of other solutions than more competition (oversight, for example), for both business and government — why suggest the competition fix as a reflex, every time?

      Also the Principal Agent Problem, which is one of the major things that makes governments terrible, applies equally well to businesses and non-profits. Simple solutions don’t work to fix it, which is why it’s a problem with capitalized letters in economics.

      But you knew all that, and still you chose to open with this, why? Why not try to argue on Scott’s level here.

      • IrishDude says:

        Governments compete with other governments in places where there is relatively free movement of people.

        It’s relatively easy to move to a new local government, and very hard to move to a new national government. That’s probably one reason local taxes are much lower than federal taxes, given the relative levels of competition.

        • Ilya Shpitser says:

          Sure. And it’s even harder to move with Iron Curtains in place.

          I haven’t written a long thoughtful essay on libertarians like Scott has. My TLDR problem with libertarians is they aren’t grappling, I feel, with the full scope of the problem of getting complex systems to do what we want. They sort of resemble Marxists in that way, actually, they want the Hegelian “world spirit” to always sort it out. But that doesn’t work in a long list of cases.

          There is no shortcut to getting complex systems right. Their study is in its infancy, I feel, but we need to grind this down over time with empiricism, and understand the laws, and design principles of complex systems. Sort of like game-theory/mechanism design on steroids.

          • Mediocrates says:

            Scott had one of my favorite quotes along these lines in his review of The Machinery of Freedom :

            Good governance is a really really hard problem. The idea that the solution to this problem contains zero bits of information, that it just solves itself if you leave people alone, seems astonishing.

          • IrishDude says:

            My TLDR problem with libertarians is they aren’t grappling, I feel, with the full scope of the problem of getting complex systems to do what we want.

            That’s interesting that you think complex systems are an argument against libertarians, as I think complex systems are an argument against government. In markets, decentralized knowledge among millions of participants gets aggregated through prices. A central planning bureaucrat that tries to set prices is highly likely to muck up the market, leading to shortages or surpluses, no matter how smart or well-advised the central planner is.

            Unintended consequences seems to be the rule when it comes to government policies, given the naivete or indifference to what happens when complex systems are tweaked by do-gooders (or bootleggers).

            There is no shortcut to getting complex systems right. Their study is in its infancy, I feel, but we need to grind this down over time with empiricism, and understand the laws, and design principles of complex systems.

            Isn’t this an argument for devolving power down to smaller polities, letting 10,000 flowers bloom, and seeing what works and what doesn’t?

            Scott had a post on archipelago that I’m highly sympathetic to as a libertarian. Such a set-up allows for experimentation, and for a variety of preferences to be satisfied with less conflict.

          • Ilya Shpitser says:

            IrishDude: not necessarily, some problems need central coordination. It’s hard, there are no general solutions.

            I broadly agree that empiricism is a great, time-proven way of understanding complex systems, though. So to the extent that archipelago type ideas let us do more empiricism, I am ok with that. In general, I feel people invented feudalism, and called it a day (as far as organizing complex systems). I think we can do better than feudalism.

      • Also the Principal Agent Problem, which is one of the major things that makes governments terrible, applies equally well to businesses and non-profits.

        Applies, but not equally well. The most obvious example, stockholders vs executives, is reduced by the possibility of taleover bids, and would be reduced much more by them if the government didn’t have laws to make takeover bids hard.

        • Ilya Shpitser says:

          Sure, and PAP applies less well in certain contexts to governments vs businesses because of townhalls, relections, various accountability and oversight mechanisms that are absent in businesses, and so on. I am seeing some of this play out live right now in Maryland, and across the country.

          It seems like a six of one, half a dozen of the other to me. Can you quantify “not equally well”? Nothing is friendly. Nothing is optimized to serve us.

          • I don’t know what counts as quantifying how bad one set of institutions is vs another, both imperfect. What I can say is that there is a general reason for bad results in both cases–the same reason. Individuals make choices which have costs and benefits. The more of the net cost goes to other people, the more likely it is that the choice that is best for the individual is not the choice that is best for everyone affected, him included.

            That situation exists in both the private market and the political market. But it is the exception in the private market, the norm in the political market. The individual deciding what to buy is bearing almost all of the resulting costs in the price–almost all because there may be some externalities, positive or negative, associated with producing what he is consuming.

            The individual deciding who to vote for, or the legislator deciding what to vote for, or the regulator deciding what regulation to make, or the judge setting a precedent, bears almost none of the net costs of his decision.

            I don’t think I can make that argument into a rigorous proof, but it seems like a pretty strong reason to expect the political system to perform worse than the market system.

    • Murphy says:

      The extremely rich have greatly relaxed restrictions on freedom to move wherever they wish. Many countries have some equivalent of an easier route to citizenship or at least residence for the extremely wealthy.

      The question them becomes: why do so many extremely wealthy people apparently freely choose to live in countries like the UK and US when there’s almost nothing stopping them from finding some state which charges close to zero tax or even finding some location with a very weak government like areas of Somalia outside the governments control? Simply hire some security contractors and go set up their own compound somewhere.

      That’s not rhetorical. What is the practical reason they do not? it’s within the power of many of these billionaires. If the alternative is hundreds of millions of their money being “stolen” why do they freely choose to continue with that status quo?

      Side note: a hypothetical libertarian society does not seem to solve the problem. So everyone gathers together with like minded people and sets up their own organizations and make their own freely chosen rules for funding the running of their own little societies and set rules for who can join from other societies. 1 generation later how is this any different from our current situation? you end up with a generation of people who’ve not freely chosen to live in the state in which they’re living and they have to contend with the immigration restrictions set by the local societies if they wish to migrate to somewhere that suits them better. 1 step forward 2 steps back.

      • IrishDude says:

        Because taxes aren’t the only thing that matters to people? For just one thing that keeps rich people where they are instead of single-mindedly pursuing the lowest taxation possible, there’s network effects. Rich people do business with people that are clustered in Silicon Valley or New York City, which anchors them to those particular locations. Other things people value are pleasant environments and living around people that share certain cultural values.

      • Whatever Happened To Anonymous says:

        Well, as evidenced by the recent Panama Papers situation, many of these people had been, until recently, successful in avoiding a large part of that burden.

        • John Schilling says:

          The Panama Papers evidenced mostly that a lot of rich people in Not-USA-or-UK were successful in avoiding a large part of the tax burden in the places where they lived. Which only strengthens the argument, in that rich people who leave the US/UK not only wouldn’t have to pay US/UK taxes but could use Panama-style evasions (which mostly don’t work against the IRS) against the lesser taxes of where they might move to.

      • Randy M says:

        You make a point that the taxes are obviously buying things that the rich obviously want in a society, but at the same time don’t a lot of rich have their income source located in another lower tax jurisdiction–Cayman Islands, etc.?

        • drethelin says:

          This is greatly exaggerated. The IRS aren’t idiots, and there are in fact a TON of provisions in in place to make this impractical for most.

  10. pipsterate says:

    I agree with most of this, but a few of the points seem questionable.

    5.1.1: The conventional wisdom among libertarians is completely different. I’ve heard of a study saying that people in the lower class are more likely to end up in the upper class than stay in the lower class, even over a period as short as ten years!

    First of all, note that this is insane. Since the total must add up to 100%, this would mean that starting off poor actually makes you more likely to end up rich than someone who didn’t start off poor. If this were true, we should all send our children to school in the ghetto to maximize their life chances. This should be a red flag.

    I’m not at all familiar with this study, but isn’t it possible that some of these people ended up middle class, or some other category? If someone says poor people are more likely to become rich within 10 years than stay poor, that doesn’t necessarily mean that >50% become rich. It could mean that 3% become rich, 2% become poor, and 95% become middle class, right?

    So I don’t know if it’s right to dismiss this position as “insane” without further explanation.

    I believe it’s perfectly justified to call it “false”, though, and I don’t have any objections/questions regarding the following two paragraphs.

    • Murphy says:

      people making the original claim tend to put in many disclaimers such that they measure in absolute wealth, not relative.

      So if someone starts off living under a bridge sleeping on damp rags they consider them to have moved to a “higher social class” if 10 years later they find them to be sleeping on fairly dry rags under a slightly nicer bridge instead. Since absolute wealth grows over time and individuals tend to learn to cope with their situations over time it pretty much loads the dice to get the answer they want.

    • Swami says:

      I have seen plenty of studies of class mobility and what they actually show is that over a ten year period there is extensive movement upward, and the movement is most upward, and by the highest dollar amount for those starting the study at the lower levels.

      The explanation is obvious. As households age, they tend to gain in experience and skill and thus they move up in classes up until retirement. New households are continuously joining from the bottom (new adults and immigrants) and older households exit via death. What we have is somewhat of an upward moving escalator for households.

      Annual inequality is thus an extremely deceptive number. Does it reflect the height and steepness of the escalator (arguably a very good thing) or differences in escalators between households based upon various factors (intelligence, education, parent SES, etc). Lifetime inequality would of course solve some of these problems, but it gains the problem of an eighty year delay.

  11. bysstah rhymes says:

    About libertarians being against labour unions, I dont think this is the case or should be the case. Just as companies should be able to cooperate in a free market, so should workers. If they arent allowed to cooperate there has to be some government interaction with the free market that is stoping this.

    I think libertarians being against labour unions is just about labour unions usually pushing for non-libertarian causes when they are lobbying against the government. The libertarians hate this. Non-libertarians looks at this and think libertarians want to ban unions. Which isnt the case. Or at least shouldnt be the case according to the libertarian principles about a free market. It isnt a free market if everything is allowed on the market that doesnt directly hurt someone except starting a association that has workers as members.

    Im not a native english speaker and its the first time im commenting in a thread here, so i hope your able to get the point im making here.

    • Swami says:

      I concur. Libertarians strongly support the freedom of workers to form any cooperative organization they like as long as nobody is forced to join and as long as the union does not use violence or threats of state violence to accomplish their aims.

      The issue, of course, is that in the real world, the latter are exactly how unions do in fact operate. IOW, unions in the real world and libertarians are often coming at strong cross purposes.

    • Nancy Lebovitz says:

      This libertarian, at least, is dubious about unions because they tend to use non-libertarian methods– blocking access to buildings they don’t own, and assaulting people who work without the union’s permission.

      • bysstah rhymes says:

        Of course this is the case in most of the countries in the world, but i would like to point to the case of the nordic model for labour relationships. In sweden, where im from, the early 20th century was very chaotic with a lot of wild strikes and alot of violence. This ended with the saltsjöbadsavtal, which was a labour market treaty between Swedish Trade Union Confederation and the Swedish Employers Association that sought to bring peace to the labour market by stipulating basic rules for the different partys of the treaty. It worked very well for a long time and to this day there is alot about the swedish labour market that should be well liked by a libertarian.

        Basically it stipulated that when there is a collective bargaining agreement between the labour union and the company, they arent allowed to use strikes or lockouts against a partner which it has a collective bargaining agreement with. It also stipulated that the employer was always the one who delegated the work.

        As a result of this labour market treaty, sweden has, in some regards, the most libertarian labour laws in the entire world. It includes: no minimum wage or any national collective bargaining agreement as in france, where a collective bargaining agreement is considered to be applicable to companies that hasnt signed a collective bargaining agreement. A company can pay its employees 1$ an hour and it isnt illegal.

        Stil, in sweden 90% of employees are covered by a collective bargaining agreement, how come?

        Well, in a free market, the unions are really powerful. Say a company doesnt want to sign a collective bargaining agreement, sure. There will be a strike, but the company wont mind, it has got a nice buffert and knows the employees that are on a strike will soon run out of money. But in countries like sweden were the unions have alot of members and are organized they will call on members from different labour unions to help. So the company in question will suddenly not be supplied electricity or have its waste bins picked up and it wont get goods delivered to it etc. pretty soon it finds it has to agree to the collective bargaining agreement or it will perish.

        This probably sounds pretty scary to a company friendly libertarian, but its actually the result of an extremely free labour market without much regulation. And the result is most of the times good wages for workers and good profits for the companies.

        Sure, sweden has walked away somewhat from this model of a free labour market, in the 80s we got new laws that made it very hard to fire employees and there is also very strict discrimination laws and work safety laws. But as a whole, i think it should really be used as an example for libertarians when ever someone says that the working man will be forced to work for scraps in a libertarian society.

        Another argument against it is that there was talk of government intervention before the saltsjöbadsavtal. So maybe it wouldnt have happened unless the government was pressuring the two parties of the agreement.

        Stil, it would serve as a good counter argument against anyone who would say that a libertarian is in some way not worker friendly.

        edit: sorry for the wall of text. TL;DR: Unions could come out on top in a libertarian society and its members be better of than in countries where there is a minimum wage.

        • random832 says:

          Basically it stipulated that when there is a collective bargaining agreement between the labour union and the company, they arent allowed to use strikes or lockouts against a partner which it has a collective bargaining agreement with.

          So the company in question will suddenly not be supplied electricity or have its waste bins picked up and it wont get goods delivered to it etc.

          This doesn’t seem to fit. If there can be no strike against the electricity company or the garbage company or any of the goods suppliers, how can the unions effect any of the things described here?

          • bysstah rhymes says:

            In the saltsjöbadavtal they did agree to not use force against companies that did not have a collective bargaining agreement, companies that didnt have a collective bargaining agreement didnt have the same protection as per the treaty, and why should they? they didnt sign the treaty, so they are bound by no treaty, so why should the workers of those companies abide by the labour market treaty which they are not bound? And if they are bound by the treaty, the treaty only stipulate that the workers are not allowed to use force against someone who is bound by the treaty. Hence forth the the companies not bound by a collective bargaining agreement will have to choose between joining the collective bargaining agreement or paying the price for not joining the collective bargaining agreement.

            I can see that this i a shity deal for the companies not bound by a collective bargaining agreement. But a worse deal is not having a peace treaty with the unions, and not having a treaty that stipulates that the company is the body that decide what is best for the company.

            its a compromise. why would the unions let go of the right to help one another? its the same argument u would probably use against why ordinary citizens would not care for one another. They do, and hence forth people wont starve. A collective is always stronger than the single citizen. So why should they signe such a deal with the Swedish Employers Association? unless its in there interest not to, and why should it be? They seem to be enjoying quite a bit of the upside of a collective bargaining agreement.

            So the result would probably be that in sweden, companies that didnt sign the treaty would probably be forced to sign a treaty.

            And this would not be a bad thing, as the treaty would only be a result of a free economy where the unions are allowed to make what ever demands they are able to make and the collective of Swedish Employers Association would only agree to such arrangement that suits there needs.

            I would agree that the Swedish Employers Association doesnt represent the entirety of the businesses of the free world, but this is a problem that is of the libertarian committee to answer and not of the rest of the world to answer as its a problem that libertarians seem to point to rather than other school of thoughts. Other schools of thoughts seem to think of it as a natural state of mind.

            Why should the majority of companies that will prospere from such a deal not agree to such a deal?

            And if so, why would the companies that doesnt prespere from such a deal not agree to such a deal if the result is bankruptcy for the company?

            Edit: i should probably point out that in the wording of the agreement, it didnt affect parties that didnt have a collective bargaining agreement, but this is all not according to the libertarian ideology. Government has nothing to do with it. Its all about pacta sunt servanda. Nonfulfillment of respective obligations of a contract is a breach of the pact. And should be punished as of the principles of the Night-watchman state.

          • bysstah rhymes says:

            should probably make it clear that there is a right in sweden to go on strikes for people other than yourself. This was a result of the saltsjöbadavtal, and was first used by the companies to a great extent to lockout workers of other companies that were being targeted by strikes in the early 20th century as a means to force the workers to quit there strike and go back to work cuz they couldnt afford to strike anymore. But again, it was deliberately left out of the agreement.

            Just as the workers side has made strides to make the labour laws less libertarian, the Swedish Employers Association has started arguing that this possibility for the unions should be limited by the government.

            But just as it was deliberately left out of the agreement when it was signed, as it was a great advantage for the companies side, the unions would now never agree to limit there ability to “sympathy-strike”, as its probably their greatest weapon in the labour market.

            And from the single companies view its also stupid, when the electricity workers sympathy-strike against a single company, the electricity company is stil able to provide electricity to 99.999% of its customers. 99% of contracts also include a force majeure-clause saying that if a company cant provide the service stipulated in the contract because anything outside there control, like a earth quake or a strike, it isnt responsible for the contract breach. This is the case in almost all of the countries and contracts in the world.

            The Swedish Employers Association could try to force the unions to accept an agreement that bans this sort of dealings, but that would only result in conflict across all sectors. So they wont try this, as the unions are to powerful to overthrow on this particular issue.

            Its really the free market forces driving this.

            And yes, it sucks for a new company that doesnt want to sign a collective bargaining agreement, as it will probably be forced to do this by the unions. But this is all according to libertarian principles about letting the market have a free reign.

            Its the same problem as with antitrust laws, where a new company would benefit from there not being different trusts working against new businesses, but for a libertarian, this isnt considered a problem.

          • random832 says:

            In the saltsjöbadavtal they did agree to not use force against companies that did not have a collective bargaining agreement, companies that didnt have a collective bargaining agreement didnt have the same protection as per the treaty, and why should they?

            I assume in this scenario that the garbage company, the electric company, and the suppliers all do have collective bargaining agreement. Why is refusing to do your job for them not considered force against them but only force against their customer?

          • bysstah rhymes says:

            I assume in this scenario that the garbage company, the electric company, and the suppliers all do have collective bargaining agreement. Why is refusing to do your job for them not considered force against them but only force against their customer?

            Its considered force, but its not covered by the labour market treaty and the stipulated ban on strikes when there is a collective bargaining agreement.

            So the two parties didnt agree to not use force against each other when the conflict wasnt about the two parties. They only agreed to not use force in regards to questions that was already agreed to in collective bargaining agreements.

            Its also allowed to make further going bans on strikes in a collective bargaining agreement, so the employer could pay the workers more for example in exchange for losing there right to “sympathy strike” or use solidarity action as its usually called. But this isnt very common as its one of the labour unions most effective weapons.

        • Tatu Ahponen says:

          Honestly, one of the things that America might need, for all the discusssions about Nordic welfare states (whether from the starting viewpoint of “Nordic welfare states are socialist paradise” or “Nordic welfare states are not actually socialist” or “Nordic welfare states are about to collapse from immigration and the impossibility of socialism”) would be a good history, written in a popular tone, about how the Nordic welfare states came to being and what sort of historical background all the individual mechanisms making up the Nordic model have. Would probably explain a lot more than people just looking at individual aspects of the model in isolation.

  12. IrishDude says:

    Scott, I’d be interested in hearing your definition of government (I skimmed over the FAQ so if I missed your definition in there, my apologies).

    There’s no difference between a town where to live there you have to agree to follow certain terms decided by association members following some procedure, pay dues, and suffer the consequences if you break the rules – and a regular town with a regular civic government.

    As far as I know there is no loophole-free way to protect a community against externalities besides government and things that are functionally identical to it.

    I posted this link by David Friedman on a recent OT, but it’s relevant here too: http://www.daviddfriedman.com/Libertarian/Capitalist_Trucks.html

    Trucks made by communists and capitalists seem very similar (4 tires, steering wheel, seats, engine, etc.). However, if given the choice people would prefer to buy capitalist trucks over communist trucks. This is because the incentive system of capitalism is to make trucks that are appealing to consumers, while the incentive system of communism was to meet quotas, such as tonnage of trucks, that didn’t really reflect the preferences of consumers. So it is with HOAs and governments, that seem similar, but have different incentive systems for how they are produced so that you’d expect HOAs to be more responsive to consumer preferences than governments. It’s an important difference.

    As a general reply to the rest of the post, I’d note that just because markets have problems doesn’t mean you should expect government to be better:
    *Myth of the Rational Voter describes why voters have poor incentives to be informed, and have systematic economic biases (make-work, anti-foreign, pessimistic, and anti-market) that should lead you to be suspicious that democracy will lead to better results than markets. Voting with your own money produces better incentives than voting in elections.
    *Public Choice shows that politics is a market too, with politicians, bureaucrats, and lobbyists pursuing their own self-interest, so critiques of free markets apply to critiques of government as well, such as externalities.
    *The Economic Calculation Problem shows that central planners suffer from a lack of information to make resource allocation decisions, due to the dispersed nature of knowledge, and therefore market prices are preferable to allocate resources as those prices aggregate the decentralized knowledge/preferences of millions of strangers across the world.

    I’d also like to plug Mike Munger’s unicorn governance argument here:
    “Unicorns, of course, are fabulous horse-like creatures with a large spiraling horn on their forehead. They eat rainbows, but can go without eating for years if necessary. They can carry enormous amounts of cargo without tiring. And their flatulence smells like pure, fresh strawberries, which makes riding behind them in a wagon a pleasure.

    For all these reasons, unicorns are essentially the ideal pack animal, the key to improving human society and sharing prosperity.

    Now, you want to object that there is a flaw in the above argument, because unicorns do not actually exist. This would clearly be a fatal flaw for the claim that unicorns are useful, if it were true. Is it?

    Of course not. The existence of unicorns is easily proven. Close your eyes. Now envision a unicorn. The one I see is white, with an orange-colored horn. The unicorn is surrounded by rainbows. Your vision may look slightly different, but there is no question that when I say “unicorn,” the picture in your mind corresponds fairly closely to the picture in my mind. So, unicorns do exist and we have a shared conception of what they are.

    When I am discussing the State with my colleagues at Duke, it’s not long before I realize that, for them, almost without exception, the State is a unicorn. I come from the Public Choice tradition, which tends to emphasize consequentialist arguments more than natural rights, and so the distinction is particularly important for me. My friends generally dislike politicians, find democracy messy and distasteful, and object to the brutality and coercive excesses of foreign wars, the war on drugs, and the spying of the NSA.

    But their solution is, without exception, to expand the power of “the State.” That seems literally insane to me—a non sequitur of such monstrous proportions that I had trouble taking it seriously.

    Then I realized that they want a kind of unicorn, a State that has the properties, motivations, knowledge, and abilities that they can imagine for it. When I finally realized that we were talking past each other, I felt kind of dumb. Because essentially this very realization—that people who favor expansion of government imagine a State different from the one possible in the physical world—has been a core part of the argument made by classical liberals for at least 300 years.”

    • Swami says:

      Excellent comment!

    • alwhite says:

      I am unsure what is being argued here. The governments of the UK, Canada, and Sweden are not unicorns. It seems like we have a pretty huge problem here if looking at real data from other real countries gets labeled and described as unicorns. I will refer to the Miscellaneous section of coming on too strong, thievery, and enslavement.

      • IrishDude says:

        I am unsure what is being argued here.

        Three things:
        1. HOAs and governments can look similar, but you can expect them to act differently due to their incentive structures.
        2. Markets having flaws doesn’t imply that government is better.
        3. Related to two, just because government in theory can solve a problem and make net improvements over a market, doesn’t mean government in fact does so.

        That governments exist doesn’t imply that government is better than the alternative.

        • 1soru1 says:

          What you seem to be saying is that while governments exist, and in some areas are verifiably better than actually-existing alternatives, it is possible to imagine a unicorn, so therefor anarchocapitalism.

          Except that anarchocapitalism may well be more of a Pegasus, in that it turns out a biological organism cannot provide enough energy to let something with that mass-to-wingspan ratio fly.

          There is no logical principle that says that an anarchocapitalism society must be able to exist. Maybe the collective intelligence of all of humanity simply isn’t enough to converge on a non-coercive optimized solution to certain problems.

          • IrishDude says:

            I didn’t make a positive case for anarcho-capitalism in my posts (I didn’t even mention the word!), so I don’t know how you arrived at your paraphrase. I just noted that if a case is being made for government instead of markets, more needs to be done than pointing out markets have problems since government has flaws as well.

    • daniel says:

      These are good arguments against a stalinist-communist state where everything is regulated by the government. While important, they are insufficient to detract from Scott’s points against a fully libertarian state, both seem to have unsolved systemic problems.

      I have no idea what Scott’s definition for government is, the one I always likes is “Monopoly of violence”. By this definition the difference between a hypothetical libertarian town and one with regular civic government. I the latter failure to comply will land you in prison (on pain of death, ultimately), in the former the case is… undefined? the default would be just refusing services thereby forcing you to move away, under certain flavors of libertarianism it might devolve into (justified) armed conflict, in ones where one may sign away one’s rights I suppose the two are indistinguishable.

  13. greghb says:

    I know you’re aware that all the figures are stale. Just FYI, the US federal gov’t budget deficit numbers have changed meaningfully — from ~$1.2 trillion down to ~$0.5 trillion, and you can find the data here:

    https://fred.stlouisfed.org/series/FYFSD

    Interestingly, this is a change larger than the article seems to consider to be reasonable to expect given somewhat insane targeted action, and I don’t think there was much targeted action. My guess is that the cause is largely “naturally occurring” decreases in unemployment insurance and increases in payroll taxes as the unemployment rate came down.

    • HeelBearCub says:

      My guess is that the cause is largely “naturally occurring” decreases in unemployment insurance and increases in payroll taxes as the unemployment rate came down.

      Also known as “automatic counter-cyclical spending is counter-cyclical”,

      Although there were definitely targeted actions, ones that increased the deficit counter-cyclically (temporary longer unemployment benefits, temporary reduction in payroll tax rate, one time stimulus spending). As an aside, you said payroll taxes, but you probably just meant revenue or perhaps income tax.

      The Bush tax cuts would also have expired during Obama’s tenure. That would have lowered the deficit by even more, but most of those tax cuts were made permanent. Some of them were allowed to expire, and that was another thing that lowered the deficit.

  14. Wrong Species says:

    I once got in to an argument with an anarchocapitalist about ethics. I set up a scenario where we knew for a fact that libertarianism would not only lead to a less optimal world, but one where the world would end up in a chaos. I can’t remember if he bit the bullet or refused to accept the scenario but I do remember telling him he would let the world burn to protect his ideology. He replied, “And you would enslave the world to protect yours”. The point is that telling a libertarian that they just need to be a consequentialist doesn’t usually work. And I can’t say I blame them. Accepting the “good kind” of theft is a hard pill to swallow, which is why most non-libertarians usually say that taxes aren’t theft and postulate an intangible, nebulous social contract.

  15. Swami says:

    COMMENT ON LABOR UNIONS

    I am not a libertarian, and agree with many or most of your arguments. However, I disagree strongly with both the facts and the arguments in your arguments on unions and class mobility.

    First on unions, I don’t buy into your framing of the issue that libertarian/ classical liberal arguments require something called “equal negotiating power”. The real argument is that over the long haul competition between employers is a great solution to most disagreements between employees and employers. Any employer not delivering what the employee has coming according to their marginal productivity, creates a strong system attractor for competing employers (or self employment) to fill the gap.

    Your argument about one person out of a million killing themselves due to job pressures is so bad that you should just delete it. Nobody anywhere in this debate is suggesting employee or employer relationships are conflict free. This is not a sound argument for government intervention as it lacks any benchmark. The real case is suicide rates in total in an environment with and without government interference.

    Next, libertarians are in no way opposed to coordinated employee unions. Most would strongly argue for their freedom to unionize. What they oppose is the power of the members to force employees to join. The reason is primarily because this would be unfair to prospective employees and customers. Libertarians don’t give a damn about employers, in terms of them gaining any negotiating privilege. What they do care about — reasonably so, in my non libertarian view — is the labor union creating a labor cartel which uses force to seek rents or privileges. And honestly, this is exactly what most unions do attempt to do, especially government service unions, and it is both unfair (to taxpayers, consumers and prospective employees) and arguably highly immoral (for those who valuing freedom, efficiency, and/or fairness).

    In summary, the libertarian opposition is not to voluntary unions of employees, which just about every impartial person would say is good, it is against coercive labor cartels which are unfair and harmful.

    As a non-libertarian, I support impartial government rules which extend a “right” to employees to form voluntary unions. I also support other limited government involvements in working conditions and terms, with an emphasis on limited. If a libertarian wants to argue that point, then I will be glad to engage them. But I found your argument in this section to be the weakest in an otherwise great FAQ.

  16. Cecil Harvey says:

    I’m a former libertarian. It’s so neat and clean and consistent and elegant — I’m a software engineer, and it really fits how I like to think of things, but ultimately I came to a number of the same conclusions as you did. I do think a lot of libertarian principles are pretty important for sorting things out, however:

    – All laws are backed by the threat of lethal force. I think this is an important principal that most people just don’t think about. I don’t mean to imply that the use of lethal force is never justified — quite the contrary. But someone says “there ought to be a law regulating that” I think, reflexively, “Is the enforcement of that law worth ending someone’s life?”

    – Non-libertarian societies, at the local level, can exist in a libertarian society. The inverse isn’t true. I tend to favor a much more minimal federal government, with more power owned by the state, more still by the county, more still by the municipality, and the most by the individual. That’s not to say the Federal government can have no power, but it should only really have power to deal with the externalities of the state that the state can’t reasonably deal with.

    – I like the idea of being able to “opt out” of society, social safety nets, etc. In general, I like to be left alone to my own devices. if one doesn’t wish to participate. The thing I dislike about the ACA is the universal mandate — I don’t like being forced to participate in such a thing. This, of course, leads to problems — if I’m opting out, but still living within a nation’s borders, I still enjoy the protection of that nation’s military, whether I pay taxes or not. However, if more were delegated to the local levels, I’d at least have my choice of municipalities to opt in to that would hopefully be radically different options from each other.

    • Luke Somers says:

      > But someone says “there ought to be a law regulating that” I think, reflexively, “Is the enforcement of that law worth ending someone’s life?”

      Some things seem less likely than others to escalate that far.

    • Jesse E says:

      “But someone says “there ought to be a law regulating that” I think, reflexively, “Is the enforcement of that law worth ending someone’s life?””

      Since other far more economically non-libertarian countries with more regulations and a bigger government have far less of a problem with the government killing citizens, it’s seems like the problem in America is a lack of training for police and easy access to guns, not regulations.

    • IrishDude says:

      I’m a former libertarian.

      What do you consider yourself now?

  17. Swami says:

    COMMENTARY ON CLASS MOBILITY

    As above, great FAQ overall, but my second major area of disagreement is over the mobility section. I disagree with the facts, the interpretation and your conclusions. To paraphrase you, this is indeed an empirical issue, and your data in this section is empirically wrong.

    It is absolutely not true that the poor and middle class within the US have not gained for the past 30 or so years. According to the CBO incomes have gone up between thirty and forty percent across the board. Here are extensive breakouts on the data:

    https://randomcriticalanalysis.wordpress.com/2012/09/08/issues-with-cbo-income-distribution-data/

    If you don’t like this source, just go to the CBO website itself . See page 37.

    https://www.cbo.gov/sites/default/files/112th-congress-2011-2012/reports/43373-AverageTaxRates_screen.pdf

    Sources suggesting that household incomes have not been going up among all quintiles are not adjusting for household size, marital trends, taxes and transfers, the increasing size of benefits, or they are not adjusting properly for inflation. And do note that the CBO doesn’t even adjust for the tens of millions of immigrants entering and pulling down the lower quintiles. The CBO is underestimating the improvement for non immigrants.

    But even more importantly, all this ignores the fact that households are not permanently assigned to quintiles. It is unusual for any household to remain in the same “class” over decade-plus long periods. The general trend, like a staircase, is for most households to grow in income with age and experience until retirement. New households and immigrants enter toward the bottom as the others tend to move up.

    Here is the key exhibit from the St Louis Fed showing that 57 percent of the lowest quintile move up to a higher quintile over the ten year time frame, and similarly a near majority of the top decile drop to lower income deciles over the decade. This is class mobility.

    http://www.stlouisfed.org/publications/itv/articles/?id=1920

    When they actually track actual households rather than abstract and constantly changing distribution classes, the data shows that real families in the bottom quintile gained an average of 20% in income over the latest decade studied, while those in the top one percent lost income over that time period. If this still seems contrary to what you have heard, it is because you are hearing about classes and forgetting that people are constantly changing between classes.

    Comparing incomes of deciles between countries:

    http://www.economist.com/blogs/graphicdetail/2013/05/daily-chart-17?Fsrc=scn/gp/wl/dc/betterlifeindex

    The rumors of a disappearing middle class show that it has been disappearing into upper classes:

    http://www.aei-ideas.org/2013/07/yes-the-middle-class-has-been-disappearing-but-they-havent-fallen-into-the-lower-class-theyve-risen-into-the-upper-class/#mbl

    And here is a chart comparing average individual consumption of the US to every other country adjusted to PPP. The US has significantly higher standards of living than everywhere else.

    https://www.oecd.org/std/prices-ppp/OECD-PPPs-2011-benchmark-Dec-2013.pdf

    In summary, the empirical data shows that there is still extensive mobility in the US, that most households get wealthier and move up in class over time, that the US has substantially higher standards of living than anywhere else. I could go on with data sets that reveal the US not only started higher but continues to grow faster than Japan and the larger countries in Europe. We have better than average unemployment, and the consumption standards of our lowest quintile look more like the median quintiles in Europe and so on.

    Finally, there is the suggestion that success in markets is due overwhelmingly to parental class. This addresses this issue. When you look into the data it shows that heritable intelligence explains a lot more than parental SES. Yes, smarter harder working parents tend to have smarter harder working kids and smarter harder workers tend to make a lot more than their opposite counterpart.

    https://randomcriticalanalysis.wordpress.com/2015/11/25/no-the-sat-doesnt-just-measure-income/

    If you want to define luck as “good genetics,” then so be it, but as any libertarian could explain, in a market economy every economic interaction is expected to be mutually beneficial. That means that those making the most, all else equal, are also likely contributing the most to others as well, and this is before we consider they also pay the majority of all income taxes (I support progressive taxation btw)

    I am not a libertarian, but in this section, I would declare your FAQ as empirically and rhetorically inaccurate. I suggest you consider rewriting this section. You probably want to remove the inaccurate allegation that libertarians preach “trickle down economics” too.

  18. Besserwisser says:

    I have my doubts to what degree libertarianism in the US currently is a reaction to overly restrictive governments. In addition to what the post already said, arguments against excessive state expenditures specifically go back at least as far as ancient Athens where Athenians criticized their fellow citizens as being greedy for wanting all that tax money in a manner eerily similar to libertarian arguments here. Mind you, this was about actually paying state employees who faced the death penalty if their duty was found wanting. And even then, many positions of power were still pretty much exclusive to the aristocratic elite whose critic of Athenian financial policies we have the most evidence of.

  19. elliot teperman says:

    While reading this article, I got the feeling a lot of it conveyed the general idea of how basically any one is sometimes useful, but lots of people want a cohesive system of ethics and say it should apply all the time (revolutionary, I know).

    This is kinda tangential, but for some reason I wondered about that maybe one of the reasons why we can’t seem to get a single cohesive principled system of ethics is for evolutionary reasons. We value fairness, avoiding pain, jealousy, love, etc for semi-arbitrary evolutionary reasons. This idea is pretty half baked, so i wanted to ask, can anyone point me in the direction of someone who says this better, or tell me why this is stupid. Thanks

  20. dawnedgrin says:

    Overall, I enjoyed reading this piece and it verbalized some of my ideas against libertarianism better than I ever could. There was one argument that seemed weak or hastily written though.

    Let’s focus on the last issue; a boss berating an employee, versus an employee berating a boss. Maybe the boss has one hundred employees. Each of these employees only has one job. If the boss decides she dislikes an employee, she can drive her to quit and still be 99% as productive while she looks for a replacement; once the replacement is found, the company will go on exactly as smoothly as before.

    I don’t believe a typical company can fire a typical, non-disruptive employee and maintain a proportional degree of productivity, because employees fit in one of several roles and may take on a wide range of responsibilities. To give an an extreme example, take a startup of 10 employees with 1 CEO, 3 developers, 1 QAer, 1 UX designer, 2 salespeople, 1 tech support. Losing an developer would cause a 33% loss of productivity; you can argue that everyone else is still working since not everything depends on code, but this is easily offset by the fact that you have to pay the other 9 employees their full salary to handle new functionality at 67% of the rate you were producing code before. And this doesn’t cover the case if the developer was an expert in an essential part of the field, halting all work until you replace him, or losing the QAer, UX designer, or tech support, who were filling the role for the company all by themselves. If a role’s unfilled, the employee who left is at 0% productivity, sure, but the company will quickly reach 0% and still lose money as they ostensibly pay the other 9 employees while producing nothing.

    I did say that was an extreme example, but any company you can think of will have different roles (the cooks and waiting staff in a restaurant dictate the flow of diners, gym facilities can only offer as many personal training programs as they have trainers, etc), and it’s not until you get to very large companies, over 1000 employees I’d guess, before losing 1 employee comes close to losing a 1/n portion of productivity.

    I agree that a boss generally has power and leverage against the employee, but it’s not as bad as Section 2.5 made it out to be, and that the company and employee suffers when they’re fired. The employee has probably near-equal power in small companies or knowledge-based work due to specialization and the company’s training costs.

    As a last point, suppose your company of 100 somehow consists entirely of worker ants I mean completely interchangeable, equally paid workers and you still manage to maintain a 99% productivity rate after firing a worker. If 99% productivity translates to 99% revenue, then that 1% of revenue is either more than your worker’s salary, or your company will eventually go under. Rationally speaking, the company would need some reason to believe the worker is responsible for less than 1% of the productivity before firing them, and that’s power to the employee that shouldn’t be waved off easily.

  21. Luke the CIA Stooge says:

    In regards to 14.1

    Canada ranks way higher on economic and personal freedom indexes that the US, with both the US and UK looking increasingly like hyper surveillance police states.
    Looking at the economic freedom indexes it does look like more economically libertarian countries (Canada, Australia, New Zealand) are less likely to be 1984 style giant panopticons.

    With the US and the UK looking more and more like big government busy bodies creating authoritarian societies just as a by product of wanting larger mandates so they can higher more people to get more prestige/justify a raise for the senior “managers”.

    • Controls Freak says:

      Every country has intelligence agencies, and people will find reasons to hate what they do (sometimes valid; sometimes not). I think you might have a false impression due to the fact that the US/UK intelligence agencies have gotten much more attention in the US press.

      • Luke the CIA Stooge says:

        And yet the US is the only country that’s set up infrastructure to collect and store absolutely everyone’s data (big bucks), and Britain has more state run CCTV in a random 4 blocks radius than the entire city of Toronto.

        Seriously Britain and the US have pulled far away from any other western country in terms of surveiling the populace and I don’t think it’s a coincidence that they’ve also fallen behind on other measures of economic and personal freedom.

        • Controls Freak says:

          I mean, you can just say that, sure. On the other hand, I linked to an article referencing a major Canadian program. Frankly, you don’t want to actually bring the rest of Europe in this, because they are way less privacy-friendly than we are (even if they’re not as good at it against State adversaries).

          Do you have anything other than ipse dixit to support your point?

          • Luke the CIA Stooge says:

            Controls Freak
            You seem to be missing the core issue here.
            I was responding to scott’s assertion that increased regulation and state intervention don’t seem to lead to orwellian societies the way libertarians predict.
            My point was that when you look at mass surveillance INFRASTRUCTURE, not isolated cases of the most corrupt police force in the country targeting critics or security forces breaking the law, but massive programs set up by the state to systematically surveil the populace the libertarian Anglo countries (Canada, Australia, NewZealand) are far less orwellian than their highly regulated counterparts (US and UK).
            I am discussing relative prevalence of surveillance. Not endorsing countries as paragon of privacy.

            Do you disagree with me: that there seems to be a correlation between lack of economic freedom and orwellian police state infrastructure in English speaking countries? And that this in contrary to Scott’s argument in 14.1?

          • Controls Freak says:

            If we’re restricting ourselves to US/UK/Canada/Australia/NZ and counting up INFRASTRUCTURE, then the former two will dominate the latter three in literally every category. They simply have six times the population and seven times the GDP, so it will always be easier to point to bigger buildings constructed by NSA/GCHQ. That’s a pretty terrible metric, though.

            > there seems to be a correlation between lack of economic freedom and orwellian police state infrastructure in English speaking countries

            I don’t think this has been supported by the above argument, and I think the restriction to English speaking countries is stupidly ad hoc. Plus, even if we look at something like the economic freedom index, the difference between Canada/UK is less than the sum of their one-year changes. I call that “noise”. We probably shouldn’t correlate noise with a totally subjective (and biased by obvious factors) guess at Orwellian-ness.

            > this in contrary to Scott’s argument in 14.1?

            Even if your correlation were true, it wouldn’t necessarily be contrary to 14.1. Scott is referring to the sum total of government intervention, not just economic freedom. I don’t know that we have good numbers which would distinguish your US/UK block from your Canada/Australia/NZ block.

  22. wipxgiii says:

    It no longer completely reflects my current views. I don’t think I’ve switched to believing anything on here is outright false, but I’ve moved on to different ways of thinking about certain areas.

    Am I too late to ask in what ways? If it’s too involved to explain in a short comment, I understand. Thanks either way!

  23. Rudbek says:

    2.1 – Not sure why you believe property rights in a lake can’t include water quality. It may or may not be an efficient way of dealing with the problem, but it is certainly possible.

    2.1.2 – There are existing fish share programs. I’m not expert in this area, but I’ve read that some have been successful and I’ve also heard quite a few examples of quotas and short seasons not being effective. To me the general resistance to trying property rights solutions to these sorts of problems seems largely ideological.

    2.3 – I’m surprised by the recurrent mention of boycotts. I never thought they figured very prominently in libertarians arguments. I also find the example weird. I’m trying to imagine what would be considered an ‘unspeakably horrible act’ that wouldn’t violate someone’s rights and thus be easily addressable by criminal law and or civil law.

    2.3.2 – I’m having a hard time coming up with examples of what’s unethical but yet is not a right’s violation. On the other hand, a number of US states have banned ‘price-gouging’ after natural disasters. Presumably, majorities (or a politically organized minority, more likely) felt price gouging was unethical. It seems to me the result of banning price-gouging is to make shortages more likely after a natural disaster. Mike Munger has written a good bit on this topic.

    3.0 – Granted people make irrational choices. It’s not clear to me that people are more rational when they vote or otherwise engage in political activity. To the contrary, a fair working assumption would be that people are more rational when, for example, buying a car for which they must live with the consequences than when voting for a politician who may or may not have influence over a decision about automotive standards that may or may not at some later point impact said voter.

    3.1.1 – Thoroughly demolished is a strong statement. Behavioral economics has contributed to our understanding of how people make choices but the choices people make still bear some relation to their preferences. Maybe I just like the idea that I’m the type of person who prefers almond butter to peanut butter. Or maybe I really do like almond butter.

    3.2.1 – This is a better argument for non-democratic forms of government than democratic forms. Why would choices made by irrational voters lead to enlightened rule by experts?

    4.1 – It’s important to understand that things like perfect information or perfect competition are simplifying assumptions in economics. Not always or often necessary assumptions.

    4.2.3 – But people will gather perfect information in order to make rational decisions when voting?

    4.3 – Small businesses predate big government. In my experience, local word of mouth about small businesses is a pretty strong driver of business. Social media is only making local reputations matter more. I find this point particularly backwards given how frequently big businesses have used regulations to hurt small competitors. See bootlegger and baptist.

    7.4.2 – “that means government, the only entity big enough to effect a nationwide change not just in behaviors but in social norms”. The only entity capable of regulating nationally. Not the only only entity capable of changing nationwide behaviors and social norms.

    8.- If the US had a free market in healthcare, this would be a relevant line of argument.

    12.2 – Taking something from someone by force (or fraud) is widely considered to be immoral. Protesting is not. There is an important difference between things that are wrong regardless of formal law (such as theft) and things that are only wrong because they are prohibited by law.

    Calling MLKJr a criminal may be technically correct, but it does not raise any interesting political theory questions. Calling taxation theft points out that something we all consider to be wrong is okay if the ‘thief’ has a flag. Libertarians are always struggling to make the initiation of force part of the political conversation. The taxation is theft slogan is one way of doing it. Not terribly effective but not so trite as you argue. The burden of proof should at least be on the person that says flags make doing otherwise immoral things okay.

    12.3.2.1 – It would be even weirder not to include fraud and burglary in the definition of force.

    12.4 – I’m no scholar of consequentialism but if you’re an ethical egoist is there much difference? Ayn Rand essentially said that morality is the rational pursuit of what you value (at least if what you value is some version of the Aristotelian good life). How would a consequentialist who values the same thing act differently?

    I’d be a lot more comfortable with a politician/consequentialist who has a very strong heuristic against initiating force than one who was willing to break a few eggs to make an omelet. What’s the value of a few million Ukrainian peasants compared to the success of the people’s revolution?

    13.6 – I’m not sure elaborate dystopian hypotheticals are much of a guide to real life morals. Lifeboat ethics are rarely applicable even in actual lifeboats.

    13.6.1 “The arguments that libertarianism will protect our values and not collapse into an oppressive plutocracy require certain assumptions: there are lots of competing companies, zero transaction costs, zero start-up costs, everyone has complete information, everyone has free choice whether or not to buy any particular good, everyone behaves rationally, et cetera. ”

    You seem to be referring to simplifying assumptions used in economic models. These assumptions aren’t necessary in economics and aren’t required in libertarian arguments either.

    13.8 – Pretty sure marketing sulfuric acid as medicine is fraud. Nothing unlibertarian about prohibiting that.

    16.1 – Agree. Libertarians hear it all the time. Somalia is a libertarian paradise? It’s okay to sell sulfuric acid as medicine…

    • random832 says:

      13.8 – Pretty sure marketing sulfuric acid as medicine is fraud. Nothing unlibertarian about prohibiting that.

      Are we talking about claiming that sulfuric acid is itself effective against some particular disease (or, say, as a disinfectant, and keeping in mind that today people will pour hydrogen peroxide in their open wounds), or claiming that some substance that is in fact sulfuric acid is some other substance that is a medicine? It’s not clear how you can prohibit the former as fraud without an FDA equivalent to evaluate claims of efficacy.

      2.1 – Not sure why you believe property rights in a lake can’t include water quality. It may or may not be an efficient way of dealing with the problem, but it is certainly possible.

      This point is clearly about “conventional” property rights i.e. dividing it up into geographical chunks and letting everyone do what they want within their own boundaries. Your thing falls into the category of “[extremely complex privatization scheme]”, which is addressed in 2.1.2. And your “back of the napkin” idea of complex property rights doesn’t even solve that – it just means there are 999 people who have standing to sue you for anything they unilaterally declare adversely affects their property (if your filter lets through even one drop of pollution they can decide they’d rather have you not compete with them), rather than a single entity that can objectively define how good your filter has to be.

      • Rudbek says:

        Sulfuric Acid – I was responding to Scott’s statement as it appeared to me on its face. If someone marketed sulfuric acid as a topical antibiotic with appropriate warnings about acid burns and such, then I don’t suppose it would be fraud. There’s a whole debate about whether the FDA should regulate for efficacy, but it didn’t read to me as the point Scott was trying to make.

        Lake Property Rights – Scott’s FAQ (which I enjoyed and learned from) is responding (mostly) to the dumber and overblown comments by some less sophisticated libertarians. As such I thought it was fair game to point out some of the exaggerations he made. Namely “(Lake) privatization doesn’t work.” All I claimed is that it is possible. Not that it is a good or efficient was to make it work.

        Digging into the idea, however, imagine a privately owned lake (they exist) – these private owners are capable of protecting the water quality of the lake they own. I’ve seen many privately owned lakes that prohibit gas/diesel powered boats. This is done for both noise and water quality reasons. The coordination problem with lakes that are owned by more than one person would clearly be more significant but not insurmountable. If the property right includes some sort of water quality aspect, civil law recourse to protect it, and the ability to trade that right away – then via the magic of the Coase theorem it wouldn’t be crazy to think the property owners would end up with a workable perhaps even an efficient solution. For all I know, it’s already happened. There’s a fun literature in economics of market solutions being pronounced unworkable in various scenarios only to find real world markets in those very scenarios.

        The scenario you describe (one drop) doesn’t really reflect how real world courts mediate property disputes on land, I don’t know why you’d think courts would suddenly be so unrealistic when their feet got wet.

        • random832 says:

          The scenario you describe (one drop) doesn’t really reflect how real world courts mediate property disputes on land, I don’t know why you’d think courts would suddenly be so unrealistic when their feet got wet.

          Because in the real world we have courts with the power of the state and the right to therefore declare that the alleged damage to your property from one drop of pollution is not really worth more than a trivial amount. In the libertarian world, it’s hard to see why anyone who’s really insistent that no-one should be allowed to pollute will submit to the authority of such a court, or regard it as different from a court that does not believe any amount of pollution does real damage to their property rights.

          Like, imagine a court that denies AGW saying that therefore I have no standing to sue when someone pollutes my air with carbon dioxide. This ultra-diffuse lawsuit regime is ridiculous enough when you assume it works, but why would anyone who generates CO2 submit to the authority of any court that accepts that AGW is real if they could choose not to?

          There’s simply no way to make a court system work with courts that can compete with each other on the basis of what laws they enforce (or what theories of standing/damages they accept, etc), and no higher authority to force them to agree, because each party to any lawsuit will naturally insist on the one most favorable to them.

          • IrishDude says:

            There’s simply no way to make a court system work with courts that can compete with each other on the basis of what laws they enforce (or what theories of standing/damages they accept, etc), and no higher authority to force them to agree, because each party to any lawsuit will naturally insist on the one most favorable to them.

            We have anarchy at the global level, with no world government. Yet, nations resolve disputes and create treaties and agreements with each other all the time. Global trade is extensive. International companies agree to settle disputes with certain arbitrators (i.e. courts). Nations submit to the World Trade Organization to settle conflicting trade rules between the countries with no supreme authority to enforce decisions. And related to the environment specifically, countries make agreements often. For example:

            “The United States and Canada have one of the world’s oldest and most effective environmental partnerships. The U.S.-Canada border includes four of the five Great Lakes, as well as many rivers and lakes, major airsheds, and migratory routes for wildlife species. In addition, there are many U.S. Native American Tribes and Canadian First Nations residents whose culture spans the border.

            The extensive border and the considerable and diverse geography of the ecosystems shared by the two countries requires close cooperation among many U.S. states, Canadian provinces, U.S. Tribes, First Nations, and local and federal governments. The two federal governments have implemented over 40 international agreements for the management and protection of environmental quality and ecosystems in the border area and there are over 100 additional such agreements between U.S. states and Canadian provinces. The two countries also share policies, programs, and goals to prevent and control pollution and to ensure sound policies and practices to protect and restore the many shared ecosystems.”

            There’s no higher authority to force Canada and the U.S. to enforce their environmental agreements, so it seems that according to you there should be no way to for them to have effective environmental agreements. But they do, so perhaps you’re missing something in your analysis.

          • Rudbek says:

            “In the libertarian world, it’s hard to see why anyone who’s really insistent that no-one should be allowed to pollute will submit to the authority of such a court”

            Huh? In the libertarian world, failure to comply with a court order has the same result as the real world.

            “There’s simply no way to make a court system work with courts that can compete with each other on the basis of what laws they enforce…”

            Oh.., your objection is to anarcho-capitalism. I’ll let IrishDude handle that (or David Friedman who sometimes comments here).

            Just let me say that small-government/watchman state/monarchism libertarianism is a far more predominate viewpoint than anarcho-capitalism. I don’t think your objection applies to a small government libertarian regime.

            What I find confusing about the lake example – is that as an attempt to describe a commons that can’t be effectively privatized it’s not very convincing. Private lakes exist. Fresh water aquaculture exists. It’s not at all hard to imagine a lake with many parcels and a private property enforced water quality regime. I’d be surprised if any libertarian found the example convincing.

            As you (Random832) point out, there are much better examples. It’s hard to privatize the commons relevant to air pollution. There are effective market-like regulatory programs (SO2 in the US, for example) but that’s a far cry from privatizing the relevant ‘commons.’

  24. JPNunez says:

    As a person who dislikes Libertarianism, I have to point out that 14.1.1 is dangerously naive.

    We are currently seeing how an elite in power is going to strip away the rights of immigrants, and after that will move to the rights of trans people…for no good reason at all? They ain’t gonna be richer due to this? as far as we can see, they are basking in the glory of keeping foreigners out, and they will be basking in the glory of humiliating trans people even though they do not derive any visible benefit from either.

    Yes, I get that they built their electorate to answer to these movements, but we have to worry about the constant, always prevalent hate mongering against LGBTQ people some groups -sometimes religious but not always so- have developed. Other groups have developed seemingly arbitrary hates for certain foods, behaviors, opinions, _art_ and have shown that once in power they will move to codify these hates as laws. Of course it may not happen in sweden…except that several European countries have been seeing a rise in far right wing parties that could easily see arbitrary cuts in freedom happen in an otherwise democratic way.

  25. trapexit says:

    “Many economic theories start with the assumption that everyone has perfect information about everything.”

    Citation needed. I’m unaware of any free market economic theory which makes such claim. It is usually, explicitly, the opposite. The Austrian and Chicago schools explicitly note that information is never perfectly known by anyone and that its the market itself which helps disseminate information.

  26. twothings says:

    1. Regulation

    There’s a difference between “regulation” that ensures that you get the product you think you get and regulation that keeps you from getting the product you want.

    2. Children

    The government forces children to stay with their parents. If a child runs away the police will bring him back. As children aren’t allowed to vote, but parents are, I don’t expect that to change.

    In a libertarian world, children could move to whomever they felt treated them best, or live on the street if they felt that that was still better then living with their parents, or work …

  27. philosophicguy says:

    Scott, you really missed the boat on the non-initiation of force. And given that this is the essential foundation, the philosophical keystone, of entire libertarian position, your whole anti-libertarian argument crumbles on your failure to get this right.

    The reason for prohibiting the initiation of force in human affairs is not about “self-ownership,” it is because force is fundamentally anti-mind.

    Another way to say it is, force invalidates the products of reason. Another way: Force prevents human beings from acting on the conclusions that they reach by reason.

    And since reason is our basic tool of survival–telling us what is right and wrong, what is food or poison, which action leads to life and which action to death–when someone interposes his brute force between your reasoned conclusions and your actions, he cuts off your means of survival. (This is why societies built on the initiation of force–totalitarian dictatorships–often result in tens of millions dead).

    Let’s look at some examples:

    Reasoned conclusion — [force invalidating the conclusion]

    1. I think I should marry this person — [no, you may not, because you are both female]
    2. I would like to become an architect — [no, you may not, because you are from the wrong caste]
    3. I think I should work my land and use the products to feed my family — [no, you must hand over your crops to the government while your family starves]
    4. I think I should have sex only with men I love and trust [no, you will have sex with me because I have a knife to your throat]
    5. I think I should ingest this plant — [no, you may not, because some puritans fear that you might have a good time]
    6. I think I should leave this hellhole of a country and go somewhere that allows me to act on my own reason [no, you may not, because our border guards will shoot you if you try to leave]

    Those are simplistic, but they make the point.

    Human survival and flourishing depends on being able to act in accordance with the conclusions we each, individually reach by reason. When force blocks us from acting on our reasoned conclusions, we suffer and sometimes we die.

    THIS is why the initiation of force must be prohibited in human affairs. Ayn Rand pointed this out more than 50 years ago:

    “To interpose the threat of physical destruction between a man and his perception of reality, is to negate and paralyze his means of survival; to force him to act against his own judgment, is like forcing him to act against his own sight.”

    Another excerpt from Rand:

    “No man—or group or society or government—has the right to assume the role of a criminal and initiate the use of physical compulsion against any man. Men have the right to use physical force only in retaliation and only against those who initiate its use. The ethical principle involved is simple and clear-cut: it is the difference between murder and self-defense.”

    And another excerpt from Rand:

    “Man’s rights can be violated only by the use of physical force. It is only by means of physical force that one man can deprive another of his life, or enslave him, or rob him, or prevent him from pursuing his own goals, or compel him to act against his own rational judgment.

    “The precondition of a civilized society is the barring of physical force from social relationships—thus establishing the principle that if men wish to deal with one another, they may do so only by means of reason: by discussion, persuasion and voluntary, uncoerced agreement.”

    The same reasoning is the philosophical foundation of individual rights. Reason operates at the individual level, and individual rights ensure that each person can act on the products of his or her reason, without compulsion by others.

    Or at least they used to. Back when we still enjoyed individual rights.